You are on page 1of 294

160070

>m
73

da.ll ^au

Mr* JMO,

OSMANIA UNIVERSITY LIBRARY


*"*

^v ^

i5 &* jv^^

Accsession

Author
Title
"^

This bo9k should be returned on or before tfe date

last

marked beWr

A TREATISE ON HYDROSTATICS.
BY
JACJIT SINGH
M.
A., F.

R.

s. s.

CONTENTS,
Pago
Historical Introduction.
...

..

(i)

Chapter

I.

The Fundamental Property


Chapter

of a Perfect Fluid.

11.
...

Equilibrium of Fluids

...

10

Chapter

III.
...
,

Fluid Pressure on Surfaces

47

Chapter IV.
Equilibrium of Floating Bodies
..

!)2

Chapter V.
Gases
...
...
..

140

Chapter VI.
Capillarity
...

...

...

182

Chapter VII.
The Figures
of Equilibrium
of
...

a mass

of

Rotating Liquid

240

Errata

Page

Line

For

Read

Tbe beigbt
163

of

Ji

above

D
u

ia

36

iuchi-s.

Historical Introduction.

The foundations
as early as
treatise

of the science of Hydrostatics were laid

down

250

B. 0.

by Archimedes (287-212

0.) in his

on Floating Bodies.

He

formulated the theorem that

pressure at any point


in
to

in a fluid assumes and preserves a position of equilibrium, arid proceeded so far as to give a of tho positions of rest and complete investigation slablility of a right segment of a parabloid of revolution wholly above or

every direction. which a solid b:>ly floating

of a fluid mass in equilibrium is the same He inquired into the conditions according

below the surface of the

an investigation which has boon fluid, characterised as a "veritable tour de force" by >ir Thomas Heathy tho historian of Greek mathematics.
In spite of the brilliant
arid

sweeping range of Archimedes'

researches, the science of Hydrostatics made no progress at all for a long time to come. For, as Hogben has pointed out, Grook science

had reached a stage when having been divorced from the social heritage of the age it had degenerated into a hobby of tho unemployed intellectuals of a leisured class. Long long after when the Reformation had stimulated interest in the practical problems of craftsmen and mariners, the study of Hydrostatic was resumed. By the beginning of the 17th century a few hydraulic;
machines such as the forcing pump and the siphon had boen invented. In 1603, Marinus Ghetaldiw in his 'Promotus Archimedes made the first serious attempt to apply the principles
1

already enunciated by Archimedes more than eighteen centuries But the laws of equilibrium of fluids were for the first ago.

time demonstrated in

the most

perspicuous

and simple fashion


Sur
1

by
this

Blaise

Pascal in his

posthumous work,

Kquilibre
of the

des Laquers.

The Galilean

spirit of scientific enquiry animating

work coupled with ample experimental confirmation

* P, 91 History of Greek Mathematics Vol II by Sir T, Heath

laws enunciated invested Hydrostatics with


science

the dignity of a

The next considerable advance in Hydrostatics is associated with the name of Torricelli, who invented the barometer and
discovered that atmosphere exerts pressure. Here again Pascal carried on Torricelli* B work a stage further. It was at his

suggestion that his brother-in-law, M. Percier, performed the experiment of carrying a barometer up the Pury de Dome in

Auvergne and noting the

fall

of the column of mercury

as the

atmospheric pressure decreased. The problem of equilibrium of floating bodies was taken up by Christian Huygens iu 1650 after the epic making work
of Archimedes on the

same

subject,

Huygens

treated the problem

by a new method based on the principle that in equilibrium the centre of gravity of tho whole mass of the floating body and of the liquid is as low as possible. A century later two
geometrers
the
his

occupied

themselves
in

with

this

problem

about
in

same time; Bouguer


Scientia

his

Traite du Navire, and Euler,

N avails.

Dupin

introduced the

surface of floata-

tion

and completed the theorems of Bouguer and thus gave the principal general methods now-a-days in use. His memoir with
the report of Cariiot

reproduced in "Les Applications Geometric et Me'caniqiie, par Charles Dupin." Bouger was the first to
is

study the question of the stability of equilibrium of floating bodies; but he only considered the particular case in which the

displacement was such that the volume displaced remained constant.

ing.

Duharne^ showed the insufficiency of Bouguer's reasonHe attempted to complete the study of the small movement

of the system in the neighbourhood of the position of equilibmade the arbitrary assumption that in rium; but he this movement the pressures can be determined according to

the laws of Hydrostatics, Nevertheless the conditions of stability of equilibrium, which ho found, are exact. The hypotheses made
^i)

Uuhamel

Note sur divers Principles de Meeanique, observations Sur la Stabilite de 1'equilibre des corps flottants (journal d'Etole Poly technique XXIV Cahier 1835^.
:

iii

by Duhamel were
taring neglected

criticised

by Clebsch a who saw that certain


\v

by Duhamel
Besides,

ere of the

same order of magnitude


did not give

as others retained.

Duhamel

under one

simple form the necessary and

sufficient conditions of equilibrium.

rigorous exposition of the theory of the stability of floating bodies was given by Guy on 3 following an idea put. forward by Bravais in his thesis. This theory which rests on
first

The

principle as that of Huygens and which is rendered rigorous by the theorem of Lejeuiie-Diridhlot is supported by a

ihe

same

simple and elegant geometrical demonstration. A very detailed history of the problem will be found in a memoir by Duhem 4
.

question of the same category but much more complex, equilibrium of a ship floating in a changing liquid, has been the object of recent studies by Guyou, Duhom, Greenhill and
Appell.

Further progress in the fiold of Hydrostatics during recent years has been almost exclusively concerned with the determination of shapes

assumed by stationary

or

subject to their

own

gravitation.

This problem

rotating fluid masses is of particular

Astronomy and Cosmogony. So voluminous is the work done on the solution of this problem that its study has by now become a separate branch of higher mathematics, A
importance in
historical account of this subject will be

given

in chapter VII.

(2) Clebsh

Uber das Gleichgewicht schwimmender Korper, (journal


iS6oJ. Theorie nouvelle de
la Stabilite

dc

crelle, 57,
:

(3^ Guyou
(4)

dc

1'tquiiibre

des corps

flottant et Theorie du Navire.

Duhem, Sur

la

Stabilite de
5

P equilibte des, Corps flottant


t.

(Journal

de Mathematiques,

serie,

I;

1895.)

CHAPTER
The Fundamental Property
1
1.

I.

of a Perfect Fluid.

It

is

matter so far as

and gaseous.
fluid.

threo states viz. solid, liquid Matter in tho last two states is also referred to as
stale*
?

known fact wo know exists in


a well

of

common

experience that

What

distinguishes the solid from the fluid

is supposed to be rigid e. to have and volume, which are not altered by the action shape of any external forces. Yet, even tho hardest solid, however succumbs to the action of force if sufficiently strong. In rigid,
-/.

Matter in the solid state

definite

practice,

therefore,

there existn

no perfectly rigid body;


Still

solid

purposes of analysis perfectly rigid body as one which does not yield to the action of any force i* e. tho

bodies are rigid

only to an

extent.

lor the

mathematical

we

visualise

particles

constituting

it

retain

tho

relative

distances between

themselves unchanged even when it is aeted upon by any force, however groat. Let us now consider a small plane area insido
a body.
R.
to

on each

body on either side of it exert and opposite action. Let this action bo equal Resolve it into two components, T in the plane and S normal it; then T is called tangential or shearing stress and S normal
portions of tho

The two
other

stress.

called

The deformations produced by tho action of stress are The effect of the shearing stress (T) is to cause strains.

the portion of the body on either sides of tho piano area to slide over that on the other side; this deformation is known as shearing strain e. g, the action of a pair of scissors in cutting through

a piece

be mentioned in passing that the normal stress (S) is either one of tension or compression according as the two portions of the body on either sidu of the plane tend to exert a pull or a thrust respectively on one another. It follows
of cloth.
It

may

at once that a perfectly rigid

body can

resist

shearing stress to

au

indefinite

extent

On

the other hand fluids

i. e.

'substances

we have
If a

classed as liquids
is

and gases have very

little

cohesion.

plane

resistance.
air is

passed through water or air ifc meets with little Hence shearing stress between the parts of water or

very small. Just as for purposes of mathematical analysis we evolved a definition of a perfectly rigid body, so also we define our ideal or perfect fluid as one between the various parts
of which no shearing stress can exist.
fluid is

In other words, "a perfect


to the

an aggregation

slightest effort

made

to

which yield at once separate them from each other."


of particles

Consider
mass.

The

a small plane area round a point action of the fluid mass on one side of

now

in a fluid

it is

a force

R, having components T along the plane and S normal to it. But by hypothesis, since the mass is a perfect fluid, T=0 hence the direction of R is normal to the plane. The action of the fluid
f

mass on a small plane area is always normal to the plane area. Thus the fluid pressure is normal to any surface with which it
is

in contact.

Actually however,

all fluids
e.

do more or less

offer

resistance

to separation or division

a jug of coal tar be poured over a slab, it takes much longer to spread over the surface than water. This is due to difference in the degree of viscosity of the fluid.
g. if

We can amend our definition of fluid


fluids of all

in such a

way

as to include

degrees of viscosity. "A fluid is an aggregation of which yield to the slightest effort made to separate them particles from each other, if it be continued long enough". In a viscous
fluid.

fluid at rest

perfect

there can be no shearing stress Hence the fundamental


is

as in the case of a

property of a perfect fluid


rest.

proved above

also

that of a viscous fluid at

It follows

that all propositions in Hydrostatics are true for all fluids what-

ever their viscosity.

REMARK
are

known

Fluids which are easily compressible as gases; while incompressible fluids e. g.


:

e.

g.

air,

water or

mercury, are known as liquida

A prefect

liquid is a perfect

fluid

absolutely incompressible stress (S) to an indefinite extent. normal


1'2.

which

is

e.

which can

resist

Pressure.

Consider an area

a fluid. posed inside is a force R the fluid


fluid

of a plane surface exhave already seen that the Action of normal to the plane. If jhe action of the

We

on

be uniform, then ~

is

the

thrust jper unit

thrust per unit


surface.

ami

is

called the

pressure at

JFhis each point of the

area.

When

the thrust on a surface

is

not uniformly distributed,

then the pressure will vary from point to point. Consider an area A round a point P indefinitely small area $A of tho plane

and

let $11

be the thrust on

it

Then

is

the

average
tends to

force
zero,

when per unit area. In the limiting caso * ~ ultimately tends toy? P tlien P J

$A

SA

dA

defined to be the

measure of pressure
1*3.

at the point

P.-

The

pressure at any point of a fluid at rest

is

the

same

in every direction.

be any, point in the fluid us take any three mut ually as axes of coperpendicular lines ordinates. Consider a small tetrahedron formed by the fluid mass enclosed by the 3 coordinates planes and the plane ABC. Let p {J p.2 p$ be the pressures on the 3 faces IlOU, CO A, AOB and q, the pressure on

Let

and

let

ABC.
Let

vV

be the area of the triangle ABC, and Z, m, cosines of the perpendicular from to the plane be the length of this perpendicular.

n the

directibn

ABC

and

let

Then
AZ, isi

areas of the triangles

BOC, CO A and

AOB

are respectively

Aw, 4

and An.

Also the volume of the tetrahedron

OABC

Now
OX.

since pi
jp a

is
I

face is

the pressure on the face BOO, the force on this acting normally to the face BOO t e. parallel to
.

Forces on the faces

AOB, and
face

AGO

have no component

parallel to

parallel

to

OA. Force on the OA.


fluid

ABC

has a compoaeiafe,-Tr<7

Let the

be at rest under the action of fotfJJIJ^X, Y, Z)

per unit mass.

Hence the tetrahedron under consideration


under the pressures normal
to

is

in

equilibrium

the

4 faces of the tetrahedron

and the extraneous

forces.

Resolving along OX,


jpi

we have

A -qA
I

Z+J
is i

*X=0,
of the volume of
h,

P being the density of the

fluid, for

the mass

the fluid in the tetrahedron

Hence

Now

let

(l) (PI -j) us diminish the size of the tetrahedron by letting the

H4

X A=

plane ABO move parallel to itself towards 0. Ultimately when the tetrahedron is reduced to almost a mere point, h becomes

an

infinitesimal

and may be neglected

jpi-2 = o

in equation (l)>

hence

or,

Similarly

i Hence, But as the three directions OX, OY,

= 2p => 2 =p 3 = ?
p*
*=

p i=

q,p $

it

arbitrary follows that the pressure p at along any direction is the same. Of course at any other point, P in the fluid mass the

OZ

are perfectly

pressure may rence always


fluid mass.
1*4.

be very different from what

it

is

at

0, this

diffe-

depending on forces which act bodily on the

Density and Specific gravity.

Density of any homogeneous substance is the mass per unit be the mass of a body volume of that substance. Hence if

whose density

is

d and volume V, then

d~

or,

-y%

of a substance is the ratio of the The|pecific gravity weight oFany volume of the substance to the weight of an equal volume of some standard substance. Usually this substance is
distilled
It is

water at

4C in

the case of solids or liquids.

evident that specific gravity of a substance is the ratio of the densities of the substance to that of the standard substance.

where,

density of the substance,

d'=
/.*=&*'.
Since in the C. G.
S.

standard substance.

system of units d
its

'

1,

specific

gravity of a

substance
If

is

equal to

density.
is

W be the weight of a body .whose volume


is S,

and

specific

gravity

then density = Sd /;=

Hence
1*5.

W = Sc2'V.
# 2 v 3J ... be the volumes of the different substances, f s 2 , s s ... be their specific gravities, and let d be the
, ,

Specific gravity of Mixtures.


t>i,

Let

and

let a lt

weight of unit volume of the standard substance. volume of the mixture and s its specific gravity.
there

Let v be the

Assuming that no chemical action takes place and is no change in the volume,

therefore

vVi +v

+t? s

..........

(1)

Also weights of the various constituents of the mixture are

Hence the

total

weight of the mixture vsd must be equal

'

to the

sum

of the weights of its constituent,


Vt s {

or

*=

*i S
I

V2

!"

If.

however, there occurs a diminution in the

total

volume when
different

the substances are mixed so that the actual volume of the mixture
is

a fractional part k of the components we have

sum

of the

volumes of the

Honce ,=

k 0,

+Vt+

Example

1.

If the specific gravity of pure milk be 1.031

and

of an adulterated specimen water (i. e. the number of gallons of water whioh

1.024, find the percentage

of added must have

been addud

to

100 gallons of pure milk).

of

Let us suppose x gallons of water are added to 100 gallons 1.024. pure milk to have the specific gravity
Total volume of the mixture
"Total weight of the
/.specific gravity

= ]00-f-# = 1.024

where d

is

the wt. of a unit volume of water.

/.a? = 29.17.

Example
removed
density
a.

2.

From
its

a vessel

full
it

of a liquid of density
is filled

is

th of

contents and

up with a

liquid of

If this operation be repeated

m times find the resulting density


volume
v,

in the vessel.

Deduce the denisty


with liquid of density
has dripped into
it

in a

vessel of

originally filled

P,

after a

volume u of liquid of density P

by

infinitesimal drops.

Let

be the original volume of the liquid;


cr;

has been remo-

n
hence the mass of

ved and replaced by a liquid of density


n'
w

n
the
,1

<

.,

A.

/.</,

density after

* first
.

operation

mass =22 of the mixture


t

Similarly,

rf

2= density

after the second operation.

-(!--VT+v n' n

n
80,
</ m

n
operation

= density after the completion of the m th

...........

(0

Cor.

If

and n be made

to tend to oo

go

that -^ , an inn'

finitesimal quantity, is

removed

m times
m

where

v=u, we have
>

Lt

u/v,

when

and n

--->

Hence from

(1) the density required is the limit of

when m, n tend

to

>

in such a

way J

that Lt

n
(l

=E
v

Hence, required density = P e

u/v

u/v
a
e
)

1*6.

Density of a heterogeneous

fluid

Consider a point P of a fluid in which density varies from Consider $ m an elementary mass of the fluid point to point. round P. Let its volume be Sv, then if P be the average density of the mass & w,

In the limit

when &

m and

$ v

>

0,

(= -p?)

becomes the

actual density at the point P.

Example

1.

The

in a cone having its greater than the density at the

density at any point of a liquid contained axis vertical and vertex downwards is
surface

by a quantity varying

Show that the density of the liquid, as the depth of the point. mixed up so as to be uniform, will be that of the liquid when
originally at the depth of |th of the axis of the cone.

Let h be the height of the cone.


Consider a circular disc of liquid at distance

x from

vertex

is

and of thickness dx. Since the depth of every point on the disc a?) below the base, density at every point of the elemen(A
the

tary disc is at the top.

same

viz.

P Q 4-(A

x) X, P

being the density

Mass of the

disc
{

is,

therefore,

+(*-)
{

*fc* tan

dx.
tan 8

/.Total mass

= f* *A Total volume =

+(h-x) I.}***
2

a dx

tan

/.Mean density=

+ X h --j

density of the liquid originally at the of one-fourth of axis of the cone.

depth

Example
of

fluids is

Show that the specific greater when equal volumes


2.

gravity of a mixture are taken than when

equal weights are taken, assuming that occurs as the result of mixing.

no ch^pge in volume

CHAPTER
*

II

Equilibrium of Fluids.
2. 1.

The Fundamental Equation of Equilibrium


fluid,

Consider a mass of

compressible

homogeneous or heterogeneous, at rest Let the component of the given force at systeir HT given forces. P (#, y, z) along the three rectangular axes be respecany point tively X, Y, Z per unit mass of the fluid.
Let P be the density of the fluid at any point P (x, y, shall suppose that P, X, Y, Z are all functions of (x, y,
require to find p, the
(x, y,
).

or incompressible, under the action of any

z).

z)

We We

pressure at

any point

as a function of

Let

p=f

(x, y, z).

Pake a point
ixis,

Q x+ <$X
<

y, z) adjacent to

P and with P Q

as

construct a cylinder of the fluid with an infinitesimal

cross

section a.

This cylinder
( i
)

is

kept in equilibrium by the following forces:

the thrust of the fluid acting along

P Q on

the cross

section at P.
(ii)

This thrust

is

pa=af
p'a=o/

(x, y,

z)',

the thrust of the fluid

acting along
(x-}-$x, y,

QP
z)\

on the cross

section at Q.
iii)

This thrust

is

acting on the mass of the fluid; the of this force parallel to or PQ is plainly XaPtfz, jomponent lP$x, being the mass of the fluid in the cylinder;
force

the external

OX

(ivl

The

fluid pressures acting

on the curved surface of the

cylinder.

They have no component along PQ, being everywhere


resol ring

perpendicular to PQ.

Hence

along PQ, we have for equilibrium

11

or

X
is

Proceeding to the limit when Sx

made

indefinitely

small,

? /

dx
Similarly,
>

= p X== P

'

jT

=PZ.
3 2 In other words,

|f=PX,|^=PY,-i-p = PZ ...... 3aj '82


'8#
Again
since,

(1)

dp=df=(^ dx + &dy + o x G
y

|/<* a) z
o

=P(X
/.

^a;

+ Yrfy-fZrf^);
(2)

dtp=P(X^ + Tdy+Zrf). .......


is

This

the differential equation which determines the pressure.


:

Remark Let PQ be the axis of a very small cylinder bounded by planes perpendicular to PQ, PQ being drawn in any
direction.

Let^? be the pressure at

P and^+<5jp

at

Q,

a the

area of cross* section and Let 8 Sm be s the length of PQ. the component of the forces acting on the infinitesimal fluid mass at P, in the direction of PQ.

Then as
of the fluid

before, considering the

equilibrium of this cylinder

we have

or, rfjps=PS

ds ......... (3)

12

Now if (*,
of

y, z)

be the coordinates of P, then the direction cosines


if

PQ are ^' i&4*\ Hence as cts as


PQ
is

*m,

*m, Z

*m

are the

in the components the force acting on an elementary mass direction of the coordinate axes, then the component of these
forces along

So that

Hence from (*)

=P
If the position of coordinates r, 0, and

(X rf*+Y
is

dy+Z

dz).

P
2,

given in terms of the cylinderical and P, T, Z be the conponents of S in

in the direction of

r, 0, z,

for the cosines of the angles

made by P

with

P,

PM,

(per-

pendicular to
respectively.

in the plane

P),

and

are

^,

-|?,

~ g

Hence,

dp=P

(Prfr+Tr

d6+Z

dz) ......

(4).

Again
nates

if

r,

the position of P be given by the ordinary polar coordiand if the components of 8 in the direction of r, 0, 0,
<t>

and

be R, N,

T then

Hence,
2*2.

dp=P (R dr+N
fluid

r Sin 9

d<f>

+T r d6) ..... (5)

For equilibrium of the

we must have

18

But p being a function of the independent variables x, y, z


'

fafty

dzdx
(1)

tfp _ Qxdz' Qxfty

Hence from

we have

oz

ox

(2)

80;

'8 y
obtain

From

these

we

z*?-^
8 z

Qz
uX

Qy
oZ

uX

2.-x
Remark L In of density is known

= P (-

the case of heterogeneous fluids, whose law (i. e when P is a known function of x, y, z,)

conditions of equilibrium are given

by equations

2, or (3),

above.

Remark
tive

II.

Elastic fluids under

the action of a conserva-

system of force.
Let

be the potential function of the system so that

X cfo+Y dy+Z dz~dV.


In the case of elastic fluids

we have

hence, the equation

dp=P X dx+Y dy+Z dZ) becomes

14

>

dp
Integrating

F"~
V/k

__ dV IT
V/k
and P =r0.

p=c

Multiplying equations (3) by X, Y, we obtain

respectively

and adding,

Y
-\r

/3Z ~ 3A.\

r*

/3X.

- 3 Y\

which

is

therefore a necessary condition of equilibrium.


interpretation of this

The geometric

equation

is

obvious

t?t>.

that for equilibrium to be possible the lines of force,

^= ^= -*

can be intersected orthogonally by a system of surfaces.

homogeneous and incompressible t e. if P is constant the necessary and sufficient condition for the equilibrium of the fluid is that (X Ja;+Y rfy+Z dz) be a perfect

Remark

I.

If the fluid be

other words, the force system be conservative, or capable of representation ,by a potential function. That is, there exists a function (x, y, z)
differential,

or in

V
,

such that

.,

Y Y X- 3V v_
are

3V -

,Z

~_

3V.

then clearly by taking jp= VP, the equations since P is constant. satisfied Conversely if 2.1 hold t. e. if the fluid is in equilibrium, then of equations (1) there exists a function jp (x, y, z)

For

if this is so,

(l) of

2.1

so that the force system


tion
is
~.

is

conservative and

its

potential func-

15

Superposed Fluids : Let two fluids be superposed so that p, P and (X, Y, Z) are the pressure, density, and force components per unit mass of one fluid and p', P', O^,' Y', Z') are similar
2. 3*

quantities for the second fluid,

The two

fluids

being in equilibrium,

we have

for

the 1st

and

for the second,

dp

P'

(X'

dx+Y dy+Z'

dz)

) is zero, Along, the surface of separation S, the difference for any element of this surface must be subject to the same

(pp

have therepressure on both sides for equilibrium to exist. the result that for any displacement ds along the surface S, fore,
P

We

(X

dx+Y dy+Z
WheiTthe

dz

= P'

(X'

dx+T

dy+Z'ldz),

which

is

the differential equation of the surface of separation.


I.

Remark

force systems (X, Y,

7 Z) and (X

Y', Z')

are conservative or derivable from potential functions

aiid V',

we have

Remark

II.

When the force (PPOrfV = 0,


P'^0,

system

is

the same in two cases

Since P

dV =

along the surface of separation.


is

The

surface of separation
III.

therefore an equipotential surface.


to

Remark

If P, P' be supposed

be constants the equa-

tion of the surface of separation is given

by

PV-PT'^cons.
In the case of an incompressible a conservative system of forces homogeneous liquid subject derivable from a potential function V, we have
2. 4.

Pascal's Principle

to

giving the pressure at any point

when

that at any other point

is

known.

Suppose now that by any process whatever the pressure at a new condition of equilibrium is is altered to p -\-& p
]

established

and the value of the pressure

at

any point P

will

become p-\-$p-

The new equation of equilibrium

is

_v

It follows

then that in a liquid in equilibrium any pressure applied


of the
liquid is

at

any part

transmitted equally to all the parts

of the liquid.
as Pascal's Principle for a homogeneous incomin Bramah's Press. pressible liquid and is applied

This

is

known

In the case of a

fluid

in

which density P

is

a function of

is pressure, the equation of equilibrium

where P
If

=/ (jp)

and.p

is

the pressure at the given point

we now change

the pressure at

P
is

to jp

+fp p

the

new

value of the pressure at

P isjH-fp, which
\

given by

17

p
rP+*l>
or
dr)

rl

P
This equation gives Sp in terras of
ralised Principle of Pascal.

and expresses the gene-

In the particular
so
is

case,

where Sp Q

is

infinitesimally small,

Sp.

rP+*P
r

dp

Sp

J
o

Hence

^=^
variations of pressure
in
are,

The

infiaitesimally small
at

therefore,

transmitted

every point

proportion

to the

density at

the point.
Surfaces of Equal pressure. In all cases in which equilibrium is possible, we obtain by integration
2. 5.

This shows that locus of


is

all

points at which pressure

is

constant

the surface

as a -surface of equal pressure. It is evident that by varying the value of the constant we obtain a whole series of surfaces of equal pressure.
is

This

known

If this constant be taken as


to the fluid

we

equal to the pressure external obtain the equation of the free surface or the

external surface.

18

If the external pressure

is

zero, the free

surface

is

given by

We shall now

prove an important theorem regarding the sur-

faces of equal pressure

viz. "The surfaces of equal pressure are the surfaces intersecting orthogonally the lines of force ".

For the direction cosines of the normal to a surface of equal


pressure at a point

(#, y,

2) ar

by

(1)

proportional to

But these quantities are respectively equal to PX, PY, PZ.

Hence the

direction cosines

of the normal

at

to the surface of

equal pressure pa sing through it are proportional to X, Y, Z. The direction of this normal is, therefore, also the direction of
the line of force
at P.

The

line

of force

at

intersects the surface of equal pressure passing

P consequently through P ortho-

gonally.

Remark
the action

I.

In the case of a homogeneous liquid at rest under

of a given system of forces, the surfaces of equal For, we have already seen pressure are equipotential surfaces. that when P is const -nt the system offeree must be conservative,
t
.

e.

must have a potential function V.


fc=P

(X rfoH-Y

dy+Z

dz)

=
or ,

PdV
V, and vice

Hence
versa.

if

is

constant over

any surface so

is

Remark

II.

If

fluid is at rest

under the action of a conser-

vative force system, surfaces of equal pressure are equipotential, and are also surfaces of equal density.

19

For

if the

force-system

is

conservative, then

X dx+ Y dy+Z dz =
Hence
for equilibrium

dV

dP=P dV.
dp
for if

Also since the

fluid is in equilibrium,

Hence P must be a function of V,

a perfect differ entiaL not P d V, and therefore


is

dp

will not be a perfect differ ential.

Hence both P aud P must be functions of V. Consequently if V is constant over any surface so must p and P be over it and vice
versa.

Surfaces of equal pressure are

therefore

also

surfaces of

equal potential equal and density.


2. 6.

Curves of Equal pressure and density in the case of


fluids.

heterogeneous incompressible

of equal pressure and density the conservative character of the 2.5 depends on proved in If the force system be not conservative, the two force system. ave a surface of surfaces will not be coincident. Suppose we
identity of the surfaces

The

equal pressure given by

p=f
Let

(as,

#, z),
z>

where p

is

taken as constant.

..

(1>

P=0

(x y>
y

be a surface of equal density,


(2)

where P

is

assigned a constant value

We

require the intersection of these two surfaces which will be the curve of equal pressure and density t e. the locus of all points where both the density and pressure of the fluid are the
.

same.
If

we

consider the

totality of surfaces of equal

the totality of the surfaces of equal density, these surfaces define for us a family, of curves whose

pressure and two series of


differential

equation

we

proceed to discover.

y^rdy^ z-\-dz) be any two neighbouring points on a curve of this family, which for the sake
(#, y, z)

Let

and

(x-^-dx,

20
to

of definiteness

we may assume
are points

be given by the intersection

of

a) and

(2).

Since

P and Q

on the same surface of equal-pressure


)

/ (x,y,z )f(x+dz, y+dy, z+dz

Neglecting infinitesimals of higher order than the

first,

or,

or P

(X

<M-Y dy+Z
similarly

dz)

........ (3)

Again, since
density,

P and Q

are points on

the same surface of equal

we have

+-=o ..... (4,


dx dy, dz
t

therefore satisfy equations (3)

and

(4^.

Hence,

dx

--

dy

dz

(f

But from equations birium of the fluid


r/
i

(3) of

2'2

we have

as conditions of equili-

--v-D' --Y r- = r(
-

8y

3*
1

to

),

etc.

3y

Hence equations
dx -=g

5)

become
...... (6)

_ =:_J?1_ 3X_8Z
eJj/.

82

3*

21

These are the

differential

equations of the curves of equal

pressure and density,

27.

Fluids at rest under the action of gravity


vertical

Take

measuring down wards as the axis of Z, then

X=Y=o, Z=g.
Hence equation
(2) of
2'i

becomes

dp
If P
is

9 g d z
i. e.

constant

if

the fluid

is

a homogeneous liquid,

Thus

surfaces of equal pressure are

horizontal planes.

The

free

surface

must therefore be

horizontal.

Let us take the origin

in the free surface

and

TT

as the external pressure,


so that c=TT.

then

p=

TT,

when 3=0,

Hence
If there is

p~g

3-+-

TT.

no external pressure on the

free surface

P=9

p *

(ii).

In the case of a heterogeneous liquid

we have

dp>=9
so that P

p d*>

must be a function of zmlone.

and pressure are constant for all points in the same horizontal In other words surfaces of equal plane. pressure and also of equal density are horizontal planes.
It follows that density
Uiii.

In the case of an

elastic fluid,

p=&

22

gz/k
or,

P=c

Thus the
planes.

surfaces of equal

pressure

are again

horizontal

Ex.

1.

mass of

fluid

rests

upon a plane

subject to

central attractive force-

situated at a distance c from the plane

on the

side opposite to that

on which the
of the

fluid is

if

is

the

radius of the free spherical surface

fluid

show

that

the

pressure at

any point
i4

is

given by

P (i-J). T a

Let
force.

be the centre of

perpendicular

from

on the given plane.


(5) of

Using equation we have

2*1

Thus if p~ const., r== const, so that locus of all points at which pressure of the liquid is the same is a sphere with
centre
is

External pressure
a,

is zero, therefore,

the free surface

a sphere of radius

given by

23

This gives pressure at any point of the


Ex.
2.

fluid.

mass

M of gas at uniform

temperature

is diffused

space and at each point (x, y, z), the components A #, The pressure of the force per unit mass are Cz. By, at the origin are p and P respectively. and density

through

all

Prove that

We

have by equation

(1) of

21
aj+B y d

dp =
and

P(&xd
Ax

y+C

d z\

=AP

" f*P.=
p

d"fl+B y d
k

y+G z d

or

p=

Since p

p and P = P when
^o_-r, n /c. v, Po =1)

x= yz^

J-

Accordingly,

p=p
(a?,

Also density at any point

#,

}is

given by

Now, the

total

mass of the gas

diffused throughout

whole space

is

GO

r J I

r
I

m Pdxdy
dz.

CO

-^00

24

00
;.

00

00

2p

f J

00

/ /

00

*'

po
CO

dxdydz
3
2

=p
1

~"~5
J
oo

a!a

x
x

rf

~^f^ x
dy

oo

V^;" d%
oo

ABCP

Ex

3.

compressible

liquid

is

at

rest

Defining the compressibility,

k by
y

the relation

p_ p

under gravity.

~p o
'

~ k (pp

P are the density and pressure, respectively, and P and p refer to the free surface, and assuming k to be constant, show that at a depth z below the free surface where
P,

and hence that

P=P

mine shaft has the same horizontal cross


h.

section at all

depths
density.
surface,
It

It is filled with water to a depth of the water were

Show

that if the

everywhere

equal to

that

at

the
.

the water would rise in the shaft a distance

-&I,

may
0. S.

be assumed that k

is

very small

(Tripos Part

1925.

Higher Applied 1937).


Ex,
4.

An

infinite
is

mass of homogeneous

fluid

surrounds

a closed surface and

attracted to a

point (0) within the surface

25

with a force which varies inversely as the cube of the distance. If the pressure on any element of the surface about a point P
be resolved along PO, prove that the whole radial pressure thus estimated, is constant, whatever the shape and size of the

being given that the pressure of the at an infinite distance from the point 0.
surface, it

fluid

vanishes

Take
nates.

as the origin Equation (5) of

and use 3-dimensional polar

coordi-

21 gives

*- - o $
Hence, p

uP
-

But p=o, when r

>

cx>,

"
Consider

=L 2r*
and
let P,

now any

surface

where

P=r, be any

point

on

Taking an element d 8 of it we find that the radial P is p cos e, e being the angle between the pressure along normal at P to the surface and the radius vector P.
it.

Hence the whole

radial pressure
cos e rfS

rfS

'dB
where
(/,

....

(1)

m>

ri)

are the direction cosines of the normal at


(x, y,
1
)

to

the surface

and

the Cartesian Coordinates of P.

Now by

Greens theorem we have

SS (lf+m

26

Hence the

surface integral (1) becomes

pressure has no physical significance whatsoever, being merely a sum of the magnitudes of a number of forces acting in different directions.
:

Note

The whole

radial

Ex,

5.

hollow sphere of radius


is

a,

just

full

of

homo-

geneous liquid of unit density


II*

placed
li'^

between two external


such

centres of attractive forces


tp*

and

7,

distance c apart, in

a position that the attraction due to them at the centre are equal and opposite. Prove that the pressure at any point is

Let A,

be the centres of force and 0, on


is

A B

be the centre

of the sphere then

given by

"AO OB
or

Adopting the bipolar coordinates fr, r') of any point of the P referred to A, B as origins we have

fluid

27

ul
(
j-

dr

u + ~-~

*
<ir'

\
|

density being

1.

^7""'

To determine

the constant C, we observe that since the sphere is only just full, there must be a point on the sphere at which the pressure p is zero.
If the bipolar coordinates of this point,

be (r^r^\ then

o+^+jl'-o
Also
if

a)
OB, then

OQ

makes an angle
2

6 with

'=OB +a a 2a.OB cos 6 r^ = OA*-fa*+2aOAcos


ri

W
,
,

Evidently
so that

is

the point on the sphere at which

is

minimum,

dp=o.
a
/

Hence

^--|rfr 1

+^

^^\ ^o

(3)

Hence from

(3)

we have

0*1

.. \Qt )
(2)

Again from

QB-i-o~2aOBco9 8 = r i^ OA 1 -r-o f +2aOAcose r*

28

or substituting the values of

OA

and

OB we get
(5)

a(n

n' )

=2

c|V

cos 6

>i

i'>

U \OB +a*-2aOBcos0/,
1

Substituting for

OB

and

for cos

from

(5)

"

(>i

ti/)a

Hence p Hence, o
Ex.
6.

+^
^
is

{c
to the
y, z)

If the components parallel an element of fluid at (#, acting on

axes of the forces


be proportional to

show that equilibrium

only possible,

if

,2X=2n = 2i;=l.
Find the most general possible expression
the
fluid.

for

the density of

If the density

/^-Tv2

from

the

plane

oj+^+z^o,

the

curves of equal pressure and density will be

circles.

Equations (3) of

2 '2 give for equilibrium

or,

y *+2\yz

+z

) {

(a-^+di-i,) x

29

or,

or,

yz

or,

a y z

(l-X-|)-3 a y (l-X-t = o
1 1

Hence
and and

X
\k

^=o^l
v=o,
1 1

X
p v

u=o,

X=o,
ji

X = o,

= o,

X=
Thus the necessary

= =4
i;

condition for equilibrium

is

2X=2|ts=2<;=l.

We

have, therefore, from Equation 2 of

2.1

dp=P

(X

dx+? dy+Zdz)

where X=fc

*-f-*4-

etc.

In order that equilibrium be possible


function of x, y,
a,

we must have P

such a

that

PS! (y' -\-yz-\-z*)

dx

IB

a perfect differential,

say of

(x,y,z).
proceed to find the primitive of the

To

find the form of 0,

we

total differential equation

80

That

this primitive does exist is

known

for the condition of

integrability, (

which

is

the

same

as the

condition of equilibrium

expressed by Equations 3 of

2*2), is satisfied.

Treating z as constant in the

1st instance,

_f*!- = xyxzzy
*

(y*+# z +s
2

f
)

dx+(z*+zx+x*) dy^o
u

leads to

"8*

(2*

-2\xy-jcz-zyY
2

'

8y
2
'

(g

-2 xy-xz-ey,*

'

(^

as

* y)

Hence
[1)

if

we

multiply (I)

by

z2

2 x

becomes
*

yx zz y

2
}

'

d dx

+ 1? a v 4- A(* ^ d +
^

or

' (

8o!

8#

7
(z

yx z-2 x y

zz y)*

or,

(zr~dx-\- J*

or,

-. =con5.

'

Thus settmg * fe

y,

)=?

31

have

p ^T

VT-

etc-

Hence P

(y -f-y

z+2 a dx becomes
)

or,

P (as+y+z>* d

<t>

(a-f-y-fs)

rf0,

where

2+2 g
It follows, therefore, 3 that P (as-j-y+z)

muet be some function

e.

fa+y+i)

Hence

P=

(flj

-L_/
+ y + )V
=ss

=/(
g
v (

^^ *+* g H-H-s

, ),

giving the most general possible expression for the density of the fluid in equilibrium under the action of the given forces.

Curves of equal density and pressure are given by the differential

equations,

dx

dy

dz

ax _ az
which become in the present case dx __ dy dz

3Y_ ax

..... ..

(i)

......

(ii).

82

Eq.

i)

leads to o;+y+2=cons; and

Eq.

(ii)

leads to

o:

+ y 2 +*2=:cons.
circles.-

Thus the curves of equal pressure and density are

under the action of a system ot forces whose components along three rectangular axes are

Ex.

7.

If a fluid is at rest

proportional to

(ay -I z\
show
that in

(c

za x\ xc y)
(b

order that equilibrium be

possible

density must

be equal to

a y

o z

F
Show

being any arbitrary function.


of equal

that the lines

density and

equal

pressure

are

straight lines.

Ex.

8.

Find the value of pressure

at

any point

in

the

preceeding example.
l

f aj
is

a
in

yb z
=^

yb

equilibrium under a given system of = x, y r *\ P a (*, y, *) be two possible values forces; if P a of the density at any point, show that the equations of the surfaces of equal pressure in either case are given by

Ex.

9.

fluid

(x, y, *)

+X

(x, y,

X being an
Ex.10.

arbitrary const.
afr

mass of gas

constant temperature

is

at

rest

under the action of forces of potential


with any boundary conditions.

any point of space, At the point where ^ is zero


at

33

and the density P . The gas is now removed the pressure is from the action of the forces and confined in a space so that Prove (hat the loss of intrinsic it is at a uniform density P
.

potential

energy by

the gas, due to expansion,

is

CCC

JJ>
where the integrations are taken throughout the gas in
original state.
.

its

11.

aud radius a is density a and external radius


under
forces.
its

uniform spherical mass of liquid of density P surrounded by another incompressible liquid of


b.

The whole

is

in

own gravitation, but with no external Show that the pressure at the centre is
2

equilibrium pressure or

?_a

_ a) +

that a liquid of density 9 occupies the entire space enclosed in the outer sphere of radius b and a liquid ot density P a occupies the space enclosed in the inner sphere of radius a.

For our purpose we

may assume

Hence

gravitational attraction

at

any point of the

fluid

inside the inner sphere is

r being the distance of the point from the centre

Also the gravitational attraction at any point inside the outer sphere and outside the inner sphere is

34

Hence pressure
given by

at

any point inside the inner sphere

is

dp ~
and
at

_p^p r

dr ........ .(i)
inside

any point outside the inner sphere but

the outer

sphere
...

(ii)

P a r, ac being the presure at the centre. This gives the presure at any point inside the inner sphere.
/.

p=c

Tl

Hence presure on the boundary of the inner sphere


c
a

is

2^P
at
is

a*

...

...

...

(iii)

Again pressure

any point outside the inner sphere but inside


given by

the outer sphere

But p

=ss

o,

when
i

=
*

4"

'^"T

/n IT ~<P-^
\

**>*

fl8

-j

ex

_ a a *(_-)
)
i
.

r as

Hence pressure a, is given by

at the

boundary Of the the two liquids

0.

at

-r

2*.

by

(iii)

35

/. c

= ~ P* a* + ~ a
Two homogeneous
and

a (J+a

+(P-cr)

(6-a)

12.

volumes

liquids of density P and P lf and x are superposed in a cylinderical vase having


,

the form of a cylinder of revolution with

vertical generators.

The upper
and

liquid is simply heavy, but the lower liquid P x is heavy in addition its elements are attracted by the centre of the

bottom of the vase, the force of attraction being proportional to The attraction of the point distance and mass. on a unit mass at unit distance is X. Determine the figure of equilibrium and
the

law of pressure.
Ex. 13.

uniform incompressible fluid is of mass in graand forms a sphere of radius a when undisturbed vitational unite, under the influence of its own gravitation. It is placed in a weak
field

of force of gravitational potential


r is

T*
-

|A n

a T

8n

(0,

0)

(n>l) where

measured from the centre of the mean spherical surface of the liquid and the squares of quantities of the type p, can be neglected. Prove that the Equation of the free surface is
--

^^ a- 1 4. S M2n1

2n

+lQ S
2

(fl '

>

A\
*>'

S m being a Spherical Harmonic of nth


Let P be the density of the
liquid,

order.

then

Let the Equation of the free surface be

^=1

a.

MM),
(2).

.-

...

(2)

We first proceed to find the potential


matter enclosed within equation

of a gravitating mass of Since a D may be assumed

36

to be small,

the potential is the same as that of a solid sphere of radius a plus the potential of a distria n S n (0, 0) spread over bution of matter of surface density aP
consider
that

we may

the

It is sphere. solid potential of a

known from
sphere of
p2

theory of Potential that homogeneous densily P at an


the

internal point

is

2^? a 2 (l

^-5).

Likewise the potential of a

distribution of matter of surface density

over a sphere of radius a


point.

is

4 71 Pa

"

u S

^TT

r (

n
)

at

an internal

The

total

potential

of the external
therefore,

field

and that of the

the liquid gravitation of

is,

3a 2

'

2/H-l
that

Now we know
we have

in the case of liquids subject to a con-

servative system of forces derivable from a potential

function V,

If
is,

o over the free surface,

the Equation of the free

surface

therefore,

V=C

or,

In order that this be identical with the equation


that
it is

=l-|-5;a n S

necessary and sufficient,

2n+l

=O
a

for

n>

<

37

Hence the Equation of the


?

free surface is

a
Ex. 14,

tBolid

P gravitating sphere of radius a and density


of volume

is

surrounded by a gravitating liquid


density
surface
a.
f

( A

a8

and

ial is i co

The whole is placed in a field of force whose [1- P a (cos 8)]. Shew that the form of
is

potentthe free

of the liquid

a spheroid of Binall ellipticity 6 given by

t=ft

eP2

(cos0)},

where

8=
coefficient of the second order.

P 2 being Legendre's
N/
2.8.

Rotating Fluids. If a particle of mass m moves in a circle of radius r with constant angular velocity o>, the effective force
is

acting on the particle L e. the product of mass and its acceleration, a 01 r, acting along the radius and directed towards the centre.

Consider

quantity of fluid revolving uniformly and without any displacement of its parlicles L e. as if rigid, about a fixed axis. The "effective force " on any particles? of the
relative
fluid is

now a

m+*r

directed towards the axis, where r


<**

is

the distance

of the particle from the

is the angular axis and velocity of the fluid. This force must be supplied by the external forces and fluid pressure acting on the Hence the resultant of the particle.

external forces and fluid pressure on the particle is a force m<*>*r acting towards the axis. It follows then that the fluid pressure,

external

forces,

and

forces

m<**r acting from the axis are in

equilibrium.

We may

accordingly consider the fluid at rest and

38

apply the eqnations of


particle as

2. 1

provided that
a
r)

we

consider

each

acted on by a force (w axis in addition to other forces.

per unit mass from the

Take the

axis of rotation as z axis.

nal forces acting on the fluid is function V, the equation of equilibrium, therefore, becomes

system of exterderivable from a potential

If the

=p <_ ox
{

>*r+ (-<r

oy

Or
assuming P
This
fluid
to be constant.

shows that

fluid

pressure

is

the same as in the case of a

at rest subject to

an external

force derivable from a potent-

ial function.

The
term,

effect of rotation

is,

therefore,

merely

to

add an additional

iu

2
(

x*-\-y*

to the potential of the external system.

Ex.

1.

A homogeneous mass

revolves with uniform angular

of liquid contained in a vessel velocity w about a vertical axis

Find the pressure at any point and the surfaces of equal pressure.

Take the axis of rotation directed vertically upwards as *, axis and the highest point of the axis as origin. Potential V of the
external force system
is

clearly g*.

Hence

39

or,

p=P

W^ +^)-9*
2

+c

Surfaces of equal pressure are parabloids of revolution.

^ Ex.
10,

rotating as if solid with angular velocity about a vertical axis. Shew that the free surface is formed
2.

Liquid

is

by the revolution of a parabola of J

latus rectum

-^ about w*

its axis.

(Tripos part 1 1924)

Ex.

3.

of radius a
vertical,

A mass m of an elastic fluid enclosed in a cylinder and height k is rotating round its axis, which is
Prove that density
r

with uniform angular velocity w.

at

auy point distant


c/k
e

from the axis and


2gz

at a height z is

wV
e
2fc

Where

c is

given by the equation

c/k

2k

gw
Ex.
4.

Prove that the common surface of two homogeneous liquids of densities P and P lf superposed upon one another in a vessel and rotating with uniform angular velicity w about a vertical axis is a Find the latus rectum paraboloid of revolution.
of the generating parabola.

(Hint use

2. 3.

Ans.

% w
2

5. A mass of elastic fluid is rotating about an axis with uniform angular velocity w> and is acted on by an attraction towards a point in that axis equal to n times the distance, n

Ex.

being

>w f

prove that the equation of a surface of equal density is.

40

*I

alinsr
r

ffo3

axis of rotation
it

as

axis

and the centre of


the equation

attraction
ot

on

as the origin,
PeJV,

we have from

prensnn

<//>==

wh*re v

1ho potential of the external foroe* acting.

or,

k log

P=c V

-V/k
>

ow

the potential of the

attractive force is \\k(

to

+>'
2

and

that

due

to the "centrifugal force" of rotation

Jt0

(oj

-f /*

Mi nee the

whole mass

is

m,

Too
J
00

fcv>

A
J
00

dx dy dz
Too

J
A

CO

W2)
2k

= 8A

a?

-~~w~ y

(]Jr-V)

41

The Equation
2

of a surface of equal density P


2

is,

therefore,

(ji-wj

(*>+/)+n*

A;

log

=
Ex.
contains
6.

fc

log
2ct

conical vessel of height h and vertical angle water whose volume is \ of the cone ; if the vessel
w>,

and

the water revolve with uniform velocity

and no water overcot a.

flows prove that

w must

not be

> V

-\ /

3A

tube in a vertiLiquid is contained in a thin circular If the cal plane which can rotate about the vertical diameter. show that it will sepaliquid subtend an angle 2<z at the centre,

Ex.

7.

rate into

Sec two parts when the angular velocity exceeds \j^


circle.

^,

a being the radius of the


(Hint:

the liquid will


*

first

separate

when

the pressure at

the lowest point" is zero.)

Ex.

8.

The

whole

circumference of a circle

and a

diameter are formed by fine uniform tubes communicating freely with one another. The diameter is vertical and initially tubes
are
filled

with water to the level of the centre of the


all

circle.

Prove that
if

water will be just driven into the circumference the system be made to rotate with an angular velocity

42

.,

about the diameter, where a


i.

is

the radius and

a(l
is

sin 0/2)
e.

the unit circular measure

a radian.

Let

Let 0, be the centre of the circle and its lowest point. be the points on the circle up to which water rises when the water in the diameteral tube is just driven into the

P and Q

circumference.

Then

clearly arc

AP=arc BQ=?.

Therefore

<AOP=<BOQ=^0beinga
Taking we have
as origin

radian.

and the downward

vertical as z

axis

c being a constant.

Since the pressure

&t

P must be equal to

that at

M, we have

cos'^-f-e
a

or,

wa eos a

=
2

a
(1 v

sinr 2

or,

Va
1

sin ~

&

9.

circular
P.

cylinder with an open top is filled with

a liquid of density

The

cylinder

is

set rotating

along with

48

the liquid with angular velocity w about its axis which is vertical. 4 Show that a volume (*ta a a P)/4# will run out, a being the radius
of the cylinder provided that

w* <
a

Examine the case when

w2 >

Ex. 10.

depth

k.

cylinder of radius a is filled with water to a fine vertical tube is at a distance d ( Z a) from the axis

of the cylinder and communicates with the bottom of the cylinder. If the whole is made to rotate uniformly about the axis of the cylinder show that the angular velocity w necessary to make
the water rise in the tube to a height

x above the bottom

of the

cylinder

is

given by

aa

Let x be the height of water in the vertical tube at a distance d from the

MN

the cylinder. Let BAG be the free surface of the water in the
axis

OA of

cylinder

when

the system

is

revolving
as

with angular velocity w.


origin
e,

Take

and the upward

vertical as axis of
is

the equation of pressure

H
w*rdr

gdz

-f"

w'r*

N the top point of the vertical tube up to which water rise*, *=, and OM=d, p=Tl the pressure at the free surface.
At
TT

(D

44

If the height of

B orC

above

is k,

then

=C
.

$r&

w *a a
z\A. i

...

...

(2)

P n i

a._

w *(r 9 - a 2 )
surface viz

The equation of the paraboloid of free


clearly

BAG

is

therefore

g (k
or, r
2

) -f-

wa

(r

- 2 )=o

= -f

-k +

^-

Now

the

Volume of the

liquid

= Volume of the cylinder GHBC


IT

paraboloid

ABC

{
But the volume of -water was originally
""a
2

or

A/ s=;

A+

..

(3)

Now subtracting (I)

from (2) we have

g(x~-k)

+ l!W*

(a

d*)

45

Hence by (3 we have
>

Ex. 11.
at the lower
is
I

A
end

straight tuba

AB

of thin uniform bore is closed

and

it

with water. The length of the tube rotates with constant angular velocity w about the

and

filled

vertical

Show

through A, to which the tube that no liquid excapes if


ff

is

inclined at an angle a.

cos

and

that all the liquid escapes

if

w?

2g
i

cos
2 3

sin

a a

Ex. 12.
tightly filled

uniform semi circular closed tube of radius r is with equal volumes of two fluids of densities P and ff

respectively
velocity
line of

which do not mix, and

is

rotated with

w
if

about a vertical radius

symmetry.

making an angle tf Prove that the pressure at the two ends will

angular with the

be equal

'
2J7

'

=
cos

+ sin a

~
cos

sin

the fluid of density a being the lower of the two, and vexity of the tube being downwards.

the con*

Ex. 13.

fine straight tube of length

Z,

closed at both ends,

and inclined
of

to the vertical at

density P. If the tube is a vertical axis through the lower end, prove that the pressure at 'the highest point is

an angle a is just filled with a liquid rotated with unifrom angular velocity

w round

Pw*

Bin

(I

cos <*lw* sin**)*.

46

provided that

w*> g cosoi/l

sin 3 a.

What

is

the

pressure if

w*

<

this value.

M/

OAU

%%

Ex
is

14.

made

hollow sphere of radius a, half filled with liquid, to rotate with angular velocity w about its vertical
If the

diameter.

lowest point of the sphere is just exposed,

prove that

2g~aw* (2
Ex. 15.

<4~)

Prove
is

that, if

a mass of homogeneous liquid rotates

acted upon by a force to a point in the axis, the curvatures of varying inversely as the square of the distance, the meridian curve of the free surface at the equator and pole

about an axis and

where an d b are the \and r(l o s~) my and m is the ratio of the centrifugal equatorial and polar radii,
are respectively J r a 71 \l
force at the equator to the attraction there.

CHAPTER

III

Fluid Pressure on Surfaces


In the previous chapter we saw how to calculate the pressure at any point of a mass of fluid at rest. We shall now consider the problem of finding th* resultant of the pressures
8.1.

exerted by fluids
contact.

at

rest

upon surfaces with which they are in


horizontal plane surface

AB

at a depth

Take first the simple case of a d below the surface

of a homogeneous fluid at rest under At any point P of this gravity.


surface we
is

that the fluid pressure P dj P being the density of the

saw

"

fluid.

The
to

direction of this force is


i.

normal

the surface

e.

vertical.

Hence

if

we

consider an

area $ A of the surface round P, the force on this infinitesimal area would be g P d 8 A, acting at right angles to
infinitesimal

the plane,
Splitting
fluid

up the surface

into

an

infinite

number of such

infinite-

simal areas

we

find that
is

pressure

just the same a

parallel forces like

g Pd

the problem of finding the resultant that of finding the resultant of A acting all over the plane. Now
is

the resultant of|these parallel forces

known

to be

g P d
since

is

the same for every point of the horizontal plane surface

we

are considering,

~
where

9 P d A,

is

the whole area of the plane

48
the

3.2.

Let us

now

take

up

more general problem of a


under the action of

plane surface

of

any kind

in a fluid at rest

any given

forces.

Take any two rectangular lines in the Consider any point plane as axes of x and y. The pressure (i. e, force P (%, y) of the plane.
exerted by the fluid per unit area) at P will dpend on the forces which act upon the fluid

be p r which will in of (x, y) and the forces acting on the general be a function fluid. Moreover, its direction will be normal to the plane. Consi-

and maintain

it

iu

equilibrium.

Let

it

area dx dy round P we see that dering as before an elementary the force acting on this area will be p dx dy at r ght angles If we imagine the plane to be split up into an to the
plane.
indefinite
will

number

correspond

of such plane areas, to each such area there a force of definite magnitude and normal to the

have, therefore, to compound an indefinite of such parallel forces; the resultant is, therefore,
plane.

We

number

p dx dy

ff p dx dy.
polar coordinates, we would have considered an area rdBdr round P and the expression for the resultant
If

we had used

pressure

would have been

ff p rdBdr.
Remark*
If a

gravity alone, below the horizontal surface.

homogeneous p= g P z where

fluid is

at rest

under the action of

is

Hence

the depth of the point the resultant pressure

^S

9 P

^A,

$A

being an infinitesimal area round P.

=aP

49

=g P
2f

z A, where

= whole area of the

plane,

and

depth below the horizontal of the centroid of the

plane.

Ex.

1.

Calculate

the pressure

on a

triangular

lamina

immersed

in a fluid of density P.

Let the depth of the 3 vertices of the lamina below the horizontal surface be rf ll d^ d$.

Then the

resultant

fluid pressure

=g

^
3

~*~

~*~

where

A = area of the
2.

triangle,

for,

z =*

*
'

Prove that the pressure of a parabolic lamina bounded by latus rectum and inclined at an angle 6 to the horizontal is | ^P (d + f a sin 6) a 2 where d is the depth of
Ex.
,

of the vertex and 4 a the latus of the lamina,

Ex.
fluid

3.

An

isosceles triangular

lamina

is

immersed in a
lamina
in the

vertically
fluid,

with

its

vertex coincident with the surface

of the

and base

horizontal.

Find how

to divide the

so that the pressure on the upper ratio of 1> n.

and lower portions be

Centre of pressure.

While dealing with plane surfaces


able to deduce

immersed

in fluids

we were

a simple formula
pressure.

for finding the

magnitude of the resultant

fluid

We

have now to investigate the line of action of the resultant So far as the direction of this resultant pressure is concerned it
the plane. Hence if we know the point of the plane where the resultant of the fluid pressure meets This point is it, the line of action of the resultant is known.

must be normal

to

known

as the centre of pressure. the point at which the direction

We may
of the

briefly

define

it

as

single force, which is

50

equivalent in all respects to the fluid pressures on meets the plane surface.

the

surface

Take any two rectangular


in the plane surface.

Let

y) of the plane. Then area dx dy round P will be

(#,

straight lines as axes of x and y be the fluid pressure at any point the fluid pressure on an infinitesimal

p dx
(#,

dy.

We

have thus parallel

forces like

p dx dy

acting at

y\
the point

The
G,
y)

resultant

fluid pressure will therefore act at

where
-

_
"""

S!

px dx dy
p dx dy
'

-^^^
~p~cta

dy
y

_ ffpxdxdy ff p dx dy

'

_ ff p dx dy ff p dx dy'

the field of integration being the whole area of the plane*

Hence the coordinates


(

of the centre of pressure are

ff px dx dy ffpy dxdyy ff p dx dy ff pdxdy


*

Using polar coordinates, we can similarly prove that the cartesian


coordinates of the centre of pressure are
(

/*/*

r cos e

dr dB
'

ff p r

sin 9 dr d9
^ '

ffprdrdQ
Remark
1:

~ffprdrd9
rest
is

homogeneous fluid is at action of gravity alone then p~g P s, where z Ihe point below the surface. Hence
*

If a

under the

the depth of

ff

dy # ^ffz ff zdxdy' dvdy

dxdy2:

x dx

Remark

Let us take the line of intersection of the


x.

plane with the horizontal surface as

Then

z,

the

depth

51
6

of

P below

the

surface=y sin

where

is

the inclination of

the plane to the horizontal!

Hence

ysinQdxdy

ff y*Js*nJL<fady ^ff y dx dy ff y sin dx dy ff y dx dy


1

This shows that the position of the centre of pressure relative


to the plane is
It follows

independent of
if
its line

its

inclination

to

the horizon,

accordingly that

and then turned about


as a fixed
will
axis,

a plane area be immersed in a fluid, of intersection with the surface


relative
to

the centre of pressure

the plane

remain unchanged.
3'4.

In the case of

the

moment

plane lamina of inertia of the lamina about


*

ff y
x.

dx dy

is

plainly

=
wher? which

ff y dx dy

A
is

area of the lamina, & = radius of gyration about #, the line of intersection of the lamina with the horizontal

free surface,

and z

is

the ordinate of the centroid of the lamina*


.

Hence
lu particular,
\

a y z =fc

if

the lamina

is vertical,

we
the

have:

depth of the centre of pressure of


*

lamina^
2

X
2

(
,

depth of

is

centre of gravity of the lamina) fc -f-5 where the radius of gyration about an axis through the centroid is obviously a fixed constant of parallel, to the horizontal the lamina.

the

=K

Hence z

(y

z)

K2

Or, (depth of the centre of gravity below the free surface) (depth of centre of pressure below the centre of gravity) const

52

This
Ex.

is

an important result

1. A plane triangular area is immersed with its plane one side horizontal, and the opposite corner downwards* vertical, Its vertical altitude is h and the horizontal side is at a depth h
}

below the free

surface.

Show

that its

centre

of pressure

is

at

a depth -^ h below the free

surface.

(Tripos 1926)

Reducing the triangle to its equimomental particles we see that the moment of inertia of the lamina about the horizontal
that of three
is

...

the

same as

particles of

mass

Q
respec-

placed at

the

mid-poinls of the
Q
L

01

triangle, or at depths, A,

&~ 2

tively,

A being the area of the triangle,

k being the radius of gyration of the area about the intersection of the area and the free surface,

line of

Now,
for

Z= depth
O
(

of the

C.

G.

of the

area=A+- =
o

4
?,

G
Since

depth of centre of pressure below the free surface)

/.Depth of centre of pressure


Ex.
2.

=iU X? = - A 8 4A
Q t

flat

circular

plate of radius a, lies in a plane in-

clined at 30

to the horizontal,

and issubjected

to

water pressure

53

on one

face.

The

centre of pressure

is at

a distance

a from

its

geometrical

centre.

the geometrical centre is at a depth 2 a below the free surface of water. (Tripos part I 1927)
that

Shew

Ex, 3

vertically in a

Prove that in the case of a plane lamina immersed homogeneous liquid at rest under gravity, centre

of pressure

is

always lower than


as the

its

centre of gravity.

Show
vertical

that

lamina

is

lowered down the

liquid, the

distance

between the

centre of pressure and centre of

gravity becomes smaller and smaller.


Calculate
7

its

rate of variation

[Ans.=r5-

where k
its C.

is

the radius of gyration of the lamina and z,

the depth of

G.]
ni lf

Ex.
a

4.

system of particles of masses


\
(#2)

mn

*
,

...placed

M #n y\

#2^ ................. is

equimomental with a
liquid

lamina
its

immersed

in

heavy

homogeneous

plane not necessarily vertical). The axis of intersection of the lamina with the free surface.

is

(with the line of

Prove that the coordinates of the centre of pressure are the coordinates of the centroid of a system of particles of masses

m yn
\

^* y
5.

2>

........... placed

at [A^

yj,

[<r2 ,

yj

etc.

that the depth of the centre of pressure of a triangular lamina, whose vertices are at depths a, i, c below the
free surface, is the

Ex.

Show

same

as the

C.

G, of 3 masses proportional

54

to [2a-f 6+c],

placed at A, B,

respectively.
is

The
particles of

triangle

reducible to

mass ~each placed


o
this

at mid/>

points of the sides,

system

being

equi-momental with the triangular lamina, where the lamina.


rt .tience in*

mass of
C-ptt
'

tr__

wi 2

._

^ TW

Wl ~

~J and * y
,

:= 6J~T"0
"

2
the

'"
v-i

b-\~C
-

'

t/a^ 21 /s

We

have therefore

to find
(

depth of the C. G. of a
c
fl

system of 3 particles
pectively at depths

%
*J

a ^

),

5
O

^
L

J
X

)pUd ressurface.

2* *^
,

below the free

&

Hence the required depth

of 'centre pressure

m
i

~
/

b+c
ijfc
2

)(
\
i

b-\-c

\>u c+a\ ~T +I T"


.

/c

} (

w
3

Ex.6.
centre

Show

that

in the

above example the depth of the


its

of pressure

of the lamina below

C. G, is

V^

a quadrilateral having its side AD in the surface of a liquid and the sides AB, DC vertical and equal to a and 6 respectively. Show that the depth of the centre of

Ex.

7.

ABCD

is

pressure

is

55

Reduce the quadrilateral to two triangles by a diagonal and deduce the system of particles equimomental to the
t:

lamina, then apply Ex,

4.)
is

Ex.

8.

A
its

trapezium
parallel

immersed

vertically

in a

liquid

with one of

sides

the other of V at a depth the centra of pressure.

of length / in the surface and from the surface. Find the depth of A
*l
-

1')

^n

9. A parallelogram has its corners at depths d^d^ below the surface of a liquid and its centre is at a depth ^41

Ex.

show, that the depth of the centre of pressure

is

A square lamina just immersed vertically in water with one side in the surface is then lowered through a depth h.
Kx. 10.
If a be the length of the square, prove that

the distance
is

of the
a

centre of pressure from the centre of the square

6a+12

Kx. 11. A regular hexagon, whose side is a is immersed in a liquid with one side in the surface, prove that the depth ol its centre of pressure below the surface is 1.107a approx imately (L C.S. 1930)
Ex.
12.

completely immersed

square lamina ABCD is in water with its

plane vertical so that the side A B is in the surface* Draw a line B E to a point H in D such that the pressures on the

^f

two portions
if this

may

be equal.

Prove

that,

be the case, the distance between the centres of pressure: the side of the
square

= V565:

48.

Let

0^=6.

Pressure on the triangle

BC E g
is

P (a

b)(

| o

Also pressure on the whole square


If the

f g P a (~
t

pressure on B E C is equal to that on A mnst be half of the total pressure on the square.

B E D
Hence

then this

or,

D E = a/4.
x and
y.

Now

take

AB

and A

D
is

as axes of

Triangle

B E C
to

equimomental with 3 particles each of


(

mass proportional
5
(

)
<w

(<*

)=

and placed

at

(,

),

a,

a)

and

g a, ?).

Hence the coordinates of the centre of pressure of the triangle are the same as those of the centre of gravity of the particles of niasf
^L

828
(

).

),

82
(

placed at

,
-:

),

"Q

>

&

Q o^s )

respectively.

23

57

Hence the coordinates of the centre of pressure are (If a,

into equimomeutal Similarly splitting the trepezium the coordinates of the centre of pressure will be found particles,
to

ABDE

be (*&
.'.

a,

T^ o)
required distance

The

=V

(!!-:&)

+(f

2
<*

i *)

2
}

48
Ex. 13.
with their
If an area
is

bounded by two concentric semicircles


in the free
surface,

common bounding diameter


T

prove

that the depth of the centre of pressure is

(a

+ b)

(a*

+6 a )|(

+ ab+ b' )

where a and b are the

radii.

and using the bounding diameter in the free surface the initial line we have

By

3'3

as

~ _ffp? sin 6 dr de y ffpr'dtdr ffyrdtdr ffr* sinfl cos drde ~ ffv* sin' 8dr dB
sfl

drde

'

fl

'

"

sin 6

d6 dr

'

ffr*
w

sin 6

dr

cfo,

Since

j?=^P

r einfl)

f
I

J_o

r
I

___
Sinflcosedfl

C
j

rdr
r
2

f
_ - J_o r

Ja
dr

_
Sin
a

flc?J

6 r " dr * 2 r

Ja
r
dr

binfidfi

SinSdfi

Jc

Jo

~
Ex. 14.

that the depth of the centre of pressure of the area included between the arc and the asymptote of the curve

Shew

58

the asymptote being in


vertical.

the

surface

and the plane of the curve


(I.C.S.

1931)

The form

of the curve

IB

roughly as shown in the

fig.

where

TT

'

is

the asymptote.

Consider an elementry area rdOd r

round

of this point below the surface (or the asymp.


P.

The depth

hence the depth of the centre of pressure below


tote) is (r cos 0- i);

the surface

-=ff j.f 8(r cos

b)

pr

pr do dr where p , dr de
.

is

the pressure at

fS (rcosO-bY gPrdedr
7T

sec 8
a l(r cos"

2
dfl

f.

*
2

0-2 br

cos

J
i sec 8

59

plane area, bounded by the parabola y ~4ax the h and the axis y = o, is immersed in a homogeneous ordinate as liquid with the axis vertical and the vertex in the surface, and is

Ex. 15.

then lowered without rotation.

Prove that the centre of pressure

moves along the straight

line.

approaching the centroid of the area as


Let
be the lamina, where Let the depth of the vertex
free

its

limiting position.
I.C.S.

ONP

ON = A.
below

1932)

the

surface
X.

in

position be

~
f

any subsequent
"7 be the cordi-

If

nates of the centre of pressure with respect to given axes fixed in the

lamina

we have by

3*4

# =
where p
is

-_ffpy dxdy ffP* dxdy y

ff p dx dy
p = gP

'

ff p dx dy
Q

the pressure at any point

(cc,

y) of the lamina.

Now

the pressure

(depth of

below the surface)

= gP (x + X)
'
'

Sf (*4 X) dx dy

'

ff (+X) dx dy

the field of intergation being the whole of the plane lamina.

x (x+ X) dy

60

y (a+X) dy

^SV ak
4

W 3A+5X""
X from

2A+3X

,~v

(x+\) dy
To
find the

locus of ( x, y*)
( ii ).

we have

to

eliminate

equations (i ) and

From

(i)

we have
7 aT

3 h

3A+5X
3A+5X

Dividing this equation by

(ii)

v\

have

or,

48 VA

j/

== Va

75

-4

regular polygon wholly immersed in a homogeneous liquid is movable about its centre of gravity prove tl M the locus of the centre of pressure is a sphere.

Ex. 16.

Take the

fixed centre of gravity

of the lamina as origin.

Take any three mutually perpendicular lines as axes of se, y and z,


as z taking the vertical through Let the principal axes of axis.
inertia

lamina be Oa/, 0#' and Qz be normal to the lamina.


of the
f

0, the origin, is the of gravity of the lamina, centre


since

Then

ffx>

dx> dy' =*

ffy' dx> dy'

(l)

61

Again

since Oa;',

Qy

are the principal axes of the lamina


f f y da/ dy'=o

SSx
Again
the

(2)

since the lamina is a regular polygon, by symmetry iiioments of inertia of the lamina about 0*' and Qy' are

equal.

ffd* where ffdx


Hence
Let
to

dx

dy'

= ffy*

dx'

dy

- Ak 2

(3)

dy'=* A=area of the lamina,


Its

be any point in the lamina.


f

coordinates

relative

(fixed in the lamina) will be (a/, y', 0) while y x y z (fixed in space) will be (a;, y, z). If relative to the axes
Go?'
z*

axes

(/!,
cf

//

1>

wt

),

(Z a> ^27^2)7 (^3,

w 3, n s ) be the direction cosines


x y z> then two different

Oz with respect to axes y Oy', P on Oa; etc. in the projections of by considering


the lines
f

Ox

ways we have

= ^+ y +m y = MI
Zx

Z.

x'
f

y'

Let the coordinates of the centre of pressure of the lamina


in this particular position be (#, y, z) with respect to axes fixed in space and (i', y', o) with the lamina, respect to axes fixed in

since the centre of pressure

must plainly be in the lamina,

y
'

= * =

x'

x* HI

+y
'

n%

also '

y x

-SSP***W
'

y y

-S
(5)

Now
surface )

pressure at (*',

jr',

o)

is ^r

P (depth ofP below the

62

below the free surface be Let the depth of the fixed origin dj then the depth of P below the free surface is d ~z, z being the corresponding z coordinate oi P with respect to fixed axes.

But z

=====

n x
l

f-

n2

y
x
1

S.p

= ffP(d-ni
(5)

wt

y')

Hence from

we have

So also

(4)

Now, by squaring and adding each one we have

of

the

Also

z=wi ^

+w

which
axes.

is

the locus of cenire of pressure, with This locus is evidently a sphere,

respect to fixed

Prove that the locus of the centres of pressure of a series of a coaxal circles immersed completely in a homogeneous and having their line of centres horizontal is a parabola.
Ex. 17.
liquid

63

area bounded by two conjugate diamelers C B, C D and the intercepted arc is immersed vertically in a homogeneous liquid, C D coinciding with the surface.
Kx. 18.
elliptic
1

An

Find ihe centre of pressure.


Ex. 19.

(Punjab Univ.

A. 1928).

area bounded by the curve r=a (l+cos 0) is immersed in a flluid, the prime radius vector being coincident

An

with the surface.


3*5.
If

Find the centre of pressur

parallel

to

a straight line be taken in the plane of the area the surface of the liquid and as far below the
liquid
is

above, the of this straight line with respect to the momental ellipse pole at the centroid, whose semi-axes are equal to the principal radii of gyration at that point, will be the centre of pressure.

centroid of the area as the surface of the

ake the principal axes of the lamina through as axes of x and y.


\

its

centroid

Then,

if

be the area

of the

lamina, and 6, a the principal

radii of gyration,

V^fff dx dy, A a -y\/V dx dy,


2

ffx y dx dy~o.
Also,
for the C.

J*J*x dx

dy^ffy dx dy~o,
the origin.
ellipse is

G. of the lamina

is at

Hence the equation of the momental


a
Let x cos
surface,

0+#

sin

and

(or, t/)

equation of the line in the free the coordinates of the centre of pressure; then
'

0~d be the

~ffp dxdy where p is the fluid pressure at


Hence
if

dx dy "f'fp dx dy

'

the fluid

is

homogeneous and

at

rest

under the

action of gavity,

64

~g9

(perpendicular from

P on x

cos

0+y
)

sin

6~

~~ cos 9 ~" y

Hence

-SS( d -~ x cos e ~V sin

fl

* dx d

y sin 8) dx

dy

and

similarly,

=
a

sin

It follows at

once that

(*, J) is

the pole of the line

x
with regard
Ex.
elliptic
1.

cos 6-j-y sin

0==

to

the momental ellipse.

Find the locus of the centre of pressure of an lamina immersed vertically in a homogeneous fluid at

rest

under gravity and moving in the vertical plane in such a way as to be always just immersed.
Let x cos fl+y sin 6 = d be the
equation of the horizontal line in the free surface touching the elliptic

lamina with respect to the principal axes of the lamina. Let the equation of the ellipse be

r*

Since (the radius of gyration about o


a and, (the radius of gyration about o y)

a?)

=
4

the equation of the momental ellipse will be

4
is

The

centre of pressure (x, y)

the pole of

65

* cos with respect


(i)

0+j

*in

= -rf

...... (ii)

_ _ = 1 with Hence comparing ^-f-+^f

(ii),

we have

T
*= ~
Now,
4o
,

cos 9,

'

77 4o
is

st

e ......

t)

since * cos 8-{-y sin 0=rf

tangent to the ellipse

a 9 cos'

e+t*

sin* 9=<f*.
(iii)

Substituting in this from

we have

as the locus of the centre of pressure.

Ex.

2.

quadrant ot a

circle is just

immersed

vertically,

with one edge in the surface in a


varies as the depth.

of which liquid, the density Find the centre of pressure.


surface,

Take

* as the edge in the

P=|* y andp=i

|i

gy

Hence x nence

SS***ulSS*9* d* *9

JdxJ
o

X y 9 dy
o

"

f*
I

^ry
I

7"
^*y .

Jo Jo
fa
I

(*

')*</<

_JL ""
f
i

(**)*

J.

66

Similarly ;

plane area immersed in a fluid of uniform donsit y moves parallel to itself and with its centre of gravity always in the same vertical straight line. Show (l) that the locus
3.

Ex

of the

centres of pressure is a hyperbola, one asymptote ol which is the given vertical, and (2) that if a, a-f-A, a-f A', n-f-7/"

be the depths of the C. G. in any positions, y, #+&> sH-' of the centre of pressure in the same positions, then

h
h'

h(
A'

k-h

V
k"
Let

(&'-//)

h"

A"(fc"-A")

be the centre of gravity of the plane area A and, C the centre of the pressure. Suppose the area is moved parallel
to
itself

so

that

moves

in

a*

vertical straight line.

Let a ba

in the first position and # be its depth when the depth of Let C" be the position of the new centre of in the position G'.
pressure, C' being the

+A

new

position of the old centre of pressure.

In the old position the resultant of pressures and was equal to gP A a. point acted at

at

every

Now
below
is

in

the

present

position

pressure

at

every point

is

increased by
its

gPh because every point is now at a depth A Hence the pressure at every point previous position.
but
the
resultant

at the pressures various points of the area is gP A a and acts at C', the present Also the resultant of the constant ot the point C. position

+ gPh]

of

pressure

gPh acting

at

every point of the area will be clearly

Hence the new centre acting at the ceritroid of the area. of pressure G" is the point where the resultant of parallel

gPh

forces

gP A a acting

at

C and gPh A
7

acting at G'

acts.

67

Hence taking moments about C",


G' C" (gP
~C
or,

h)

C'

C" (gP A

a)

rC" = k
r\n

p/

G'C'
take the vertical along which moves as y axis C on the horizontal free surface as

Let us

now

and the projection of re axis. Let (# y


,

be the coordinates of
in

in the

initial

position of the lamina; then

any subsequent

position
f

of the

lamina when

it

nates ol C', the subsequent position of Now if (a-, ;/) be the coordinates
pressure, then since G'

has been* lowered through depth A the coordiwill clearly be (# >2/o+A).


ol
:

C", the

new

centre of

G"

C'

C"=a

A,

~
obtained by
1

Ix)cus of (x, ^) the

centre of pressure is clearly It is the variable parameter 'h eliminating


.

which

is

a hyperbola, whose one asymptote

is

05=0,

the

vertical

along which
If y^

moves.
f
i

-+-kj

y Q -\-k

4&"
to

he the depths of the centres of


the depths
a,

pressure corresponding

a~M,

of the C.

G M then we

have

(k
h'

(y

-2a)+k'a+h

(k

-h )-c -*') =
<>

68

h" y
Eliminating
(r

- 2a)+k"a+h"(k" - h"} - o
,

~2a
h

a,

we have
h

(k-h

h"k"(k"-h")h"
3. 6.

liesult.ant pressure

on curved

surfaces.

We
in a fluid

shall first consider the case of curved surfaces

immersed

of uniform density

and

at

rest

under the action of

gravity.

To
tant.

find the resultant of the fluid pressures acting


first

on a curved
this resul-

surface let us

find out the

vertical

component of

the components of the resultant in two mutually perpendicular horizontal directions. It will then to deduce the resultant from these three components. be
shall later

We

find

possible

(i)

Let the curved surface P

be bounded by t.he contour Let us drop perpendiculars

QRS P Q R.
from

point of the contour on the free surface so that the plane curve

every
q r curve

is

the

projection

of the

skew

These perpendiculars clearly generate a closed surface bounded by the cylinder on the The mass of fluid enclosed base p q r and the surface P Q R S
in the surface

PQ R

q r

P Q RS

is in

equilibrium
r

under the action

of fluid pressures

acting on the surface p q

PQR

8,

and

its

own weight

acting vertically downwards through the centroid Hence resolving vertically the resultant of the enclosed fluid. of the vertical compon ntB of the fluid pressures on the surface

q r

PQ RS
pq
r

is

equal

to

the weight of the fluid enclosed in

the surface

P Q R S and

the eentroid of the fluid

acts vertically upwards through enclosed in this surface. But the fluid

pressures on the cylinderical

part

of the surface

q r

PQR

69

have no
verrical

vertical

components at all
fluid
;

Hence the

resultant of the

components of
is

pressures acting on

the surfacb

PQ R S
surface

equal to the

we ght

of the fluid enclosed in the


G.

q r

PQR

S,

and acts through the

G. of this mass.

(it)

Secondly, consider

now

biirface

pressed upwards instead of downwards. Let A B be


the horizontal surface. Let us project the contour bounding the surface

P Q which is

P Q on the
surface
surface

surface

B.

Consider the surface formed by the

P Q and

the

projections of its contour

P Q on

the free
fluid of

B,

If this surface

were supposed

filled

with

the

same

density,

the

resultant

vertical pressure

on the surface

P Q
fluid

will be,

by the above argument, equal to the weight of the enclosed in the surface A Q, and act along the vertical

through the centroid of this mass.

But the pressure at any point of the surface

PQ

is just

the

same

in the

two

cases,

for

the depth of the point below the free


in either

surface remains the

same

case

and the pressure

in the

under gravity depends only on depth. Hence .the resultant vertical pressure on the surface P Q in the actual case is also known.

case of a

homogeneous

fluid at rest

Ex.

1,

hollow spherical vessel


together.

is

formed by two hemi-

spherical cups joined

The

vessel is placed with the

plane of the join horizontal and contains liquid up to height h abovb this plane. Shew that the pressure of the liquid produces

a force tending to
proportional to A
8
,

lift

the upper hemisphere from the lower

but independent of the radius of the vessel.

(Tripos Part

1,

1926)

70

Here we have

to find the fluid pressure

on

the upper hemisphere. In this case the liquid is pressing the surface upwards. Hence the
vertical pressure

equal to the

by what has been jnst said is weight of the fluid enclosed in the space between a cylinder of height h based on the circle in the plane of the join and the upper hemisphere

i,

e.

(
TT

a* h

Ch *

(a

x*)dx

Jo

(iii)

Thirdly,

suppose

now

that

the
i.

surface
e.

is

pressed

and partially upwards


part

partially

downwards,

pressed the part S Q downwards. * upwards and Vertical pressure on the surface S Q is by to the weight of the liquid (i) clearly equal

PS

of the

surface

P Q

is

contained in the surface


vertically

QB

acting

through

its

centroid.

Now

surface

S P

is
is

wholly
equal to
acting

that by (ii) the vertical pressure pressed upwards so the weight of the fluid enclosed in the space A S P
vertically

through

its centroid.

Hence the
is

equal to the weight

resultant vertical pressure on the surface P S of the fluid enclosed in the spaces P
its centroid.

RQ S R

and

BQR C
3. 7.

acting vertically through

next to find the horizontal component of the surface in any given resultant fluid pressure on a curved
direction.

We have

Let the curved surface in question be

PR

Q,

71

Let us project the surface P R Q on a vertical plane perpendicular to the

given horizontal direction. Let p q be the curve into which the surface thus
projects.

Consider the mass of fluid enclosed in the space p q P R Q. It is in equilibrium under the action of (i) fluid pressure acting on the given surface P R Q (ii) the fluid pressure on the plane

on the cylinderical surface generated by perpendiculars from points like P of the bounding contour P Q of the surface P R Q on the vertical plane, and (iv) the weight of the fluid which acts vertically, (and has therefore no horizontal component^.
q
(iii)

curve

the fluid pressure

Resolving horizontally aldng the. given direction we see that fluid pressures on the cylindericai surface being everywhere at right angles to this direction have no component in this
direction.
It

follows that the horizontal

direction of the resultant fluid pressure on P the fluid pressure on the plane area p q. Hence the required component is equal to the fluid pressure on the plane area q and passes through its centre of pressure.

component in this R Q must balance

we

the resultant fluid pressure on any surface have therefore to find (i) the vertical pressure, and (ii)

To determine

resultant horizontal pressure in

any two
in

direction.

These three forces


into

may

some

a single force, in which case line of force meets the surface is called the centre of pressure

be compounded the point in which the


cases

of the surface.

Ex.

1.

vessel in

the form of an

elliptic

paraboloid,

whose

axis, is vertical

and equation

^+j= J*

{B

divided into

72
its

four equal compartments

by

principal
A;

planes.
if

Into one of

these water

is

poured to the depth

prove that,

the resultant

fluid pressure

on the curved portion be reduced


horizontal, the
line

to

two

forces,

vertical

and the other

of action of the

latter will pass

through the point (~,

5~
f

%
We require the
on the curved

Let

OAGB

be one of the four equal parts into which the


divided by the planes xQz and yOz, the plane
water.

elliptic paraboloid is

ACB

being the horizontal surface of the

horizontal

component of the

fluid pressure acting

surface

AOB.
horizontal

The
is

component of the pressure

parallel to

Ox

equal and opposite to the fluid pressure acting on the projection


z
i.

of the surface on the plane y


equation
is

e.

on the parabola BOG, whose


line.

y*/&*=z/h, x=o,

BC

being the water

The

coordinates of the centre of pressure

are (#,

^, z)

where

jpydx

16'

fh
I

pz ydz
....

rh gP (hz)zl
VA
-.._

^T and, z =

J. o

JjQ

J^
o

-vA

73

Hence the horizontal component of the


to ox acts at , \o^

fluid pressure parallel

5b
*g
>

3A

Similarly
parallel to

the horizontal component of the fluid pressure t n OL


at
(
,

oy acts

0,

).

Ct^

QI
,

The

line

through

(o,

parallel to ox clearly

meets

the line through


3

(-JL

parallel

to

oy in the point
the point through

jg

y-)>

which

is,

therefore,

which the resultant of these two horizontal components must pass.


Ex*
2.

hemisphere

find the resultant fluid

with a homogeneous liquid; pressure on one of the four portions into


is filled

which

divided by two vertical planes through right angles to each other.


it is

its

centre at

(Resultant force

= -^^P a
:

tt

^+

acting along x

y=

2 ^a)

Ex.

3.

hemispherical bowl

is filled

vertical planes are

drawn through its semi lune of the surface; if 2a be the angle between the planes prove that the angle which the line of resultant pressure on the
Hurface

with water, and two central radius, cutting off a

makes with the

vertical is tan (

Let

ABC

be the semi-lune of the


off

hemispherical surface cut


vertical planes
to

by two
inclined

AOG

and

BOG

each other at an angle 2a.

The
equal

vertical

component of the pressure on the surface

is

to the

weight of the fluid enclosed in the semi-lunar surface

74

ABCO. Now the area of the circular quadrant is


troid

|
taken

with
as

its

cen-

*a
at

(-^L, JjTT

Qlt

-) OB,

00

being

axes of

coordinate.

Volume

of the surface thus

genarated

(area of the quadrant) (distance travelled by

as the

quadrant moves from


/

its

position

BOG

to

AOC),

4a

2a

Hence the

vertical

component of the

fluid pressure is

|0Paa.
Horizontal component of the fluid pressure perpendicular
to the plane

BOG

is

9-

=
(-Jj-)

IgPa*, acting at Q.
fluid pressure per-

So also the horizontal component of the


pendicular to the plane

AOC
at

is J

^Pa* acting

at P.

By symmetry, normals

at

P and Q to

AOC

respectively meet

so that

<PMQ = *
3

the planes
2a.

BOG

and

Now, the
aud

resultant of equal forces

gPa* acting along

MP

MQ is

clearly |

^Pa

cos
s
(Z,

(^~) = I gPa* u
the angle

sina. Since the

vertical

component
-

is |
,

^Pa
"\\ (|

between the resultant


.

and the
Ex.

j.i

vertical

is

tan

gPa *p 8

n --

=
in

tan

nv
).

4.

vessel full o:T water

part of an ellipsoid (axes a, b, c), planes. The axis c is vertical, and the atmospheric pressure is neglected. Prove that the resultant fluid pressure on the curved
surface is a force of intensity J^P
{

the form of an eighth bounded by the three principal


is

iV

-f a

V+ We*
i*-

*'
}

75

the form of a quarter of a circular b is immersed in a fluid of cylinder of radius a and axial length in the free surface. Neglecting density P with its axis and edge

Ex.

5.

surface in

of the fluid atmospheric pressure, show that the resultant pressure 9 on the concave surface is *093<;Pa 6 and is inclined at 32,5 to
the vertical
(Tripos, Part
T,

1925).

3 8.

Fluid pressure on a
shall

solid.

that the resultant fluid pressure on the surface of a solid either wholly or partially immersed in a fluid at rost under gravity is equal to the weight of the fluid displaced

We

now show

and

acts vertically through the centroid of the displaced fluid.

Draw

parallel

lines D, P' touching the surface


it;

and forming

a cylinder which encloses surface of the solid into two


tal

the

curve of contact divides the

parts,

on which the resultant horizon-

the axis of the cylinder are equal and pressures parallel to It follows that the fluid pressure. on the solid opposite ( 3.7).

has no horizontal component and

is

therefore entirely vertical.

lines parallel vertical E, E' touching the surface and let

Now draw

be the curve of the cylinder generated by E, E' and the surface


of the solid.

C D

The curve C

D divides

the surface of the solid into two parts, on one of which the resultant vertical pressure act upwards, and on the other downwards, the difference of the two is plainly the weight of the fluid dis-

placed by the

solid.

76

The

resultant pressure

is

therefore

equal to the weight of

the fluid displaced.

Remark
immersed
partly of
in

If the bounding-

surface

of a

solid

completely

fluid

consists parlly
areas,

of a carved surface and


easily find the resultant

known plane

we can

fluid pressure on the curved surface

Suppose

for

the

sake of definiteness the solid consists of


surface.

two plane areas and a curved

The

resultant fluid pressure on

the

entire curved surface of

the solid viz 8, and the two plane areas is vertical and equal to the weight of the fluid displaced, acting through its centroid.

Now
in

the pressures on the plane surfaces can be easily found both


direction.
.

magnitude and
is

Henc<3 the resultant fluid pressure

on S

clearly
1.

known.
unclosed curved surface which has a plane curve
is

Ex.
for its

An

boundary,

immersed
liquid

in a

homogeneous

liquid.

Prove
is

that the thrust

of the

on one side of the surface

equi-

valent to two forces, one vertical and the other perpendicular to

the plane of the boundary.

Slate precisely the magnitude ot


its line

each force and the position of

of action.
is

Ex.

2.

cubical box of
lid

edge a

made

of thin metal and


side.

has an accurately fitting


lid

smoothly hinged to one


s

The
its

has a piece of uniform metal of specific gravity

forged to

inner face, the volume of the metal and the distance of its centre
of gravity from the lid being
filled

v and
lid

h respectively.
is

The box

is

with

water and the

being held closed,

placed with

the side to which the hinges are attached on

a horizontal table.
not open

Prove

that,

if6(s

1)

vA>a*

>

the lid will

when

released.

(Tripos Part I 1923.)

77

Let

BODE

be the

lid

and

BC

Fluid pressure on the lid acts normally to the lid at P


the line of hinges.

which
fluid

is

the centre of pressure.


is

The
^ 2
)

pressure
)

clearly

gPa*

= kgPa 3
pressure

acting at the centre

of
is \ a,

P whose depth below ED


o
,

or

whose height above


This pressure tends

BG

is

-~-

P being the density of water.


lid

to rotate

opens the lid. The weight of the metal of volume v forgrd to the lid has opposite
the

about

BC

outwards

i. e.

tendency.

The weight
v
(sPff).

of the
fluid

Also the

metal acting downwards through G is pressure acting on the solid is equal to


it,

the mass of the fluid displaced by

acting also at

but vertically

upwards.

This

fluid pressure is

gPv
at

Hence, the net vertical force acting


force tends to

is

gPv

($-1).

This

counterbalance the effect of the fluid pressure in

opening the

lid.

Hence, taking moments about BC, the


long as

lid

will

not open so

fP*(f-l)*>**Pa
or,

(-f-)

6 (s-1) h

v>a

4.

Ex.
depth

right cone is totally immersed in water, the of the centre of its base being given. Prove that, P, P', P"
3.

when the being the resultant pressures on its convex surface, &ines of the angles between its axis to the horizon are S, S', S"
respectively

P3
Ex,
4.

(S'-S")

+ P" (S"-S) +

P"* (S-S')

o.

a tangent line

formed by turning a circular area round through an angle 0, and is held under water just
solid is

78

immersed with its lower plane face horizontal inclination between the horizontal and the resultant
on the curved part of the surface of the
Ian
solid,

If
fluid

be the
pressure

=
is

tan

6 --

+ cosec

6.

Ex.

5.

solid

formed by

turning a parabolic area,

bounded by the latus rectum, about the latns rectum, through an under water, just immersed, with angle 6 and this solid is held Prove that, if be the inclinaits lower plane face horizontal. tion to the horizon of the resultant pressure on the curved surface
of the solid,
a 3 sin 6 tan

=5

sin 0-3 sin 6 cos


is

20.

Ex.

6.

hollow ellipsoid

filled

with

water and placed


its

with

its

a axis making an angle a

with the horizontal and


fluid

c-axis horizontal.
surface on either
is

Prove that the


side

pressure on the curved

of the vertical

plane through the a-axis a2

wrench of pitch equivalent to a


3c sin

a cos a
2
4c a

+9

(a

sin'a

2
fc

cos 3 a)

Let OA, axes of the

OB

be the a and b

ellipsoid,

OA

making

an angle a with the horizontal. Since OG, the c-axis is horizontal and normal to the plane of AOB, hence AOB must be a vertical
plane.

Let

0V

be the vertical in

in th s plane.

Let us take OA, OB,

We
on

have

to find

respectively. y and out the resultant pressure on the curved surface

OG

as anes of

or,

either side of the plane

AOB.

The
of

00

horizontal component of this pressure in the direction is the pressure on the curve of projection of this surface

79
to

on the

vertical plane

normal

00

projection is precisely the ellipse

on the plane AOB. This with axes A A' and B B' whose
*'.

e.

equation with regard to

OA,

OB

as axes

is

Let LL' be
since
it is

the

horizontal line touching the ellipse, then

inclined at an angle

to

OA,
a

its

equation will be

sin

-f-

y
Z>

cos
2

a'sin'a

= cos*a =

rf,

where
...

d2

(2)

Now
(1)
is

the pressure on this plane area bounded Jby the ellipse gP (area of the curve) idepth of its C. G. below the

horizontal)

= gP

ab

rf,

where d

is

given by

(2).

This pressure of course acts normally to the plane therefore its line of action has direction cosines (o, o, l).
Further It must act at
of the centre of pressure.

AOB

and

P (,

*l),

these being the coordinates

ifl is the pole with respect to the momenta! of a line parallel to the horizontal line LL' ellipse of the area, and as far below the centroid of the area as LL' is above it t e.

By

3.5,

P (.

of the line x sin a

+ y cos a
m
I

d.

Also the equation of the

momental

ellipse is

a 1 14
IT Hence

TTT~.

r s

*=

<*>

sin

a
7
,

4
ti

a
8

-*1 g
4

a d

'

Thus the horizontal component of the pressure on the curved surface is w ab dgP acting along the line whose direction cosines are (o, o, 1) and which passes through

'

,
f('
*

o)

...

(3)

Cl

The
surface
is

vertical

equal to

component of ihe resultant pressure on the the weight of the liquid enclosed by the surface

and acting

at its centroid.

Now

the volume

of the

liquid

enclosed by

the
7r

surface in
ab<-.

clearly half the total

volume

of the ellipsoid, viz.

Hence the
i".

vertical

e.

and

parallel to acting at the centroid

0V

* abc component is gP acting vertically whose direction cosines are (sin a, cos a, o)
of the
i. e.

semi-ellipsoid into which thn


0,

plane divides the ellipsoid

at (o,

(4)

X, Y, Z, and L, M, N be the resultant pressures and the resultant couples about the axes, respectively, then from 3) and (4)

Hence

if

parallel to the axes

X =

^ abc gP

sin a,

Y
Z

* abc gP
9

cos

= *abdgP
-

gP ab

2
(fe

-J- 6**)

cos

M =
N =
o.

gP ab

(a*

+ c*} sin

a,

Hoiice the pitch of the wrench (X, Y, Z, L, M, N), to which


;lw iliiid p/essure is equivalent

XL+JYMjfZN
'

X 2 + "Y +
2

Z2
sin

a cos a

81

~
8.J).

_ T"9 4?

(a sin

<r

a~+6*"ooe

___J
'

a).

Let us

now

find the resultant

pressure on any surface


forces.

of a fluid at rest under the action of

any given

have already seen how to find out pressure at any point of a fluid at rest under the action of any given forces.
Consider
fluid;

We

now
(cc,

a surface

S exposed
it.

to the action of such a

and
Let

let

y, z)

be any point on
direction

(Z,

m, n) be the

cosines

of the normal to the


is

surface at P.

Let
#, z

be the pressure at P; p
forces that act

then a known
fluid.

function of

.r,

and the given

on tho

Considering an element S of the surface about the point P, the force on this element is p$S acting along the normal at P to
the surface.

np$S

This force therefore has components Ip 8, mp S, We have thus an infinite number of parallel to the axes.

such forces acting at every point of the surface. We know from statics that these forces can be reduced to three forces X, Y, Z
acting along the axes of coordinates and three couples L, M, whose 'axes' are also the coordinate axes, where

x Y =
Z

=
= Sp
(ny
(/*
-

mz)

S=y\/' p

(ny

//) dS
f

=Sp

nx)
ly)

*S=yy p (lz - nx) rf8


58=

N = Sp
the integrations being

(mx made

ffp (mx - ly)

rf8 f

to include

the whole of the surface

under consideration.
Again, since
z o
I

d S
I

is

y plane therefore

the projection of the element d S on the S=r/// d z hence the three components
;

of the force X, Y, Z, aro also equivalent to

82

= ff pd y

dz,

Y-yY" p Iz
dx,

dx,
to

Z =1

ff pdx dy]
)rf/y

and the couples L, M, N, are equal

L~yy* p (ydy zdz ) N~yV* p (xdx

M=/V* p (zdzJc dx
dz.

ydy)

In the particular case of a fluid at rest under gravity let us take the free surface of the fluid which is known to he a horizontal
plane as

Let P" o y plane. (x, y, z) on the x o y plane.

be the projection of any point

We

know by

the

above that the component of the


is

fluid

pressure along the vertical

Z~

ff pdx dy,
curve
into

(1)

the field of integration being the projects on the x o y plane.


liquid with a small area

which the surface

Now
v//

the weight of a column of


as base

ax dy
I*
I

at

and height P" P

is

dx dy
But
/.

gP dz
dz,

dp=gP

the weight of the fluid in the column

P" P

is

dx dy
is

JV.

dp~p dx

dy,
surface
is

the pressure at P, (the presure at the free

to

be zero).

in

Hence by (1) Z is equal to the weight of the fluid enclosed the surface and its projection on the free surface.
3. 10.

We

proceed to deduce

now

the results of

3.

6 analy-

Suppose that the fluid, homogeneous or heterogeneous, tically. is at rest under the action of axis gravity only, then taking z so that is a function of z as vertical dp Let #P d z, only.

83
(z)

ydz

= ff$

dy dz
9

This expression clearly shows that the horizontal component


of the pressure on the surface along o x is the same as the pressure on the curve, which is the projection of the surface S on the yon
plane.

For
;

if P'

be the projection of any point


z coordinate
at P.

of

S on yoz then
pressure at

P and P have
P' is also
(z),

the same
the

so

that the

same as

Similarly OZ.
3. 11.

Y is

equal to the pressure on the projection of S on

If

the surface

is

closed,

we have by applying

Green's Theorem,

X=
L=

~
I

dx dy

cfe,

etc.

and

(y

~*

d dy *>

the integrations being made over the entire space enclosed by the surface, and p being the value of the pressure function at the
point
fluid
(a;, y, z) of the enclosed space which is supposed to contain with the same law of pressure as the surrounding fluid.

Ex.

1.

small solid body


at

is is

held at rest in a fluid in

which the pressure p

any point

rectangular coordinates (a:, y, z)\ the couple which tends to make

a given function of the prove that the components of


rotate

it

round the centre of

gravity of

its

volume are

and two similar expressions, where A, B, 0, D, E, F, are the moments and products of inertia of the solid with respect to axes
through the centre of gravity.

We know

that,

with centroid as origin

84
dz,

(f +s

2
)

dx dy

B^fff

(z

+a

dxdy dz

y z dx dy dz,

E^fSJ* z x dx dy dz,

F^fffxydxdydz.
Now

=
the space enclosed by the solid. by hypothesis is small and origin is the centroid Let (#, y, z,) be the coordinates of a neighbouring
y, z

integration to be

made over

But the
of the

solid
solid.
a?,

point so that

are small.

Hence

3z

8a

3*3*

3P^

*P

^
o

^ +
have,

v (

3//

Substituting in (1)

we

r L

yW* y dx dydz = y v =
Ex. 2
2 Sin

o, etc.

Prove that the resultant thrust on the curved surface of a right circular cylinder completely submerged with its axis
at

an angle a

to the

vertical is

a and

acts at right angles

85

to the axis

through

its

middle point,

denoting the weight of

the liquid displaced by the cylinder

(Punjab Univ. 'M.A.' 1935).


a liquid with its plane vertical. It is rotated in its plane about a fixed point lying in the surface of the liquid. Prove that the locus of the centre
3. is

Ex.

plane lamina

immersed

in

of pressure of the lamina

is

part of a straight line so long as the


is

lamina

is

totally

immersed (atmospheric pressure

neglected).

plane lamina in the form of a regular polygon having n *wles oi length a is totally immersed. Prove that centre of
pressure lies

wulun

or on a similar polygon of side

12L

IT a F oi 2 cos-n

* ~ Sec -

1 \ J
(Tripos Pt. II 1927)

Ex.
radius a
h.

4. is

uniform sphere of weight w, sp. gravity a and wholly immersed in water with its centre at a depth
that
if

Show

through

its

centre

the sphere is divided by a vertical plane and the hemispheres are smoothly hinged

together at their highest point the magnitude of the least horizontal force through the centre that would have to be applied in order to separate them is 3 / , 4h

is

w(1

+*

(Tripos Pt. II, 1925).

Ex.5.
action

mass of homogeneous liquid is at rest under the of a force whose potential is a quadratic function of rectancoordinates,
so

gular

that the
if

surfaces of equipressure
is

are

ellipsoids.

Show

that,

a body of any shape

held immersed

in the liquid, the resultant thrust on ,the body may be represented as a force acting through G, the centroid of its volume, and

directed along the normal to the surface of equi-pressure through G, together with a couple which depends on the orientation of

the body but not on the position of

G in the liquid.
(Tripos. Pt. II, 1924).

86

Let

be the potential so that

VWe also know


By
3.11

thatj?-f-V=cons.
that the

components of the lorca due to the pressure of the liquid are given by (X,Y,Z, system

we know

L,M,N) where

L=///(y|P
the
field

-a

|P) ay'

dx dy dz

etc

of integration being body.

the region enclosed

by the

solid

+ E.y+Gz-\-u) dx dy
(He, y,

dz

where
of

t is

the volume of the solid and

the co-ordinate 8

its C.

G.

Transferring the point of action of the force components (X, Y, Z) to the centroid of the mass viz (5T, y,T) the components
of the force acting at
~

are proportional to
v):

orF?Yl x
*-3x

i >y,z=*, y,
to

(G *
>,

.f^l L
'

3 Jx,y,z=Ar, 3y

__ -F^l L
'

^, *

8 2 ^x,y,z=.v,

or, proportional

the direction cosines of the normal at


equipressure passing through G.

to

the surface of

Thus

87

the resultant force


surface,

is in

the direction
of the

of the normal to this

The

transference
Y, Z) to

the forces (X,


couple,
z

point of application of involves the introduction of a

whose components are

jTZ, lr

z X,

VX

Y.

Thus the system reduces


components

to a force

through

Gand

a couple of

L-f z
Let

Y~y Z,
</ t

etc.,

o, b> c, /,

of the body centroid G.


;txes

k be the moments and products of interia about axes parallel to x y z passing through the Let a
1

V,

c' f /',

A'

be similar quantities about

xy
Then

z.

!^ffj\tf+z*} dx dy

dz,

dxdj
dx dy
It follows at

d*

dx dy dz
d*.

once that

fff**
2
2

dx dy dz~ A'i?'~~'
),

ete.

Also,
t

a'-a+

Mj +7
Y
y

/
solid.

being the volume of the


L-f-"S

Now, we have

-f 2

7iH"
(~2P)
{

(Ph'-Hg')4(C-B)

f-f-F (o'-b')

88
)-h

+ 2Pt (G
{

lr

y
r

-H1E7

(C-B)7*+F
(sf-tlT*)

(y

) }

=(-2P)[

G(*

-tFy )-B

This depends only on A, B, C,...and

a, b, c...,

none of which

depend on the position of the centroid. a, b, c...,depend only on the orientation of the solid. Thus the couple depends only on the orientation of the body but not on the position of G in
the liquid.
hole in the Bide of a ship is closed by a circular door 5 ft. in diameter hinged at the highest point and held inside against the water pressure at its lowest part by fastening. If the highest and lowest parts of the door are at a depth of 4
6.

Ex.

and 8

ft.

find the least force exerted


7.

by

fastening.

hemispherical body of radius a is entirely submerged in a liquid of density P so that its diametrical plane makes an angle with the horizontal and has its centre at a
c^epth h.

Ex.

Prove that the resultant fore* on the curved surface

is

a*gf>

(Tripos

pt, I,

1920;

I.

C. 8. 1935)

Kx. 8
its

portion of a sphere cut off by two planes through


at

centre inclined to each other

an

angle

is

just

immersed

in a liquid

with one face in the surface.

Find the resultant


it

thrust
-i

on the curved surface and show that


7T

makes an angle

tan

(--

l)

with the horizontal(I.

0.8. 1937)

Ex

9.

In the two immersed ellipoidal surfaces

89

>', i<6\ e<c\e-axis is


ut a height A

vertical

and the surface of the water

is

(>c) above the plane dfxy. Prove that the resultant horizontal pressure on that part of (i) which lies on the positive
of y 7. and without the surface
(ii)

is

6V
^ v'u*^/ 1 ~<i *b
f

^a*

c'

a /a

c )

W,

being tbo \veighl of wat^r displaced by volume of


(I.

(i)

8.

1935)

Kx, 10.

plane area bounded by the arcs of the four


,

parabola*

y*s*a x

^4bx, x*~4cy and


with x

f
aj

=4Jy

is

placed ver-

tically in a

Shew

homogeneous liquid that depth of centre pressure is

axis in the liquid line.

Kx- 11.

heavy heterogeneous liquid


is

fills

a semi-ellipsoid
in

wlos* equation
the liquid be in

Q,

--f+ O

=1.
C

Show
fluid

that

order that

equilibrium

the

arranges

itself in

such a

way
only

that
i.

e.

density of any element is a function of of its depth below the horizontal surface.
1

V coordinate

""
prove that the resultant action
it is

Assuming tha* P^iire


011

oae of the octants into which

divided by the vertical

planes viz

x^

and

yo
-

is

a single force acting in the line

rTiJT

(T-FT)

"r(i)n'-)

90

>

Ex. 12.

cubical box of side a has a


It is
tilled

heavy

lid of

weight

water, and held one extremity of this edge vertical. If it with the diagonal through be now made to rotate with uniform angular velocity w show must not be less than that in order that no water be spilled,

W. movable

about one edge.

with

"
6

2V

where
Ex.
13.

is

the weight of (he water in the box,

A liquid of depth iJa and uniform density P m on a liquid of density I P and depth greater thau a. superposed A circular lamina of radius a is placed with its plane vertical
and
its

centre in the surface

common

to the

two

liquids.

Deter-

mine the depth of the centre of pressure neglecting atmosphenV


pressure.
,

(Tripos part

19S7)

Kx. 14.
of radius a
vertical

A
is

form of a right circular cylinder held with its axis inclined at an angle a to the
vessel in the
IK

and contains a volume of water which


base of the vessel
is just

such

that

tix*

circular

covered.

Show

that the resulHud'ac**

tant of the pressures exerted by the water on the cur red of the cylinder is a single force of the magnitude
" 9 gP a (Sec a- COB

a),

where P
pressure
is

is

the

density

of the

\vater r

and a tuiosphenV

neglected.

thin glass flask consists of a portion of a sphere of radius a on a plane circular base of radius a cos a, and a neck in the form of a circular cylinder of radius a cos a. The flask is symmetrical about a vertical line. Water is poured in until

Ex. 15.

the horizontal thrusts on half of the

neck and on half of the

9i

spherical

Obtain portion bounded by a vertical plane are equal. an equation giving the depth of water in the neck and deduce that whatever the value of a the ratio of the depth of water in the neck to the depth in the spherical portion must exceed
1

V2.

rectangular block whose edges are of lengths divided by a plane through the centre perpendi2a, 2i, cular to the edge of length 2 c, and the two halves are hinged

Ex.

16.

2c is

together along
is

edges parallel to those of length 2a. The whole then immersed in a liquid with the line of hinges inclined at
6 to the horizon

an angle

hinges being in

and the dividing plane vertical, the the upper face. Prove that the two halves will

not separate unleess

pc*
where d
is

j?//

\Cos6
a the

the depth of the centre of gravity of the block, density of the block and P that of the liquid.

whose vertical angle is 2a, has its lowest generator horizontal and is filled with liquid prove that the 3 15 Sin resultant pressure on the curved surface is V ( 1 a)
Ex. 17.
cone,
;

times the weight of the liquid.

CHAPTER

IV.

Equilibrium of Floating Bodies.


shall first consider Principle of Buoyancy: the case of a fiuid,-liquid or gas, homogeneous or heterogeneous, at rest under the action of gravity only. If any curved closed

4.1.

The

We

surface be traced out in imagination in such a fluid, the pressure* exerted on all the elements of this surface by the surrounding
fluid have a single resultant, which is equal and opposite to the the weight of the fluid enclosed by the surface. This follows at once from the fact that the fluid inside the surface is in equili-

brium under the action of


(i)
its

own weight

acting vertically through

its

0. G.

and

(ii)

the fluid preisures acting normally on the surface


resultant of
(ii) is

The
(i).

If the

curved surface

is

accordingly equal and opposite to not one merely traced out in imag"

ination in the fluid but the surface of a solid body placed in the Hence the resultant pressure of a heavy fluid, result is the same.

body placed in it is a vertically weight of the fluid displaced by the upward force equal If the solid, acting through the Q G. of the displaced fluid. under the action of its own weight solid is in equilibrium solely and the fluid pressures on its surface, the weight of the solid must be equal to the weight of the displaced fluid and the centres of gravity of the solid and the displaced fluid must be in the
fluid

on the surface of any


to

solid

the

same

vertical line.
1

Ex.
W"

and weight

normal cross section has a mass of metal of small volume and weight
uniform rod
B, of small

attached to

extremity B. Find

the condition

that the rod

shall float at all angles in a

given homogeneous

liquid.

Let

A B

be the rod,

its

lenght

be the C. G. of being 2 a. Let the rod and the piece of metal attached a HtB, BO that

BG=

r+i
n
If

P and

vely,

we

be the density of the liquid and the rod respectihave \V = 2 acgs, and by the first condition of equilibrium,
s

weight of the
displaced,

solid

must be equal

to

the weight of the fluid

we have

gP

BE =
the

1
(

7v

+1) W, c being

the cross section of the rod.

A Iso

BE

must be

identical with the point G,

0. G. of the fluid displaced viz. the mid-point of by the second condition

of equilibrium

Thus

E=i BO -

T+T
n

2a

n
or,
(

n
is

-f-

'

s^ Pw 1 which
j

the required condition.

Ex

2.

solid ellptic paraboloid

whose equation

is

and of height h is floating with ita axis vertical and vertex downward. If the horizontal surface of fluid is at a height k above the vertex, prove
h*

P
as;

p and

cr

being the density of the liquid and the

solid respectively.

The volume
the plane
section

of the solid bounded by

*~h

is

fkab
I

z for

ihe
ellipse
so*

by any plane
V*
au(^

Z-* is

an

of axes a

Wz

respectively

that the

volume of a thin element of thickness dz


is

is

Hence
solid is

if

or

the density ot the solid, the weight of the

^00

ab h*.
c

&
is

Similarly the weight of the fluid displaced

k
'KabgY t
c

z
o

2c
first

fLpfljfe^p

being the density of the liquid


cr

Hence by the
Ex.
S.

condition of flotation

A*~P/fe a .

hollow hemispherical shell has a heavy particle fixed to its rim, and floats in water with the particle just above the surface, and with the plane of the rim at an angle of 45
to

the

surface;

shew
it

weight of the water

that the weight would contain

ot the

hemisphere: the

;;4V2

5;

6V2.

Let a be the radius of the hollow G' its C. G. so thai hemisphere, a

Let

be

its

weight
at

and
B.

W
downwards through

that of the heavy particle floats under the action of

The

hemisphere
(i)

a force

acting vertically

;
,

(ii)

a force

95

(iii)

the

resultant

BC
i.e.

pressures acting on the surface of the hemisphere, which by the principle of


of fluid

buoyancy

is equal to the weight of the fluid displaced the fluid that can be contained in the segment

bounded by the horizontal plane

Taking any plane


at a distance

#t

right

angles to

P,

the

vertical

* fropd 0, through small element of thickness dx ib^a*

we

see that the


dz.

voJuwe of a

#*)

Thus the

of

the spherical segment

IB

fOQ
I

(a*

x*)dx.

JoP
Taking moments about B, we have

)=

JL.

gP
V2

a
o2 _^, Va
.

JOP
I

{J

/JJ

jfliC

to

height of a right weular the diameter of the base it wij) float with
4.

Ex.

11

the

it* slft^t

side

horizontal in

any

liquid of greater density.


elliptic

Ex.
(2 a,

elliptic

major au<J miwff axes 6) homogeneous liquid with vertex of the yection at the top jusj; aljpve thp If the angle liquid.
cylipder
floats

5.

An

of

in a

that the axis


ot the

makes with the

vertical
its

heigut h of the cylinder to

prove that the ratic semi axis a is


0,

be

Let the plane of the paper represent the vertical plane through the major axis AB. Let AE represent thet race of the
horizontal plane

of

flotation.

Since

the angle between the vertical and the axis is 0, the angle between AB

and

AE is

also 0,

Hence taking AB,


to

AC

and a

line

through

perpendicular

the plane of the paper as the equation of the cylinder is

axes of x, z and y respectively

~*
Now by

b*
z

and of the plane AE,

xtan

6.

the principle of buoyancy the C. Gr, of the cylinder and the C.G. of the fluid displaced by the cylinder i.e. i.e. vizG" must be on the same vertical line, of the portion It is obvious that C.G., G, of the portion AEB must lie on the

ACDE

line

GG"
Now

so that

GG"G'

is

a vertical st line and therefore

at

right angles to

AE.
If

any ayt the volume below the plane


of the element being (x, y, *

the coordinates of G* are easily computed. element dx dy of the section of the cylinder by

we

takn

AE
tan

is

x tan

dx

the plane dy, the C* G,

9).

Hence

if ( i, y,

It

be the coordinates of G'

ff ff
z ~~

>

x tan

tan e

dx dy dx dy,

ff
^
o

x tan

dx dy
from symmety.

and y

dx
x =.

(x
a-

*tan

dy

5a

TSimilarly

5a -~- *on O

*.

Also clearly coordinates of

(3-

are (a

o,

).

A
direction cosines

Since

6?' is

at

rt.

angles to A.E

whose

are (cos

9,

of

ss'n * v\

we have
(
)

l-f

sin 9

or '

^
ratio of the densities of
is

Ex.

6.

Prove that the

the cylinder

and

liquid in the

above example
<r

See*8

P
Ex,
7.

2+

5 ton 2

9.

Prove that a triangular prism .floating with horizontal has in general three positions of equilibrium
.

its

edges

Ex.
has
its

cone whose height is h and vertical angle 2 a vertex fixed at a distance p beneath the surface of a
8.

solid

homogeneous liquid. The cone is inclined at an angle to the


thesurface prove that

in

equilibrium

with
base

its

axis

vertical

and

its

above

cot 9 cos

*a

(9 -f

Let the plane of the


paper represent a vertical
plane through the axis Let of the cone.
.1 to

VZ
be

/I

VX

VZ

in this

plane and

VY
Z.

be

to the plane

XV
as

With VX,VY,VZ

rectangular axes the equation of the right circular

cone

is

(*

+y

*)-z* (an

a,

a being the semi

verficai

angta

Let the trace of the horizontal plane of flotation on the Draw VX 1 parallel to AB. In order vertical plane XVZ be AB.
to

find the

flotation, let
is JL to

equations of the section of the cone by the plaue of us transform axes to VX', VY and VZ , where VZ'
7

VX'

in the plane of the paper.

Let

<X V
P

X'^0, then

if

jfy'X

) *

the coordinates of

point

( .v,j/jZ )
.r

in the i*ew system

x
y'

cos 8

f 2 sin 9

z^
9-z

x sin

9 -f

ff

ca* 9.
iji

Hence the eqwtiofl of thp cppp

the
1

new
v
6

coordinates is
cot 9

(' VH

tin

u )
-f

/f

=( x

+ z'

of the horiftootid plane


z***p, where VZ'*=^p
.

AB

is

clpar]y

Hence the equation of the


flotation is

elliptic

section

by the plane of

99
*

(x

cos 6

siii 6) 2

+y

*( *' sin 6-f


2'

cos 6

)*tan*a

and
,

*=.

(t?

/_
2 Cos~

Sin

fl

7 Sec a

-Sin 2

or

__

(Co8

-8in Ttan a)
and z
f

__
a
2

^ tanV
>

/f

jp

tan > ct

=p.
elliptic section
2 Q 8 Sec cc

Hence

if

be the centre of the


i

Also, the

volume of the

fluid displaced

volume of the cone


1

VAB
AB)

(area of the elliptic nection

Let C. G. of the cone

V A B be
G

G'.

on the

line joining
flotation.

to the centre

Evidently it must lie of the ellipse cut out by the

plane of

Hence the
vertical force of

resultant fluid pressure

on the cone

is

an upward

magnitude

w jP
3

tan* a

acting through G'.

The cone
its

is

in

equilibrium under the


1

action of this force,

tyaA'tan *, acting and the action of the hinge at V.

weight

viz.

at

where

VG=t V Z,

Taking moments about V, we therefore have

100

Sin CVZ')

3(Ccs 0-Sin

'

tan a (V
or p*P
tan'a

Sin

Z')

(Cos'a-Sin'fltan'a)*
or,

Pp tan*a cos 8 sin 6 sec*a "* (cos*e-sin*etfln*a)


4
s c0$ a cos a

fan

Pjp

=a
=

7i

4
[cos*fl

co
fl

A4

[cw

cos ( 6

+a

6/a
)J

above example the density of the liquid r varies with depth according to the law JA z find the correspon9.

Ex.

If in

the

ding

relation.

Ans. n

p*

cos 8

cos*a= (r+4) a
is

h*[cos (8

a) cos

Ex.

10.

A solid
about

y=/X
is

),

its

axis of

formed by revolving a symmetrical curve X, (vertex symmetry which is


its

being the origin 0). It is divided by a plane through aud the parts connected by a hinge at the vertex* The

axis

system

now

ward.
axis

placed in a liquid with its axis vertical and vertex downIf it float without separation and k be the length of the
solid,

immersed and h the height of the

shew that

k 2

dx
3 /

dx

Let
Tolution

j. Q be the solid of reand let A Bbe the level up to


P
is

which
Let
it

it

immersed
to the

in the liquid.

be divided by the plane

at right angles
paper.

Let

be

plane of the the axis of

101

symmetry and

Y
in

be JL to

OX
L

in the plane of the

paper and
the entire

Z
solid.

JL to

OX

the
in

plane

M. Consider

first

Since

it is

equilibrium

we have by

the principle of

buoyancy.

r
<?*
r
I

*y*dx--gP
<r

P
I

J''
where
is

J
the detiity of the solid

and P that of the

liquid.

'

-_? "
P

P
o^*
solid

(1)

Now
L M.
(i)

consider one half of the

divided by

<he

plane

The
the
viz

forces acting

ou

it

are:
its

weight of the half solid acting at

0. G.

G;
weight of the
the
fluid displaced

(ii)

the

by

this

half of the

solid acting at

point

7
,

the centroid of the fluid

displaced;
(i;i)

the pressure

p oa

the plane

acting at

its

centre

of pressure N; and
(iv)

the action of tha hinge at 0.


(i) is

Now
7T

evidently

f* \y* dx acting ^ J
.

a*;

G, where y the distance of

from the

OX is
fj'

Jo
IT

V\

f*
y''

102

Similarly

(ii)

is

- f* ~n\y dx
1

acting at G', where y' the distance of G'

from

X is
<*

(* Je

J
Evidently
~k
(iii)

IB

2p y dx acting at

where

f
I

2pyas

da
,

ONI

being ^

?py

Hence taking moments about

"k r*

(k 2
\

px UP* y d

x'

2pyd

103

Ck
or
f

Hx % y*
fk

9a

fh

Ck
I

>g
Clf

yVa-2

j(fc-)y
Ch
I

</

*__

or

ef*-f 3 9 y*

o *y(fc-a;)<te>P

* f

J'L

{.v'+Mfc-a

Ex.11.
centre fixed

hemisphere
is

is

just completely
its

and

kept at rest with

immersed with iti base vertical by means


or

of a couple Gr, determine for at the centre and the couple,

any values of P and


i

the reaction

Tripos

pt. 1,

1915).

Ans.

Horizontal component of Reaction


Vertical

= w a^gP
7r

,,=|

(a-P)

4, 2.

Body
body

floating in a heterogeneous liquid:


is

If the

floating under the influence of gravity alone

in a heterogeneous liquid the

resultant pressure
to

given by the principle on the floating body


the

positions of equilibrium are still For the of buoyancy proved in 4. 1.


is still

equal and

opposite

the

gap

weight of the liquid displaced by it the law of density mast be maintained


density

Only
i.

e.

in filling up the surfaces

of equal

muwt be continuous with those of tie surroun-

ding

liquid.
4. 3.

fluid

General Principle of Buoyancy: The resultant pressure en a solid bodv whollv or oartiallv immersed in a

<

104

fluid f

which

is

at rest

under the action of given

forces, is equal

and opposite to the resultant of the external forces which would For imagine the solid abolished and act on the displaced fluid.
place occupied by the fluid the action of
its

This

fluid

its

in equilibrium

under

(i)

external forces acting at


fluid.

each

point

f the tnolosed

(ii)

fluid pressures

acting
fluid.

normally on

the surface

(8)

enclosing the

Hence
(i).

the resultant of

(ii)
(ii)

is

Now
S

clearly the system


is

equal and opposite to that of is the same whether the bpace

inside

dary

It follows occupied by the solid or by the fluid. then that the resultant of the fluid pressures acting on the bounof the solid is equal and opposite to the resultant of the

external forces acting at each point of the displaced fluid. It should be noted that this resultant may not reduce to a single
force.

equilibrium under the action of the fluid pressures and external forces which act equally buth on the solid and the fluid, the resultant of the external forces acting ou

Hence

if

a solid

is iu

the solid must be equal and opposite to the resultant of the external forces acting on the displaced fluid.

homogeneous liquid of density P surrounds comThe solid and of any shap 3 whatever. pletely a solid mass M the liquid attract one another according to tde law of direct

Ex

1.

distance.

Prove that free surface of the liquid

is

a sphere.

small cylinder, which is also attracted by the solid according to the same law floats in the liquid. Show that it must and float with its axis oriented towards the centre of mass of

(9k*~ PA")

105

P,<r

being the densities of tin

liquid

and cylinder, h the height

axis

of tin cylinder/ V the length of the immersed and d the distanc of

the centre of the face nearest to the


centre of mass of the s
/ltd

from
is

it.

Since the law of fon;e

that

of direct distance, the resultant attractic n of the solid mass on rfny particle is in the direction of and proportional to the distance of

the

particle

from the centre of mass

of the solid.
is

Taking

as the origin, the equation of equilibrium

dr
so that

p^^M P

(/1

f
),~

where the radius of the

ft\e surface

where p^o

is

A.

Again the resultant mass


body.
(

attraction

of the mass Af on another

is

n AilH'O P,

P
is

being the centre of mass of the second


subject to

Now
i)

the cylinder

two

forces

the resultant pressure of the surrounding fluid, and


the resultant attraction of
(ii) is

(ii)

Force

where a

is

P, and acts along OP, evidently Jify (^ a?h <r) the radius and P is the mid point of its axis.
(i) is

Force

the force which

is

equal and opposite to the force


f

OQ, OQ, h being the lensrth of axis immersed in the liquid and along Q is the mid point of immersed portion of the axis, assuming
is,

acting on the
f

fluid displaced.

It

therefore, MJA (*a*h P)

that a is very small

compared with A.

In order that the cylinder be in equilibrium

Q and

must be in the same line so that P,Q,0 are collinear. As P,Q are two points on the axis, it is therefore oriented towards 0.
Also M,* -*a*h a

(OP)M|*

-"a^h' P

(OQ)

106

or

o h

+ \) ^

V(d +% 4
2

2rf( P/*'
Ex.
2.

cr4)-(crA

-~P

A'

2 ).

right circular cone floats with

its

vertex

downward
/,
1

in a rectangular tank

moving

horizontally

with

acceleration

Show

that the

axis of the cone

makes an anklet any with th^

/
horizontal.

Find also the nature of


Ex.
liquid,
3.

froe surface,

circular cylinder of radius

flo.ts

in

rotating

remaining Shew that h unimmorsed.

itself

at rest with its axis vertical


if

and a length
Jong,
it

the cylinder

is sufficiently

will float in equilibrium with its upper rim in the surface provided

the liquid

is

made

to rotate with

angular velocity

yh
a

(Triops
4.4.

Pt.T,

1920

Surfaced of Flotation.

body

In order to specify the origination in space of a floating and draw out lines take a point ot the body say its C. G.,

GA, GB, may be

in all directions tixed

in

the body*

These lines

called axes of orientation

orientation of the body

by saying that

We may then specify the GA for example is vertical


base.

and directed downwards towards the

of the body obtained by rotating purposes all positions must be regarded as equivalent vertical

For our present it round

GA

a plane (AB) which cuts off a volume of the liquid is equal to such that the weight of volume the body (W) so that W-#PV, P being the density of that of

Plane of

flotation

is

the liquid.

Volume
Section
is

is

called displacement.

The
area,

which

body by a plane of flotation is a plane called the the Area of Flotation. In all possible

of the

107

positions of equilibrium

of a given
is

given liquid the displacement/

body floating freely in a For if we orient the constant.

body in
plane

s ieh

way

of flotation

hat the axis of orientation JL to a particular is vertical, the plane of flotation itself being
t

made

to coincide with the surface of the liquid, then

by the

first

condition

of flotation

weight of the body must be equal to the

weight of
1

fluid displaced so that

V=
to

W = const
p

Hence without any reference


not be fulfilled
all

second condition (which


conditions

may

or

may

possible

of equilibrium are

exhausted by describing planes which cut off a constant volume


-p

'from

the body.
for

All

such

planes

are

possible

planes

of

flotation

we have

only to

make

the axis of orientation at

right

angles to this plane vertical an.I the intersecting plane coincident with the free surface of the liquid in order to make it

a plane of flotation.
cousiderthe envelopeof all the planes of flotation Such a surface is called the surface rutting off equal volume. of flotation and is characterised by the fact that every tangent

Let us

now

plane

IB

a possible plane of flotation.

Ex.

Prove

that,

in

all

cases

two and only two tangent


to a

planes can be drawn to a surface


plane,
4'5.

of flotation parallel

given

A Theorem

of Bouguer and Dupin'.

B with the point of contact of each plane of flotation envelope surface S is the centre of gravity of the corresponding area of flotation.
The

To prove
If

this

we
a

use the lemma:

we
to

consider

tangent plane to a
of this plane
is

surface, the point of contact

found

lie

on

the line of intersection of this

plane and any other tangent plane infinitely Thus the common point of the close to it.

108

>

lines of intersection

of this plane

with

neighbouring tangent

planes

is

the point of contact.


let

Now
at

AB

and A'B' ba two plants of flotation inclined


eaoh other.

an

and

infinitely close to

Let ns take a system


7 7

of rectangular axes, the line of intersection of AB and A B as OX,OYin the plane A B at right angles toOX, and Z normal to the plane A B. The equation of the plane A'B' is accordingly tan 8. Since the volumes displaced by the two planes are z=y
equal, the
equal.

volume enclosed in the angles

A',

and B
7

are

Let

dx dy be an element
let

of the

piano xy at a point
,

(x,y

be the height of the point P which is the intersection of the perpendicular from P on the plane A B, and the plane A'B
)

and

Thus

zy tan

8.
7

volume enclosed in the wedge B O B will be y\/* * dx dy, z being+ive and that in the wedge AOA' will be y*./*3 dx dy, z beiiig ive. Volumes in the two wedges

The

total

being equal numerically

integration being now flotation in the plane AB.


or,

extended over the whole urea of

y\/* y dx dy^o, as

z~y

tan 0;

or

y^o.
C. G. of the
of

Thus the
tersection
infinitely

area

AB
is

lies

on

OX

i,e.

on the

in-

of the plane
close to
it.

flotation

(A B) with any other plane


the
point

Thus

the C. G.

of contact of

the plane with the surface of flotation,

Centre of gravity, C, of the mass of the liquid displaced

is

The locus of C for various called the centre of buoyancy. of the plane of flotation is called the surface of positions
buoyancy.

109

Ex.
surface
flotation.

1.

Prove that the tangent plane at any point C to the


is

of buoyancy

parallel

to

the corresponding plane of

Ex.
right

2.

Prove th

it

so

long as no part

ot

th^ base of a

circular cone
is

floatsug in a

homogeneous

liquid with verttx

down-wards
hyperbloids

submerged, surfaces of flotation ai:d buoyancy are

of revolution.

Let V

I'

be the

cutting the cone of the cone. The


will

perpendicular from V on any plane (AB) and 6 be the angle between VP and the axis

plane
in

cat the cone

through VP and the axis of the cone two lines VA and VB which mterseet the

given plane in the points

A and B which

are the extremities of

the major axis of the ellipse,

Then

VA= Cos
VA

(6+a)
2

/'

V VB=

Cos(0 a)

If

is

the volume of the displaced liquid,


(area of the ellipse)

V=4 p

(See Kx. 8
$

41)

so that --------

-----------------

*=Cons.

or

VA. VB-Cons.

Since
line

AB,

the centre of the ellipse is th* mid-point of the the locus ot C will be a hyperbola with VA, as its

VB

varying porstions of A B. Hence for all possible positions of the cutting plane the locus of the centre of gravity, of the area of flotation is a hyperbloid generated by the revolution
asymptotes, for

no

)
It

of this

curve

about the axis of the cone.

follows then from

the theorem of Bouguer


flotation.

and Dupiri that

this is the surface

of

lies

be the centre of buoyancy in any position, then H such that V H -JVC; so tha the locus of on the line V

IfH

H,

surface of buoyancy,

is

a similar hyperbloid.

Ex.
ellipsoid

Find the surfaces of buoyancy and flotation for an and deduce from this those of an elliptic cylinder.
3.
4.

Kx.

Find the surfaces of

flotation

and of buoyancy in

the case of a cylinder of any cross section.

Let
base, the

be the
line

oentroid

of tin
1

base and OZ, normal to the

of centroids.

he oentroid of the cross section by

any plane

of flotation is the point

where

OZ

intersects

it.

Thus

all possible planes of flotation meet in one point, which is accordingly by the theorem Bouguer and Dupin, the surface of

flotation.
If Oil',

Oz Oz be taker
y

Ix -f wy-\-c, the coordinates <#, y, ? ) the cutting plane as z of the centre of buoyancy are given by

as rectangular axes

and the equation of

J'f & =
I

dx dy
'

field of integration

17

fJ^lxTy^ x + /" /* *
(

bei

the base.

m y +c) dx dy

W
:i

al -f~

Aw,

where

= ffx* dx dy h~ff xy dx dy, ff x dx dy ffy dx dy = o


.

Similarly

Vy=/V yz dx dy~ffy
=
(hi

Ix -f

my

dx dy

ftm),

where b

ffy*

dx dy

111

Also

Vz

= i ff z
(al*

dx dy

~4

2 h Im -f

Im

Also

Ac = V.
m
from these we have

Eliminating

Z,

A*
ThiiK
y
in the case of

= =F e 9 %

a solid box bounded by planes x =F</, & floating with the base 2 o wholly immersed,

we have
J* +f x* dx dy^;;

(4 dt\ <ffy*dx dy =^
y dx dy

yy
so that
the;

^
i.

surface of

buoyancy

is

2,

-_c

V
2

=
'

_8 a
(

^2

3 i2

or

-7T +
as

TL

W
c.

Prove that the curve of buoyancy in the case of a rectangular lamina is a parabola.
Kz'5.
4.6.

Metaceutre:

Let
surface

C
of

N, C'

W be normals to the
at

buoyancy

C and

C'.

Draw

(AH',

the shortest distance line


is

A
*,

between the two normals, then p


to the line of intersection

called the meta- centre corresponding

of the adja-

cent planes of flotation A B, A' B', -^-called the axis of inclination ceparallel to respectively perpendicular to planes
is

lt is

obvious that m*'

x for C N and A B and to A' B'.

C'

are

The metacentre

p-

is

situated

in general

between the two

and of the surface of buoyancy centres of principal curvature It coincides with one of the centres of (C) at the point C.
curvature

one

the axis of inclination Oa? is parallel to directions of the surface (C) at the point C. of the principal

or

when

This theorem

is

really

general

theorem applicable

to

curved surfaces and therefore also in particular to a surface of

buoyancy;

we

proceed to prove this theorem.


point C as origin, CN the normal at C as Z-axis, and in the direction of principal radii as axes of

Take the
and CX,
Y.

CY

equation of the surface in this system of axes usual notation may be written as

The

with the

2
y\
*

=4

r x*

-f"

y*

) -f-...,

for small values of


at

a?

and

being the principal radii of curvature

Thus

CM =
X
p

l
t

Cm-Y
q

1
.

The equation

of the

normal C'N'
v

is

Z
1

be the projection of pp' on XOY plane. Clearly since |i|A is normal to C|i, pp' will be parallel to CV and coordinate of |i is equal to Z coordinate of |A' CJA Le, the Z

Let

0V
/

\V
\JL\H'

Now
is

let

the

direction cosines of

nji'

be (L, M, Nj then since

perpendicular to

(A

N=0

and C'n/ and Lp

+Mq=o

Hence the direction cosines of CV are are (L, M, o) where =o. The equation of the plane CV n'n is accordingly

We
of intersection of

now

require

the z~~ coordinate of

n' f

the point

CV

with the plane

Any

point on the line being given by (a?-f-jA,

we have

113

80 that

Hence

V^=C^ =

substituting for

p and q from the equation

to the surface.
to zero in
is

Now
a

in the limit

when

C' tends to

C,cr, y,z

tend

such
the

way J

that the ratio

^ tends x

to the limit

tan a where a

angle made by the limiting direction of CO' with CX.

The
i.e

limiting value

oi CJA lies therefore,

between

and*'o

**o

lies
J

between
i+e.

CM

and Cm.

Til

the particular case

when
of

a^=oor
4?

when

the direction of CO'

coincides with one

the principal directions of the surface at C, Cji=CM or Cw. and m correspond to the maximum The centres of curvatures

and minimum of

Cj*

and may be called the

'major'

and 'minor'

meta-centres respectively relative to the point C.


4.

If 1 be the

moment

of inertia of the area of flotation


Off,

having an axis of intersection


centre
JA

about this
is

line,

the rneta-

corresponding to the axis

at

a distance I/V from

Oi.

e.

Fig-

(0

Fig.

(ii)

Figure

(i)

shows the

infinitely close

normals

ON

and C'

with nn', the shortest distance line between them. Draw e jm C'N' and let e be the projection of 0' on the plane parallel to containing CjiN and e JAW. Fig. (ii) represents the section of the

C |iw. Since is parallel to the axis body by the plane e^N and of intersection (See 4.5) and \JL\L' is noi mal to the plane conit follows that the axis of inter section taining CnIS and e \in,
1
T

is also

normal

to

this

plane (the plane of the section shown in

Fig.

ii>

Let Nn?*=0

Take
of a
axes,

as before for

axis a line perpendicular to

Ox

in the

and Qz as plane AB,


f

vertical.

Let

(;r,

?/,

z)

be the coordinates

point p we have then

f or q ol the plane of flotation A'B' relative to these

z~ y
Let dx

tan

0.

be an element of area of the plane of ilotation AB or 7, the moment of intertia about Ox is there fore, pliced at/? % 2 dx dy, integration being over the area 01 flotation, %/

dy

]=y
1st

y Let us apply to
case

the elements of the volume displaced in the limited by the horizontal plane AB fictitious forces in
all

the

vertically

element.

upward direction equal to the volume of the These forces then have a resultant equal to their

sum

viz,

V
CM.

acting

at

(heir

centre

of gravity

along the

direction

Similarly let us apply at each element of the volume displaced in the second case and limited by A'B' forces equal to the

volume of the element but acting


resultant is again a force

vertically

downwards*

The

acting at C' vertically downwards.


is

The
is

resultant of these fictitious forces

a couple whose

arm

The result C'D, being the distance of C' from the normal ON. may be calculated in another way as follows: The forces acting
the

at

common

elements of the two volumes cancel each other

116

equal and opposite; there romain therefor^ only the forces elements of volume in the wedges A A' and BB' applied to the

An
the

elementary cylinder
1st

pp

based on

dv dy and

of height

iu

by a vertically downwards force of magnitude zdx^y^ the volume of the elementery cylinder. The

wedge

is

acted

yzdxdy Similarly the volume of the cylinder <j(] in the second wedge is acted upon by a about Ox is yz dx dy. veitically upward force, whose moment The resultant of the forces in the two wedges being known to be a couple, the sum of the moments ot these forces about Ox is in a plane equal to the component of the moment of the couple normal to Ox. Thus y* dx dy^ V (projection of the arm a the of the couple,-C'D in Fig (i) on a plane normal to Ox
force about Oa;
'

moment of this

is

ff

i.

plane of the section in Fig.

(ii).

= V. eD.
oi

But for every given

#,

whether

pp

or ol jj', z

is

equal to

y (an 6

so that tan

ffy*
tan 9'

dx dy**V.e

D.

or, '

Also, front the

D
u

ey

whose angle

at n is 6,

we have

= D
tan 6

The

point

D
C

is infinitely close to 0, in

the limit, therefore,

Hence
4
8,

ii*=I/V.

Positions of equilibrium:

The

conditions

buoyancy are
(i)

of equilibrium according to tLa principle of v \


r

the volume
\

of the

immersed part

IB

equal to

W W p
,

being the weight of the body, and P the denbity of the The free surface is a tangent to the surface of liquid
flotation.

(ii)

The
is

line

GO must be
A B.
to

vertical

i.e.

normal

io tiie surface

of flotation
parallel

But as the surface of


the
(Ex,

flotation

AB

tangent plane to the surface of


1

buoyancy

at

4.5),

GO

is

also

normal

to the

The centre of buoyancy surface of buoyancy. the ponding to a position of equilibrium is therefore
foot of a

corres-

normal from the centre of gravity

to the

surface of buoyancy.

Conversely every normal

GO

dropped from

to the surface

of buoyancy .will correspond to a position of equilibrium obtained whose centre of gravity by immersing the solid up to a volume
is

the foot of the normal considered.

In this position

can bo

above or below G.
It

follows then that the orientation of the flaotingbody in the

position

of equilibrium

as

well

(stable, unstable or neutral) is

nature of equilibrium the same as those of a heavy body


as

the

bounded by the surface of buoyancy resting on a horizontal plane. From this theorem it is easy to deduce the necessary and sufficient
conditions of stability of equilibrium.

Equilbrium of a heavy solid on a horizontal surface. We shall employ the well-known theorem of Statics that when a heavy
4,9.

system
ol'

is in

equilibrium,

stable

possible

i.e.

equilibrium lower than

the necessary and sufficient condition is that its centre of gravity be as low as
it

when

is

in

any of the neighbouring


convex

positions.

Consider
at a all points

now a heavy

solid

bounded by a surface

and resting on a horizontal plane

O'

Let
fixed

its distance from the be the C. G. of the body, The positions of equilibrium are obtained by plane,

GH

is maximum or minimum determining the positions in which the positions in which GH is minimum being stable and those in

GH

which

it

is

maximum

being unstable.

(The segment

GH must

be regarded as positive or negative according as G is above or below the horizontal plane. The C.G. can be below the horizontal if it is external to the

body as in

fig. II.)

be the point of contact of the body with the horizontal plane, then the necessary and sufficient conditions of stable
equilibrium are:
(i)

Let

The
It

C.

must be on the normal

at

0;

(ii)

must be below the two centres of the principal curva-

ture of the surface relative to the point 0.

The

first

condition

is

evident, for if there

is

equilibrium

it

is

necessary that the weight of the body acting at


the reaction of the plane acting at 0. fore must pass through G, so that

must balance
at

The

reaction

there-

GO

mubt be the common

normal

to the

plane and the surface


condition

at 0.

The
stable
it is

first

being

fulfilled;

for

necessary and

sufficient

that

in

equilibrium to be the position of the

body considered, the distance of


lees

from the horizontal plane be


near to
this.

than in

all

other positions

infinitely

Instead

of leaving the tangent plane fixed and displacing the body it is more convenient to regard the body as fixed in its equilibrium the tangent plane. It is necessary and suffiposition and move
cient for equilibrium to be stable that the distance

GO of G
G

from

the

tangent plane at

is less

than the distance of

from other

tangent planes at points 0' infintely close to 0.


as origin, the tangent plane at the 'principal directions of the surface at

Take

zoy plane, and as axes of a? and y,


as

and

let

Oz

be on the same side as the surface.

MM.)
Let
;

(x,y>%)

be any point close to O.

Let the equation of the surface be

.^
*%>

Qm=
t

'being the principal radii of curvature,


at 0' is

The equation of the tangent plane

The

distance of

(0,0,

from the plane

is

=< ~ z+px
the point O,/? 5 ===#==*=<>, BO that at 0' a Hence point infinitely near 0, these quantities are very smalll. the expression for G H has the same sign as S.
at

Now

Hsubstituting
surface.
ior

p,q

and

^:

from the equation of the

2 Q Expanding in series of a: and y the expression on we have up to terms of the 2nd order

the R.H.8,

G H =*- tf(r B V-Ko V)+ = H4r -5)


3
(

For the value of

G H
and

to be

minimum
2

it

is

necessary

and

sufficient that the coefficients of

and y be

positive.

or,

C<
o

^<

Hence

must be below the *minor meta-ceritre, ** * I: In this we can Remark


indiffejj$btly
Y&sadjjfc

^posetW

point

to be

iuterior

to

the

surface,

bein^had tbJIe

proper

sign of Gr H,

119

.Remark

II:

The necessary and

sufficient
is

condition
that

that

the equilibi ium* of a floating body be stable below TW, the minor metia^centre.
.

must be

We

have already sean that the distance

Cm = -?

where

= the

minimum moment

of inertia of the
its

plane of flotation with respect to an centr\

axis passing through

Henre

for stability of equilibrium

CG Z J
necessary and sufficient condition of stability of equilibrium by another method.
4.

10,

We
it

shall

now prove

the

has b^en pointed out that when the direction of CC' coincides with one of the principal directions of the surface
In
4.

of

buoyancy

at

C Cn=OM
5

or

m.
to

In thin case the normal

at C'

the

surface

interects

the

normal jt

and
the

n, p

coincide
in

The

axis of intersection
to

with the point of intersection. the area of flotation is a line

paraPel

line

and C so that the axis of intersection

of intersection of the tangent planes at C ita line through the centre

of gravity of the area of flotation parallel to one of the principal There are accordirections of the surface of buoyancy at C.

dingly two such axes of intersection corresponding to the two


principal
directions
of the
inertia

surface

of

buoyancy

at C.

If I

Ia

be the

moments of
1-6

of the area of flotation about these


2'

axes then by
Since

OM^^C w~y
Cm
are the

CM
If

and

maximum and minimum


of
inertia

value of
line

GH(*=^>

being the moment


airea

about any
it

through the centre of the

of flotation,

follows that I f

220

are the greatest and least moments of interia of the area of flotation. Consequently the corresponding
I

I 17 so that I]

and

Consider now a rotation of axes are principal axes of inertia. the body about any one of these two axes through a small angle
6.

In the displaced

position,

a vertical force

W=(^PV)

acts

C'M, which is the vertical in the displaced the fluid pressure gPV acts vertically upwards (ic. position and along C'M) through the new centre of buoyancy C'. Hence
through
parallel to

assuming
is

G
6

to be

G M Sin

below M, the moment of the restoring couple (</PV) and is about the same axis about which

the body

is rotated.

,/PV.

GM

Sinfl=flrPV

(CM ~CG)0,

being small,

on the other hand the body be rotated through a small angle 9' about the second principal axis of inertia of the area of
If
flotation the

moment
(!

ot

the

restoring

couple about the axis of

rotation is
If the

yP

-Vh)6'.
is

rotated through a small angle about any axis through the C. G. of the area of flotation, it may be resolved into two small rotations 0, 8' about the principal axes of inertia.

body

ihe

restoring
9

couples set

up,

therefore,

will

have moments

In order that the equilibrium be stable


these

ii

is

necessary that
(J

moments be
if

that A positive, so

CG<^I,

and A

<*?.

Conversely

G<

(where

<I

)the two couples

have positive moments and tend brium position.


EJC. 1.

to restore the

body

to its equili-

with

its

cylinder of radius a and length ft floats axis vertical. Find the condition of stable equilibrium.
solid

If the length immersecl

is &',

we have

00 - JI, G = 1, so that CG= A


2?

~A
w

'

For

stability
(*-a*)

a*)
*
-

p a^ CG<
or

V~ llA' < <' 2 ^


water with

a*

-*>

-4*'

Ex.2.

base

is

uniform right circular cone of specific gravity a an ellipse of semi axes a, b (a>b) and whose height
f

is h

floats freely in

its

axis vertical

and vertex downwards


that

If the equilibrium is completely stable,

shew

If h be the height

immersed, then by the principle of buoyancy

^..A-f-jPA'-l-^^A)
or f Aa*-=A'
(1)
flotation.
7 /
1 1 /

being the area of the base and A' of the plane of


of flotation is

The plane
hence

an

ellipse

whose axes are

(^~
/*
ii

T 1 , the

moment

of inertia about major axis is

Tt r*

75 (

n~

Also

G = |A and OC=|A',C

being the centre of buoyanoj

For stability

CG = \(h - h')<

-122

or

Ex.3. If a solid cone of density a floats in a liquid of density how that the equilibrium is stable if

V
V
/r

~
a
p ~

vertex

downwards

'in

r>

'A

Pcr

^
'

'-i

'

vertex upwards,
ccne.

r ? A being the radius

and height of the

thin rectangular strip of uniform material having Bides of length I and ^a is bent into a channel of semi circular

Ex.

4.

section (radius

a and length

with open ends and immersed in


is J.

water.

The

specific density

of the material

Confining attention to positions in which the straight edges of the channel are horizontal show that there are two configurations

Are there any of stable equilibrium. unstable equilibrium ? (Tripos pt 1 1927)


Ex.
5.

configurations of

The

water level IB 20 ft,


cubic
ft.

radius of gyration of the section of a ship at the and the volume of water displaced is 24

where

is

the area of the water

line.

Assuming the

curve of buoyancy to be a parabola show that if the C. G. of the ship is !" above the meta-centre, the ship will heel over through about 5 into a position of stable of equilibrium.

(Only rolling displacements are

to

be considered)

123

Let the centre of gravity G be on the axis of the parabola ef buoyancy and let m be the corresponding meta-centre.

Then

Alt' _ -_- _

A(20)

._

_ 50 _

ft .

Also since
vertex

Gm

-= radius of
its

2a,

2a being

curvature of ths parabola at the semi latus rectum.

Thus

2a = ^ft. u

Other positions of equilibrium are obtained by normals from G to the parabola,


If
|i

drawing

be the blope of a normal,

its

equation

is

Since

it

passes through

^ + r~ o 16

we have
/

50

1 x

The

solution

|i

o corresponds to the position in

which the

axis is vertical.

The
brium.

other two solutions give the oblique positions of equili-

We

have then

^^50
3

"*"

J.
16

__50
3

""

J_
16
1/

+ 1 = T_4 Va
= -f.0866

V3

124

the angle between the normal through axis of the parabola is 0, tan 6 ~. 0866
if

Thus

and

th<*

or 9

=5

appro*.

Thus
position in

the ship be rolled through 5 on either side of the which is vertical, the oblique positions will be
if

Since the intermediate position positions cf equilibrium. unstable equilibrium (G being above the metacentre* these
tions will be stable as positions of stable

is ot

posi-

aud unstable equilibrium

occur alternately.

beam of length I and density or has a square section ABOD of side a and floats in a liquid of density P(>cr> with the side I horizontal aud A B above the surface of the
Kx.
6.

liquid.

Prove that a beam of the same dimensions but of density


P
cr

will

float

in

the inverted position

with the same piano

section in the free surface.

If

:j~(l

p)

<^

shew
it

that the only position of equilibrium

is horizontal

and that

is stable for

small displacements about


(I.

the length

I.

0.8.

1934)

angle 50 and specific gravity J floats with vertex downwards in water with the axis vertical. The vertex is loaded with a particle whose
7.

Ex.

uniform solid cone of semi

vertical

mass
is

is

n times
if

the mass of the cone.

Show

that the equilibrium

neutral

(L C.S. 1981)
iHint
;

Here

CG=CM.)

Prove that an isosceles triangular prism or wedga with its base not immersed aud its edges horizontal ha* floating three or only one positions of equilibrium,
.

Ex.

8.

125
the

In the base horzontal


case
it is

former

position with in the latter is unstable, while

casa

stable.

edges d be any plane of flotation. Taking oblique axes AB and AD as axes of X and Y we fiind that the coordinates of the
horizontal

Let

the

wedge

float

with

its

and

let b

centre of gravity of the triangle

d are

X
(

Y
a

where

Since the weight of the wedge ABD is equal to the weight of the volume of fluid displaced by it, we have

(AD) Sin
I

2a.

0<rf

= 4XY

Sin

being the length of the horizontal edge.

/.XY=
The
locus

p*p

where

AB = AD

=jp.

of the centre of gravity

of the triangle

Abd

in

therefore the hyperbola

VY A l ~~ ~P

**

"

'

(\\ 1
I

'

j|~p"

Clearly AB, AD are the asymptotes of this hyperbola. Th surface of buoyancy is therefore a hyperbolic cylinder whoss

asymptotic planes are the sides of the wedge.

The
from
angle

positions of equilibrium are given


to this

by drawing normal*

G
Now

surface.

Since

is

on AE, the bisector of tha

BAG, G is a
it

p^int on the major axis of the hyperbola.

ptoved that from a point G on the axis of a hyperbola, either three normals can bo drawn or only on*
is

easily

according as

A G
axes.

a, b

being the major and minor

126

2a be the angle between the asymptotes of th kyperbola, we have


Also
if

tan

a
-

=
a

and

-4

^^TQyp
is

since the equation of the hyper-

bola with asymptotes as axes

XY~

-=-

by

(1

Thus there are three


acecording as
or

positions of equilibrium or only

one

G = -pcos a
as,

^0(1 -f tan*a)

p \l^cos a sefa,

according

/?

there

is

only OK e position

of equilibrium

which

is

the position with


of the

its

base horizontal,

One
i

the

of the surface of buoyancy principal sections section by the vertical plane perpendicular to the edges of
t

2nd being normal to this. If K be the vertex of the hyperbola, K M, the height of the minor meta-centre,
the

wedge

the

(radius of curvature of the hyperbola at


~

its

vertex

K)

P\!b
Hence

COS

a tan * a

M^A K+K M - ^p
1

co5
-yj?

a (l+tan*a)

Q JP

Ap

5ec

The necessary and


een,
is

sufficient condition of stability, as

we have

that

G be below M so that AG < AM


or,

127

Or, cos* a.

<\ I?
Vp

which

is also

the condition that there

is

only one position of equilibrium.

Thus
also stable.

if

there

is

only one position of equilibrium the position


her hand
if cos
2

is

On

the

ol

>

/?

there

are three

positions of equilibrium, but


position, is unstable.

the intermediate one viz. horizontal


other two must therefore be stable in

The

accordance with the well-known principle that positions of stable and unstable equilibrium occur alternately.

when a sega liquid, the distance between the centre of buoyancy and the metacentre may be constant, whatever be the height of the segment immersed.
Ex,
9.

Find a

solid

of revolution such that


in

ment, of

it

is

immersed

(Tripos part

19*9)

of the coordinates at its lowest point, the Take the origin, x being vertically upwards and that of y horizontal. Then are the coordinates which determine the surface of flotaif (x,y) tion in the axially vertical position and (a/,y') those belonging
axis of
to another parallel plane,

we have
IT

at

once

/X
o

*dt&

.%

it

CM

is

constant and equal to

whatever

x may

be,

we have
1

4
~* V JB y-

fit

x 9 m | rf flr f y WtAs

128

y
Differentiating

*
~

= my

The generating curve


of revolution
is

is,

therefore, a parabola,

and the

solid

the paraboloid generated by the revolution of the


x.

parabola about

Ex.
above

10.

Find the
it

solid

of revolution

such that

when

segment of
its

is

immersed
is

in a liquid, the height of meta-centre

lowest point

independent of the segment immersed.


the revolution of a circle about

(Ana Anchor-ring, formed by


a tangent).

Ex.11

If the height in the above

example were a function

(x, y)j x,y being the coordinates characterising the plane of flotation, show that the differential equation of the generating

curve

is

'

i
.

#0 L
do;

dij

d$
.

dx dy

cross section of a cylinderical ship is two equal arcs of equal parabolas of latus rectum 7, which touch at the keel, the common vertex of the two parabolas so that the sides of the

Ex.

12.

The

ship are concave to the water. The ship is floating upright with Prove that the height of the metacentre its keel at a depth Ji.

above the keel

is

] ( t

A*

Ex.
4

13.

solid

of uniform

density a

floats partly

imsolid

a) inverted with the same plane of flotation in the same that if the equilibrium is stable in one case it is also liquid; and stable in the other case for like displacements.

mersed in a homogeneous liquid of density P. Show that a of the same size and shape and of uniform density (P
float

can

129

4. 11.

Equilibrium of a vessel containing a liquid:

Suppose a vessel containing a given volume of


liquid of density

floats in a liquid of density 9'.

If the vessel receives a small angular displacement there will be a force of buoyancy due to the exter-

nal fluid acting upwards through its metacentre M; the line of action of the contained liquid acts along

the vertical through


intersects the line
liquid.

its

now centre

of gravity and

it

GM

in M,, the meta-centre


(It is

of the contained
is

This force acts downwards.

assumed that the body

symmetrical about the plane of displacement through its centre of mass and that the centres of mass of the body and of the liquid are in the same vertical line). moments about G, the Taking
resultant fluid pressures will tend to restore equilibrium or reverse it according as

W.
r'

GM- W'GM^
W'

<

GST*'

Consider a cylinder containing water floating in a liquid of specific gravity iv with its axis vertical.
1.

Ex.

Let k
//,

= height of the cylinder,


liquid,

the height of the contained water.

Let d be the depth of the cylinder in the given


a be the radius of the cylinder.

and

Than W=-*a*gh* P

W = ira*gd 9w
Again

GM.=OM-OG=OC + CM-OG
_
d
,

a f /4

GM X

130

* 2
'.

'

h'

For stable equilibrium

GM

'

2(*+a-2rfA

W ^ dm

h'

~h'

Establish a similar relation lor a right circular cons floating with its aiis vertical and containing some liquid.

Ex,

2.

\ Ana

( z

8 \

(^

\ >

^a
2

87*
,

j^-^

where A = the length of the axis of the cone,


A'

=
fluid,

in the contained liquid,

= tlu
and
Ex*
3.
tt

length beneath the surface of the external

= the

semi vertical angle.

parafcoloidal cup, the

weight of which

is

u\ standing

on a horizontal table, contains a quantity of water, whose weight If h be the height of C. G. of the cup and the contained is n w.
water,

the
is

equilibrium

is

stable

if

the

latus

rectum of

the

parabola

>

(n+1)

*.

Let the equation of the parabola generating the paraboloid x and tangent at the vertex J eing ybe y* ==4 a, axis being
axis.

Let z be the height of the water in the cup, then


the C.G. of the contained water,

0,

being

Lot the height


then k

ol C.Gr.

of the cup above

be

It

= -kw + nw <**) = AJ_?L1.


mer-a-c-putre

Height of

above C.

V
Thus
taiut;d
/.

s
".

W
weight of the conat

iu the subsequent displacement the

water must be imagined to act at


il the

M
ie

i.e.

height

+ 2a.

metaoentre of the system

given by
-

\\ ^tl yy \*

_ '*
-

+ -----+ 2a _
?*#
(
(

??

) -------4* 1)

fl

A!BO the radius of curvature E, of the paraboloid


vertex
== 2a.

at

the

Hence from a well known


equilibrium

result in Statics

we have

for btable

JL OM
~

'

=
>

J_
*a

^H

or>

'

whioh leade

to

4 a

>

2 (n

1)

Kx.

4.

oi

rough sphere with the centre of its base in contact with the sphere. Find the condition of stability for infinitesimal

neglected, a fixed

cylinderical vessel, the weight of which may be contains water, and the vessel is placed on the vertex

Prove also that if the equilibrium be neutral for displacements. such displacements, it is really unstable.
Ex. 6. A hemispherical shell, containing liquid is placed on the vertex of a fixed rough sphere of twice its diameter; prove that the equilibrium will be stable or unstable according as the weight

132
less

of the shell
liquid.

is

greater or

than twice

tbe weight of the

Ex

6.

thin vessel in the form of a surface of revolution

contains a given quantity (volume V) of homogeneous liquid of It rests wilh its vertex at the highest point w. specific weight

of a rough curved surface, prove that

the

equilibrium

is

BtM*

or unstable for lateral displacements according as

>
where

or

<(

(Vtok

Ito

WA)

is

the weight

of the vessel

without

the liquid,

Iho

moment

of inertia of the free surface of the

liquid

and

//,

k the

heights of the centres of gravity above tho vertex of ihe \eesel and the liquid respectively, R,R' being the radii of curvature of thr
vessel

and the fixed surface in the plane cf the displacement.


Stability of a
float

4. 12.

body floating

in

two

liquids,

Let a solid

in

two

liquids of densities P

and P

P',

Let V be the volume of the the former being above the latter. solid immersed and only that part of the volume which is

immersed in the second flotation of the two liquids.

liquid only.

Let A, A' be the areas of

to be immersed evidently suppose the voluiub to be immersed in a liquid .of in a liquid of density P and density P'. The forces which support the weight of the body are

We may

g9

and (#P'V) acting upwards at H and supposed gravity of the volumes V and

H
to

the

centres of

be homogeneously
consider only

filled.

Let

G be the

C.G. of the body.

We

shall

the simple case in which the body is symmetrical with respect to a vertical plane perpendicular to the plane of displacem nt so that G,H,H' are in the same vertical line.

Let the body be displaced through


axis in the plane of symmetry, the coupje will be

moment

about any horizontal of the restorative

133

<7p (i l

_ v. HG) + 0P'

(I,

V'.

H'G)

0,

being the moments *of inertia of the areas of axis through the centres of gravity of these areas.
TT
,

TJJ

flotation about

For

stability of equilibrium

P
Kx.
1.

(I z

V.

HG) >

9'

(V H'G

Ta

uniform liquid of density a a overlies ai. other of greater density cr, and a body with a plane of symmetry floats with its plane vertical so as to be in contact with both liquids.

Prove that the meta- centric height from the bottom of the body

is

___
where \\
is

"V, (<*,-,)

+ ,,
the lower liquid, Zj the cf this volume above

the volume submerged in height of the centre of buoyancy the lowest point of the body, A x
,

the area

of gyration of the lower "water line"; and


1 ,

Va

is

and radius the whole volume

below the upper "water line z a is the height of the centre of buoyancy which this volume would have if it were submerged in
a single liquid, and

A a> K 2

refer to

upper water

line.

(Tripos Ft. 11,1921)


4. 13.

Stability of a

body floating

in a heterogeneous liquid.
floats in

It

should be noted ihat

when a body

a heterognee-

ou3

liquid,

which

is

subject to the action of gravity, horizontal

planes are surfaces of equal density.

a heterogeneous liquid arranged in horizontal layers of equal density viz. the topmost layer of density P lf next of P^Pjj, next of Pi+Pt+Pgj etc i^et the volume of the solid

Consider

now

cut off

by these layers be r x v 2 v$ .. etc. and let H 1 H 2 H 3 ... be the centres of gravi ty of these volumes when homogeneously A 3 ... and k\ & 2 & 8 ... be the areas and radii filled. Let A lf A a
, ,
,

of gyration of the sections of the


density.

body by the layers of equal

134

by the immersion of the body in such a system are the same if the volume v^ of the body were immersed in a homogeneous liquid of density P 3 v 9 in that of

The

forces

called

into play

density P 2 v s in that of density P,


,

etc.

the body be displaced through a small angle 9 about any horizontal axis in the plane of symmetry (assuming thai it had one), the moment about G, the 0. G. of the body, of forces
if

Now

tending to restore equilibrium are

gPi(AkS-v
,'.

UiG),j/P a

(A

* a f -u a

H a G)

,...

Total

moment
\

act at

lt

H^,

......

HG = SP m
1

vm

Hm

G,

where

11 is

the centroid of the liquids displaced by the body.


vm

Hm

= (IIG) M

o:

Mo

being the mass of the liquids displaced by the body.


total

Hence the

moment

of the couple

is

The case of a heterogeneous liquid whose deiibity varies continuously is immediately deduced from this by dividing the
liquid into

an

infinite

member

of infinitely thin layers.

Let

the

functional

relation

between P

depth

bebw

the

horizontal surface

any point and its of flotation be given by


at

135

Considering two consecutive layers of density P x

+ P*4-

P 2 -K^Pr at depths * and * and P..J, and P! that Pr, thedifference between their densities,

+cfe we see

/(* + <!*)-/

( z)

Hence

in the limit the expression

where A

is

the depth of the lower most layer.


is

Moment

of the restoring couple

For

stability of equilibrium,

we must have

M HG

Ol

UG<
height,

Mo
HM =
j

.'.

Metaoentric

P, Atk,

f*Afc

Remark

P.Ajfc,

136

~if

Prf

the solid does not have a flat bottom.


for solids without a flat
,

Hence

bottom,

metacentric height

MS=

^r=

Prf(A&*)

the limits o and k refer to the top and

and bottom
Ex.
1.

sections.

Show

that in order that the equilibrium of a right

circular cone

density % is
Alt

of semi vertical angle a floating in a liquid whose 2 directly proportional to the depth, be stable cos a

<

--7-

on

where

K is the length of the axis immersed

and h

is

the

height of the cone.

Here
section

by horizontal planes are distant z from the vertex. by, a plane


sections

circles.

Consider a

Then.

The

required condition of stable equilibrium

is

HG <
("a
2

tan'a; dz

tan

--A'

tan2

since

P= H

( A'

-* )

137

f* a AlsoHG=f A-4r, h
I ./

Ps tan a) ---- -----dz


(

* z* tan 8 a

P</2

Hence the required condition

is

V 4

'

'*< "^ 5

= F ( L-Man'aH' o

*'

See

or, cos

^a

<--condition

of stability of a solid homogeneous cylinder of radius a und:r the same circumstances.

Ex.

2.

Determine the

[Ans. a

,^

/i

(/fc

r | A )j.

immersed heavy homogeneous cube is completely times the with two faces horizontal in a fluid whose density=&
Ex.
3."

cube of the depth. Prove that the meta-centric height

is

fr

where

M is the mass and


4.

a the length of an edge of the cube.

Find the metacentric height in the case of a solid of a revolution formed by the revolution of a curve y () about x with vertex downwards in a liquid whose axis,
Ex,

=/

floating

density at f is h .

depth z

is

z \

The length

of the axis immerbed

f *0(.'-*)/Ve/
!

/It! o
is

Ifthe meta-centric height iu the above example constant whatever be the law of density of the liquid show

Ex.5.

that the solid is a paraboloid of revolution.

Ex.
vertical

6.

and

axis ils floats with paraboloid of revolution vertex downwards in a liquid whose density varies

138

as the depth; the equilibrium will be stable or unstable according or 3 (4 a -f- A'), where h is the length of the axis, as 4 h

<

>

f 4 a the latns rectum of tho generating parabola, and h the length of the axis immersed.

Ex.

7.

ATI oblate spheroid floats half

immersed with

its

axis

rertical, in a liquid

depth.
,.
.

Prove that the height of the


is

whose density varies as the square of the meta-centre above the


.

surface

5 -

a
-

--

-/. 2

(Apply Ex.

4.)

Ei.

1.

A
is

uniform right circular cone of weight


vertical

float*

with
liquid

its

axis

and vertex downwards in a homogeneous

contained in a circular cylinder of radius r witl; ^ r2 h vertical axis; the con in equilibrium displaces a volume of the liquid and its vertex is at a depth // below ma surface

which

The cone

is

slowiy

lifted just clear of the


tli.tt

liquid,

its

axis

being

kept vertical; prove


{

work done
- 2 h').

is

(3

It

(M. A. Punjab Univ. 19SB


be the weight of a body and V the volume o the .part immersed in a liquid contained in a fixed vase. Prove tha S, th< it'the body be lowered vertically a small distance

Kx.

2.

If

floating

centre of gravity ol the liquid contained in the vase rises by

th<

quantity

W\ being
Kx.
3.

the weight oi the contained liquid and P

iti

Deduce by means of the above result the principl of Archimedes from the theorem that the height of the oommoi o i^ciiXj of gravity of the solid and liquid must-be a maximum

minimum.

139

Ex.

4.

life

belt

iu the

by a

circle of radius

a floats in
* the

form of an anchor- ring generated water with its equatorial plane


is

horizontal;

shew
z

that,

depth immersed,

given by

a^l

cos a)

2
5

s=

(2 a

sin 2 a),
of

being the specific gravity


5.

the material of the belt.

Ex

An

isosceles triangular

lamina

ABC
-

right angled at

floats with its

liquid of

plane vertical and the angle which the density veriea as depth.

immersed

in a
if

Prove that
in

-{- 9 be the angle

which

AB

makes
is

\*ith the vertical

either
is

of the positions in which

AB

not horizontal, the value of 9

given by

sin'Qcos**
(I.

= (sin 0+cosB)*.
C.
iS.

Higher Mathematics 1933)

body consisting of a hemisphere of radius a with il,H plane face attached to one end of a circular cylinder of radius a and length a iloaln iu equilibrium in
6.

Ex.

homogeneous

solid

a homogeneous liquid. The axis of symmetry of the body is vertical and the cylindericul portion is uppermost. The length
of the cylinderical portion immersed is '% a. of the metarentre and determine whether
Htahle or unstable

Find the position


the equilibrium
1

is

(Tripos Part
'

1938)

(Ans.
L \f\'

from the centre of the hemisphere.)


if

Ex.
with
its

the equilibrium of a solid cone floating vertex downwards and axis vertical is neutral for an
7.
it;

Prove that

infinitesimal displacement,

is

really stable f jr

any

finite displa-

cement

CHAPTER
Gases.
5.
1.

V.

a gas is its in practice nearly) incompressible is called a liquid. A perfect shall now fluid which is compressible is called a gas.

have seen that what distinguished a liquid from A perfect fluid which is absolutely cdhipressibility.

We

We

describe

some of the more important properties of compressible

fluids or gases.
f>

2.

Turricelli's

Experiment.

Take

glass lube about 3

open at one end and closed at the other. Fill it with mercury and invert it in a vessel of mercury so as to immerse its open eiid. It will be noticed that mercury in the tube will fall ]fA be any until the height of the column ib about 30 inches. point on the level surface of the mercury in the vessel and Q be
ft.

long

a point in
tube,

the

same

level

inside
at

tin*

we know
pressure

that pressures

A and

Q
is

must be equal.
the
of

Now

pressure at

the pressure at Q is mercury of height PQ, i. a ya PQ^ob 'ing the density of mercury, wince the
pressure at

atmosphere TT and that of a column of

P
TT

is zero.

Hence

~
first

This simple apparatus was

devised by

Torrieelli

and

is

known

as barometer.

Clearly

we may take

the height

PQ

of the

column of mercury as a measure of atmospheric pressure. We may also use any other liquid such as water. Since the density of mercury is about 13. 5 limes that of water, the height of a
water barometer
Ex.
will

be about

13^

30 inches,
to

33 J

ft.

Prove that the barometeric heights

which liquids

rise are inversely proportional to their densities.

141

5.S

Laws
^

of Boyle

and Dalton
If

Consider a given mass of gas enclosed in a volume.


neglect the
-enclosure is

we

weight of the gas


given by
</p

its

pressure at any point of the

=a

It is, therefore,

the
at
IH

same

at all points

and

is

ever value

it

has

the surface.
said to be at
to.

Whenever
pressure
jp f

equal to whata given raabs of


this constant

gas

in

an enclosure
is

it is

pressure that

referred

Now
or dilation

the volume of
i.

the

gas

may

be altered by compression

change of prtssure, keeping temperature constant, or by a change of temperature keeping pressure constant, or by a change of pressure and temperature bimultaneonsly. The law governing the change of volume and pressure when temperature
e.

remains constant

was

firs!

discoverd

by Robert Boyle.

He

showed that the pressure of the gas varies inversely as the volume of the enclosure provided the temperature remains
Constant.

Thus jjv~ const.


Since density P ol of the gas varies inversely as the volume, follows that pressure varies directly as the density so that

it

1^= const.
change of volume consequent upon a change of temparature, (pressure remaining constant \ was

The law

relating to the

discovered independently by Charles, Dalton

atui

Gay

Lmssac

They found
u

that:

If pressure
in a

remain constant, an increase of temperature of

1*0 produces
at g<7."

mass of gas an expansion

of

its

volume

be the volume of the gas volume at t0, then


let

Thus

at

oC

and

let

v t be its

142

(27340

pressure

p remaining constant

Similarly,

,,'

^.(

80

If

we

define 'absolute temperature of the gas

by the
-

relation
(1)

T = 273+,
it-

...

..

the

pressure remain constant, the volume of gas varies directly as its absolute temperature,
follows
that
if
i.

e.

acconst.

We

shall

now deduce

the law

of change

of volume,

when

both temperature

pressure, volume*, constant and change pressure to //, so keep

and pressure vary. Let (p, ?;, T) be the and 'abtolute temperature' of the gas. Lai us
that by

Boyle's

law, the volume changes to

^~
P
temperature

If

we now
law
~

alter

the

to

T' we have by

Da

ton's

ss.
'

&J-1

>

"f

y
f

t"

where

v is the

now

volume.

The equation

pv 't

= const.
.

143

satisfied
is

by the pressure, temperature, and volume of the


state.

known

as its equation of

Experiment, however, shows that not all gases obey this equation accurately and even those which do, do so only approximately

ranges of temperature and pressure, beyond which the relation is no longer satisfied. A gas which obeys
for certain

the equation of state


-PL'.
ss.

exist

is

known
for
is

as an 'ideal' or 'perfect' gas.

Various equations of

state

imperfect' gases have been formulated but the best


that given

known

by Van der Waal

T (P+
where
a,
/>

*j

)(*-

4)*= Mnrt,

are constants for the

same mass of
its

gas,

but depend

on die amount of gas as well as on


5
4,

nature.

Nature

oi

Gas

Presbure.

of this pressure is best understood on the basis of Kinetic Theory of Gases. According to this theory the molecules of a gas are in a state of incessant motion describing
rectilinear paths
ter
'

The nature

other

with uniform velocity except when they encounthe of molecules or walls the enclosure-

I he pressure of the gas on a email element of a wall of the enclosure is due to the continual impacts of the molecules on it

any other point of the volume is the result of molecular impacts on a small area imagined to be placed at With the help of certain appropriate assumptions it the point
at

So also the pressure

can be shown that when the gas is in a steady state, the pressure due to molecular impacts on any element of the wall of the
enclosure or on any plane element imagined elsewhere
IB

the same,

being equal to

144

where

is

the

number
c
2
,

m
v

the mass of a molecule,

of molecules of the gas in the enclosure, the mean square velocity i e the

mean
is

value of

where

c is

the volume of the enclosure.

the velocity of a molecule, am! Now according to the funda-

mental hypothesis of Kinetic Theory., the temperature of the gas depends only on its mean square velocity, HO that if temperature

remains constant, so would <*


remains constant,
its

Hence

if

temperature of the gas

pressure varies inversely as the volume. Thus Kinetic Theory gives a simple explanation of Boyle's Law. Indeed Kinetic Theory postulates the relation
i

RT,

......
same

(1)
for all gases,

where

is

a universal gas constant, the

and

is its

absolute temperature.

We

have

therefore,

NRT p _

__
eon*
..
..

It

will be seen that

we have

tacitly

assumed the

identity cf

'absolute temperature' as defined by equation (l) ot 5.3, with the absolute temperature' of the Kinetic Theory as defined by
'

relation (1) of this article.

This identification

is

justifi

d in works

on Kinetic Theory of Gases.


In establishing the formula quoted above for the pressure exerted on the walls of an enclosure Kinetic Theory assumes the molecules
It

be spherical points devoid ol size and .shape. further assumes that the forces of cohesion between the
to*

molecules are non-existent.

In

any

real

molecules must have size and shape. Hence it comes about that the cohesion be entirely negligible.

however, the Nor will the forces of


gas,

equation of state (2) which is accurately satisfied in the case of an ideal gas will only be approximately satisfied when applied
to

a real gas.

It

can be shown that the equation of state, which

145

takes into account the site


forces

of the molecules and the cohesive

between them

is

Van der Waal's Equation

where
ii

#,?>

are constants ior the

nature and amount, R is tho number of molecules of the gas in the enclosure.
5. 5.

same mass of gas, but depend on a universal gas constant, and N is

Mixture of

Idt al

Gasea.

If two enclosures containing two gases at the same pressure and temperature art collected together, the molecules of one gas diffuse into those of the other until they are completely mixed

provided there

is

no

chemical
is

action

between

the

gases.

The volume
enclosures,
its

of the mixture

the

sum

of the volumes of the

pressure and temperature bp>ing the sarae as those of


It is

the constituent gases fact that


"If

easy to deduce trom this experimental

2 of different ideal gases at pressures absolute temperatures TI, 2 are mixed together, the pijp*and volume V, the pressure p, and absolute temperature T, of the

two volume* V\,

mixture are connected by the relation.

If the

temeprature

changed

to

and

and pressure P! of the first, gas be volume will become respectively, its

TI

Ti P
similarly the

and pressure p

volume of the second gas will be

at

temperature

P.V.

T
P

T,
Now
if

two gases of volumes

JkLuJL
Ti p

nd

3
p

T,

146

respectively at the same temperature and pressure be mixed, the wd pressure p of the mixture at the temperature volume

will be

Hence

t,
The result can number of gases.
6. 6.

obviously

be extended

to a

mixture of any

Isothermals.
ideal
is

For an
temperature

gas the

relation

between

pressure,

volume and

expressed by the equation


...

(1)

and draw the various curves by giving to T different constant values in the above these curves are known as the isothermals of an ideal
If

we

take p,

t;

as rectangular axes

equation,
gas.

The

isothermals of an ideal gas are plainly rectangular

hyperbolaa

For a real gas, the isothermals are the system of curves obtained by assigning constant value to T in Van der Waal's
equation

147

These isothermals have been i raced in the figure below.

<?L

*&
2)

I)

fferentiating

with respect to

t',

we have

Hence the

locus of the
k

points

of the isothermals

at

which

dv

^o^-- i

g.

the points of the isothermals


axis,
is a

where the

tangents are parallel


<>

to v

curve

2i)

This curve
that

the

ciuve

MPN MPN

It is easy to see traced in the figure. has the maximum ordinate at the point
is

P whose coordinates

are (36,

--

The

isothermal through
is

value of clearly corresponds to a

which

given by
(*>

JSRTThis isothermal
curve
of
is

?
b
in

MPN
which

the figurb. repesented by PiPP2 intersects ail isothermal coh'&spdndihg to a

The
ffcltte

than that given by equation (4) itt two poifcts the isothefi&al at which M, N. Thus there are two points of curve MPN cuts it. As we move closer towards thfe isothermal

is less

148

and the two points coincide with the point P.

PiPP2

come closer and closer till they The curve MPN, therefore, meets

the isothermal PiPP2 in two coincident points. Since at every common point of this isothermal and the curve

MPN

dv

?~

-r

= o at P for

the isothermal.
at P, the

The isothermal

therefore has a

point of inflexion

tangent at which is horizontal. The curve MPN, however, does not intersect any isothermal which corressponds to a value of T exceeding that given by
equation

such isothermals are therefore everywhere convex to the axis of v. Consider now again an isothermal
(4).

All

lying below PiPP 2 such as


it

L MNB. The
art

curve

MPN inter esects


dv

in

two points r

and N,

which therefore

f = o.

Take any

point
it

Draw through

and N. the isothermal lying between a line parallel to traxis. It meets the isother-

V on

mal in two other points U and W. It is then clear that the points U, V, represent the states of the substance having the same pressure and temperature. We shall now show that ihe

Btate

represented by

the

point

is

unstable,

and therefore
-^is positive,
a

incapable of actual realization.


it

At
state

the point V,

accordingly represents a

such
is

that

decrease

in

volume, keeping temperature constant,


in pressure,

accompanied by a deer ease

therefore a slight increase in external pressure the decreases the volume of the gas in the state represented by

When

pressure of the gas also

decreases.

There

is

therefore

an

unbalanced external pressure which tends to decrease the volume


still

further.

The

state represented
arbitrarily

Since

was taken

by V is, therefore, unstable. and anywhere between points

N of

the isothermal

LMNR,

between

and

every point of the isothermal lying represents an unstable state. At points


so that the states represented

U and W,

^-S-is dv negative,

by them

149

represents the gaseous state and* the point U corresponding to lesser volume u believed to With this interpretation it follows represent the liquid state. immediately that a gas, whose temperature is kept above that
are stable.

The

point

W obviously

of the isothermal

PiPP2 cannot be
of

liquefied

by any amount of
PI

compression.

The temperature

the

isothermal

P P2

is

known

as the critical temperature of the substance.

"So loug as the temperature is above the critical temperature, no pressure, however great, can liquefy the gas."
substance in the gaseous state whose temperature exceeds whereas if it is below its 'critical temperature' is a 'gas
1
,

its

critical

temperature

it is

called a vapour.
in the preceding
articles

The laws enunciated


true of vapours.

are equally

Point

of the isothermal

P P P2
l

is

known

as the critical

c corresponding to this point of the gas. The values vc} p cj and are known as critical volume, critical pressure and critical point

temperature.

We

have already seen that

.....
If

275

pressure and critical temprature as the units of volume, pressure, and temperature the equation of state assumes a simple farm, which is known as the u reduced equation of state". Denoting volume, pressure, and
critical

we adopt

volume,

critical

temperature by
v

t;,

p,
*

respectively

we have

*>c

*~

~ p*
"

-_
L\

so that

1=8

ft

rf

=
values

B N
(2)

T=
the

Substituting these

in

equation

of

state

reduces to

150

This equation

is

the

same

for all

gases,

for

the

quantities

a and b which
It disappeared. Van der Waal.

vary from one gas to another, h;iv$ entirely is known a& the reduced equation of State of
.

Kx

1.

he tube of a barometer
arid

rises

to

34

;/

above the
is 30''

trough, mercury Find what changes are produced

in the

the

mercury column
in

high, the height of the column


:-

by the following operations successively performed


(i

As much

air is

allowed to

rise

through the mercury as

would

at atmospheric pressure

occupy 2" of the tube.


is

rod of iron whose volume equals 5" of the tube allowed to float at the top of the mercury column.
(ii)

(Take the

specific gravities of
.

mercury and iron as 13.5 and

7.5 respectively

Let the height of the mercury column be x" after the The volume of air enclosed in the tube above the operation (i). A (34 a*)", A being the cross mercury column is therefore If the pressure of the the air is p, we have section of the tube.
since by Boyle's law,
is

its
TT

volume

at

atmospheric pressure

TT

2 A,

A(34- x)p=-2 A

But the pressure p and

the

pressure of the column x of

mercury must be equal

to the

atmospheric pressure

where P

is

iho density of mercury,


(30),

Now 1T//P

since

the length

of the column under


(1)

is 30", atmospheric* pressure

whe have from

o4
or,

X
960 =

x 2 - Gix

151

Hence a = 24",

40"/.
is

Clearly the second solution 40"

inadmissible.

Hence the mercury column

will fall

by

6".

Let the length of the column after the operation well be x l


.

(ii)

as

Let a ba the cose section of


length.

the iron rod and

its

total

Then

The volume

of iron
,

immersed
9

in

mercury

= 75 *~
13.5
/.

_ j),

Volume of iron unimmersed


if

= ~ la
u

Now

the height

of the

mercury column

be

x\

inches, the volume occupied

by

air

=(34-*,)

A*,)

~-la

A-A
2TT

...by(2)

The

pressure of the air is by Boyle's law

n2A
(

34_a5

1)

20

A "34

xl

20

Now the

pressure at the top of the trough is

px

21T

_n
9~

or,

(13.5)

x^+2

(BQg) (13'5)

34^2^ 9

=30^(1315)

152

or,

(901286) (30-0:04-540 =
.*.

Xi

= 23
u

inches appro x. a further


fall

accordingly in the mercury column.

There

is

of approximately \T

Ex. 2. A diving bell is suspended in water at a fixed depth below the upper surface of water. The area of the surface of water within the bell is A. There is in the bell a volume pA of air under a pressure h as measured on the water barometer scale. A small object of volume A a and of specific gravity a falls from a shelf inside the bell and floats on the
enclosed water. Show that x the resulting rise in the level of the enclosed water satisfies the equation

x*

Will the bell contain less or more water than before ?


(1.0. S.

1927)

Ex.

3.

canister of weight
in the
is

without a

lid,

made of

uniform thin material


radius a ancj height h

form of a right circular cylinder of inverted and sunk in water, the whole

of air originally in
canister is

being imprisoned. in equilibrium with its axis


1

it

Shew

that

when

the

vertical, the height of

TT

the air column


at a depth

is

-5 xi

=-'

+ crA

and

that the water level inside is


'

crA

below the level outside, where <r~

a*hw

w being the
Show

weight of unit volume of water and H, the height of water barometer.


that the canister is stable in this position only if the is below the centre of gravity of the displaced

centre of gravity water, that is, if

JL
2A

^(1+*)
"

H(l-g)
<r)~ crA
(Tripos, 1932).

PI (2

153

great weight is lowered into the sea by a rope attached to its vertex prove that the depth of the level of water inside the cone when the vertex
4.
;

Ex.

circular

cone, hollow but of

is at

a given depth

below the surface

is

given by

where h

is

the

height of the cone and k

is

the height ot

water barometer.
Ex.
certain
5.

cylinderical

diving bell sinks

in water until a

portion a quantity of air,

is

remains occupied by air and in this position whose volume under atmospheric pressure was Show how far the bell must sink in forced into it.
air

order that the


position.

may occupy

the

same space as
C. 8,

in the first

(I.

Lower 1932)

(Ans. Twice the height of water barometer),


bulbs containing air are connected by $ hornoutal glass tube of uniform bore, and a bubble of liquid in this tube separates the air into two equal quantities. The bubble
6.

Ex.

Two

is

then displaced by heating the bulbs to absolute temperatures T and T'; prove that if the temperature of each bulb be
t

decreased
of

degrees

the

bubble

will

receive

an

additional

displacement which bears to the original displacement the ratio

Let

be the volume

one of the bulbs,


of the tube
is at

any and v the volume


bubble

of

>->*.

^^
( )

AO, when the

the middle point of the tube so that air on both sides of the

bubble

is at

the same pressure and


at the

T
same temperature (T).

volume and consequently also

154

Let the bubble move from

to 0',

when

one of the bulbs


bubble
equal.
is in

is

changed to

'.

the temperature of Let 00' x. Since the

equilibrium the pressure

on the two sides must be

pressure and absolute temperature of air to the left of the bubble are V-f 3 (/+$) y p,"V, where a IB the cross section of the tube,
r

The volume

are

Corresponding quantities for air to the right of the bubble

a <l^-x),p, T'
Lussac'si

Hence by Boyle's and Gay

Laws

p
or,

V+a

(H-a

_
be

V-f a (i-x)
^_
...

(T

T')+fa

(T-T')-aa T+T')
f

(l)

now changed to T t and T' will be x given by respectively, the displacement 00" ... y (T-T')-H* (T T0=a iT+T' 2t)
If the temperature
;

(2)

^abstracting
2

2) from (l)
(ce'-.r)

w=
A

(T-f T'~2*)
_2* _
2t

x '~ x x

T+TT

tube of uniform pressure guage consists of a U bore containing mercury, one arm of the tube being closed at the

Ex.

7.

of air at atmospheric pressure, and c. c. top and containing 15 the other arm being c nnected to the receiver of a condenser

the pressure of the receiver is atmospheric and by working of the condenser, the volume of the air ia the guage is reduced to -3*5 c c. aud the mercury is 15 cm. Find the pressure
Initially

in the receiver

Assume

meter

is

760 mm.

the height ol a mercury baroand that the temperature in the guage is


that

unaltered.

(Ans.

Approximately 4.68 atmosphere)

165

Ex.

8.

The

lengths of the arms of a

vertically are A,

k (h>k\

The siphon

is

siphon measured with liquid of filled

density

a greater than the density P of the liquid iti the vessel. Prove that the siphon will begin to work with the end of the longer arm immersed in the liquid in the vessel provided the
depth
of that

end below the surface of the


-=-*

liquid in

the vessel

exceeds (A

fc)

Let
filled

A B

0'

be the siphon

with a liquid of density cr. Let the arm B A=fc be immersed P contained
'

in the liquid of density

in a vessel.

Pressure at

of the liquid in the vessel is


IT is

#P

of-fTT,

where rf=AB, and

atmospheric pressure.
is

Plainly pressure at

of the liquid in the siphon


...

gak

.\ri~g(crhPd)
Pressure at
or

...

w
if

(I)

C
icr

is

gek.

The siphon

will

work only

crfe>n

gak

>g

9d)

or,

>-~(k-k\
The
figure illustrates a

Ex.

9.

siphon for

from a cask.

inches respectively.

The lengths of pipe DB, B A The end A is initially closed

drawing water are 40 and 60


1

arid

the pipe

from A to
pressure.

C filled

with water, the a!r enclosed being at atmospheric The end A is then opened and as water level in C A

decreases the level in


before
the

D B
risen

rises.

water has

to

If a balance is obtained just B. what length df vtratar will

156

Show that the siphon cannot be started by the process descibed above if the initial length of water in the arm A C is
less

than 120 inches.

Let
reached.

A C*#

and

let

by the time water

in

x be the length of water that runs out D B rises up to H when equilibium is

be the pressure of the pressure at A would be


.

If

air

in

tlm

equilibrium

position,

p+gP (y-x)
for otherwise

COB 80 -IT,

the liquid at

will not

be in equilibrium.

Here

TT is

atmospheric pressure*

Likewise pressure at

will

be

p+g? (DB)jp+0P
TT,

(36)

p+gP (ya) cos 60==p-f <;P


or,

(36)

157

Thus length of water


Also by Boyle's

that will remain in the

arm

is 72*.

Law
(160-HO
//)

p (160

2/-Hc)a^lT

a,

a being the

cross section of the tube

= ?P
Substituting for

(12) (30).

p and

y.x

iii

the above #)

gP (12)

(30) (160

72)=^P

(12) (33) (200

or,y^!20\
Ex. 10.
density at
level

One end
is

of a siphon

is

immersed

in a liquid

whose

any point

proportional to the nth

below the surface, the highest point


with

power of the depth of the siphon being at the

of the free Kuriace of the liquid, and the siphon is filled homogeneous liquid whose density is equal to that at the

Prove that liquid will flow ut the free end of the siphon even if it be above the level of (he immersed end
provided
(i)

immersed end

that the
less

vertical

distance

of the* free

end above the

immersed be

than

th of the vertical distance o* the former

n below the surface of the liquid


(iij
;

and
exceeds

that ihe atmospheric pressure

of the

pressure at the immersed end.


5. 7.

Work done by

the pressure of a gas in expansion:

Let us consider a gas enclosed


in

an

expansible

envelope.

Its

pressure on each element, d S of the envelope is continually pushing the element d S outwards along

-A-

*lj* >

158

Hence when the volume of the gas increases, a certain amount of work is done by the pressure on the various elements. Taking the element d 8 at P, the work done by
the normal to
it.

pressure
in the

will

be

/>d8. $n,

PP' taken along the normal at

where $n is the small length P. Hence the total work done

small displace ment produced by the movement of every small elem ent of the surface a distance $n normally outwards is.

Since

ihe

pressure

is

the

same

at

every point of the

surface of the enclosure,

we have

where dv

is

the

total

change

in

volume by the displace-

ment $n of every element of the envelope.


Hence work <iW done by the pressure of the gas ABC to A'B'C' is given by
in

expand-

ing from

Total work done by the pressure of a gas on its envelope in expanding from an initial volume v c to t>, is, therefore, given by

W=
we have

f+
pie.

o-

In the particular case of isothermal expansion of an ideal

coast.

w-f
*r

8'

dv *= c log

---

pv log

O ~J..

In the case of an actual gas, where the pressure and voluitoe in an isothermal expansion are connected together by Vita der

Waal's relation

159

tr

we have

Ex.

1.

piston

of weight

rests in a vertical cylinder of

section k. being supported by a depth a of air. rod receives a vertical blow jp, which forces the piston through a distance A; prove tha*

transverse

The down

(+TT*)

H-o

io g

Jtr

4 ffi 2i/
The temperature
of air
is

If

being- atmospheric
to

pressure.

assumed

remain constant,
a blow

imparted to the piston, At once with a velocity u given by

When

is

it

starts

moving

w
9

up *
is

Hei^ce the Kinetic energy generated

(^) tt 9

= #?L
y>

if it

Further a force (nfc-h*0) acts on the piston downwards and moves a distance A before coming to rest, work done this

by

force id (Tf&-H0) h.

160
in

Hence the work done


must he equal
to

compressing

air in the cylinder

^ w
volume of the
air

Now
final

the

initial

in

the cylinder

is

Since the piston is vt rest originally thrust of the atmospheric pressure on the piston and the weight
h).

being k (a

of the

pision BO that fcolow,

must balance the thrust of the pressure of the


TT

air

p being
gas
is

the pressure of the

air

in

the cylinder.
it is

As

the

compressed isotiiermally

work done on

given by

k(ah)
pdv

-1:ka
a-K) rk(a-i

J ka

'A v

ok log ~~ <*^~J

a log

a a

Hence we have

which gives the required


Ex.
2.

result.

piston without weight

fits

into a vertical cylinder,


is initially at

dosed

at its

base and
;

filled

with

air,

and

the top

water be slowly poured on the top of the tf the cylinder that the upper surface of the water will be lowest piston, show
if

when

the depth of the water

is

(VoA

*)>

where h

is

the height

of the water barometer and a the height of the cylinder.

Ex.

3.

An

ideal

gas

is

compressed to

th of

its

original

161

volume and

in this process the relation of pressure to


>f

volume

fa

expressed by

j9v ==const.,

where X has a constant


its

value.

It is

original temperature. Finallj constant temperature to its original \olume. expanded Show that the net energy expended on the gas in this cycle (i. a. is k times the work done in compression, work done on the
it is

then cooled at constant volume to


at this

gas)

where

(X

1)

log n

~^Ci
Let the
initial

volume, pressure and

abfaolute

temperature of

the gas be t>ijpi,T] respectively.

\\orkdon6onthegasiB

compressing

it to

the volume

is

given by

Since during the compression

X
pv

=c=
-X-l

I^et the

temperature of the gas in the

new

condition be

T/

Now

pressure p\ in the

new

state is

given by

is

constant for an ideal gas,

it

follows

that

162

The gas
volume

is,

therefore, at pressure,

V
temperature n
alter

*
'i

It

we keep volume constant and


r

temperature n

T,

to

l\ pressure becomes np

We
Volume
its

now have
*

the gas at temperature

T lf

pressure

n p^ and
to

If the

gas be allowed

to

expand isotherrraily
is

oiiginal

volume v^ work done by the gas

given by

n
total

work done on the gas

~/,W. where fr^l-

X^ n
It
IP.

accepted physical that a quantity of heat is the same thing as a quantity principle of mechanical energy. This equivalence was first established
Joule showed that a unit of heat viz a by Joule's experiments. calorie was equivalent to 41.8X10 6 ergs. This constant is known as Joule's mechanical equivalent of heat. With its help we
could express any quantity of heat in terms of units of mechanical energy.
If

5.8*

Thermodynamics.

now an

wa now conwder a gas


forrett

which there act

as a system of molecules between which have a potential, the gas wili

possess energy, which

will

consist

of the kinetic

energy of

molecular motion and the potential energy of intermolecular


his energy, whjeh yyill of course, depend on tjie confiand motipQ of the molecules is known as internal guration of the gas. Lpt U be tfie internal energy of a gas, so energy
forqes.
'1

that

wp may

write

m
wfrere
is
(

is

the

mean energy

of the

mplecqles,

and #

the potential energy of the intermolecular forces.


Tt is clear that

the difference between the energies" in two given states depends only on those states and not upon the mode of change from the one to the other.

Let us suppose that a quantity dQ, of heat expressed in* mechanical units is absorbed by the gas from some external source so that as a consequence its pressure, volume ancj temjLet us suppose that after this absorption the assumes a bteady state and that in this state its internal gas again $ow the work done by the gas in expanding du. energy is u

perature change.

dv is pdv. Hence by the principle of of energy change in i he energy of the gas must con&eivation bu equal to the energy supplied minus the work .done by the gat
from volume v to v
iu expansion
i. c.

du~dQ

pdv

or,

dQ=du

+ ffdv.
Law
of Thermodynamics.

This equation expi esses the First


5.9.

energy of an ideal gas. The State of aa defined by its pressure, temperature arid volume. id^al gas Since these three quantities are not independent being connectInternal
is

ed by the relation^

const , the

state

of the g^s

is

given by

any two of the

quantities p, r,

and T.

Hence

internal energy

164

a function of two indepetdent variables only. We can easily show that in the case of an ideal gas it depends only on its

is

temperature.

If a

suddenlyexpand by communication with a previously exhausted vessel, a number of intricate mechanical and thermal changes will at first occur,
Let the walls of the enclosures be absolutely rigid and non Let W T be the internal energy of the gas in the conducting.
initial

gas initially in steady state be allowed to the opening of a &top cock which establishes

state

and u^

in the

final

state.

Keither thermal nor

mechanical energy has been added

The

walls of the enclosure being of Thermodynamics rigid, it follows from the first law

the gas from without* non conducting as well as


to

wait long enough to allow thermal equilibrium to be established, we shall find that temperature of the gas remains But the volume of the gas changes. The change of unaltered.
If

we

volume, therefore, has no effect on the internal its temperature remains constant,
or

energy provided

Hence taking r,T as independent

variables

(J)

6. 10.

Specific

Heat -

Specific heat of a substance

is

the

amount of heat required to raise by lc the temperalure of the body to the amount of heat required to increase by lc the temperature of an equal weight of water. If an amount of heat rfQ produces in unit mas? a change of temperature dT, the
ratio of the
specific

heat I'B*~;

165

At a given temperature any number of specific heats might be defined according to the conditions under which heating' For the quantity of heat absorbed in a given Cakes place.
change of
state

depends partly on the manner


about.

in

which that
consider

change is brought *mly two casea:


(i)

In the case of a gas

we

shall

when

the

pressure remains constant,

the gas being

allowed to expand,
(ii)

when

the volume remains constant


in

The

specific heat

the former case

is called

the specific

heat at constant pressure and is denoted by cpy whereas that in the latter case is known as specific heat at constant volume and
is

denoted by c^

From

equation (l) of
,v
_

5.9

it is

easy to see that

=
dT

rff
/.

3T

we have
But
for

rfQ

cv

+ pd*.
~ =
const

an ideal gas

/
no that, rfQ

pdv+vdp

cd'
-vdp,

= cdT

c. c*T

or, cv

cw

const.

Thus

cp
it

>cv
is

since c

is positive.

Now

found that cf
have,

is

constant within a considerable


is also

range of temperature.
range.

Hence

c,

a constant within

tfte

We

166

For a perfect gas 5tis found e


v

to

be

1.4 1,

tie mark:

In 5.8

we found

that

But as pointed out

in

M,

~m 7

=f

energy of the 'molecules be ignored, ibis expression coincides with that given It ib thus seen that the internal energy of an ideal- ga& "in (l).
If

^ the

contribution

from

the

potential

oi the kinetic energy of its moleeuUs, the due to the molecular forces being &ero. potential energy

consists

entirely

An adiabatic expansion or Adiabatic Expansion compression is cue which takes place without any heat being imparted to or lost from the gas. For an adiabatic change therefore, we ha\e dQ==o,
5.10.

bo that
or,

cfo+

pdvo

cv d

But
Eliminating
cfT,

we have

But c

or,

~p^==const., where
is

or,

'

the equation of adiabatic change which no heat is lost or gained. Change during

This equation

ive,

167

The
any

relation

holds in the case

of a sudden dilatation or
is

compression which occurs so rapidly that there


sensible
loss

no time

for

of heat, or for

any addition of heat from

external sources.

Ex.
perature

certain
is

volume of a

perfect

gas

at absolute

tem-

To

becomes T^
isothermally
adiabatically

compressed adiabatically till the temperature and volume *>i. It is then allowed to expand

till its

volume

is

t>

a,

after
is

which

it

again expands

till

its

temperature
till

If the
to
its

gas

is

again

compressed isothermally prove that the work done by the gas in the cycle of transformations is

it is

restored

initial state,

BCTi-T
where
Let p Q v c
,

)log2f-s-

R
T

is

the gas constant defined by tho relation^*? ^R.

denote the

initial

state of the gas,

and
In

1919 letp^
first

be the state after the


transformation.

adiabatic figure

the

denotes the

initial state

and B,

the final slate.

Workjby the gas


is

during the compression

=
1]

where

pQ

vQ

=j

But

168

/.Wi-^
The gas
is

(Tx-TJto

...

...

(1)

now
*>

allowed
a , its

rolume Vi to volume done by the gas is

expand isothermally from temperature remaining TT, work

- f"
J X

...(2)

V1

During the next adiabatic transformation, work done by the


gas
is

W
where

s=

P* **>

1-X
at

are the pressure and volume in the slate the temperature of the gas is To.

p B v^

whioh

B*

**--TT
:.

m
8 =]

...

...

Work done
is

by the gas during the final isothermal transformation

W 4 = RT

Ze#^

......

(4)

vs
vs
*V

=p
v

t?

=R To.
i

= R T ^_ = R TT v X
S

"

Similarly

it

may

be shown that

169

It follows

*o

"3

Equation

(4) gives

^L

......
total

(5)

From
gas
is

(1), (2), (3)

and

(5)

we

see that

work done by

the

R (T T

-T

log

J
v\
'

Remark: The cycle of transformations which the gas supposed to undergo is known as Carnot's cycle.
5 12
If the

ifl

Atmosphere

iu

isothermal

(conductive) equilibrium,

temperature of the air be as3Umed to be uniform,


is

throughout, the relation between pressure and density

=kP
dz

...

.*

(!)

The equation of hydrostatical equilibrium under the aciion of gravity, which may be assumed to be uniform, is
dp**
-axis

ffP

...

...

(2)

being the vertical oriented upwards.


(1)

From

and

(2)

we have

log

=
Po

3 (8~
k

);

p9
If z

being the value of p at height

=o

and

H be the height of
,

here of density P

the homogeneous atmospthat could produce the pressure jp ol we hava

170 )

/.

=
Z
*

Hence

pp

Q e~~

or,

=
off exponentially

Thus the density of the atmosphere falls with the height above the earth's surface*

H
is

Ex'l In an atmosphere at rest under gravity, the height of the equivalent homogeneous atmosphere above any point

at height A,

measured in terms of
h.

air of the density at

height h
at height

given as a function of
is

Show

that the pressure

given by

C h dh
J
o

where p^

IB

the pressure at A = o.
in

Show

that

independent of #, extends to infinity.

an isothermal atmosphere H is a constant and deduce that an isothermal atmosphere

Ex.
weight

2.

spherical balloon of radius r and total floats with its centre at a height h above the snrlact-,

large

of the earth.

Show

that k

is

given by

</\V

kg
of the earth

~-

is

the pressure at the surface a certain constant.

where p Q

is

and

It

171

Let

be the centre of the balloon

and P be any point so that angle between OP and the vertical through is
9.

Consider

now

an area

of

the

spherical surface formed by the revolution of the small element PP'

(=r

dO) round the vertical through 0,


is at

Every point of this area


height (h
that pressure at

the

same

r cos 8) above the earth, so

any point Pis


r cose)

P.*

| k

(h

acting normally along the radius

PO

Hence the component of the pressure along the upward


vertical

through

is

pe

cos

6.

Vertical component of total

pressure

acting on the belt

considered above

is

k
2 * r 2 sin 8

k
e

d8p

cos

0.

Hence

total

upward

thrust of the atmospheric pressure is

-y-

COS 8

sin 8 cos 8 e

d8

-*
k
'*

f +1
\

"5
e

dx

J -1

172
Tr

'

k 2 sinh

Since the balloon


its

is

in

weigh W.

equilibrium, this must be equal to

of gravity with height and assuming that the temperature of the air is constant at all heights prove that at a height x the pressure p of the air

Ex.

3.

Taking

into account the variation

is

given by

P
'

=_
k~
,

the earth's radius, pj P and p P ol #. ar the values of the pressure, density and gravity at the earth'
is
,

where a

balloon

pressures

carrying a self registering barometer records equivalent to h and A r inches of mercury when it
fractions

ascends to heights equal to


radius respectively.

a and o^ of the

earth's

Prove that

gas tight sphere containing a force mg to prevent it from rising when the hydrogen requires lowest point touches the ground. The total mass of sphere and hydrogen is M. Show that the sphere can float in equili-

Ea

small

hollow

brium with where

its

lowest

point at

a height h above the ground

>

Pe bsiag the density and pressure at the ground.

( 5. 13.

173

Atmosphere in adiabatic (convective) equilibrium.

in

The law of variation, of atmospheric density investigated the preceeding article is the law which would undoubtedly
if

become established
for a sufficient

time.

the earth's atmosphere were left at rest This is, however, far from being the case,
the
earth's

for

under actual conditions

atmosphere

is

con-

is

tinually being agitated by currents and storms, so that there an incessant mechanical transference of air from one part
Besides, conduction of heat in

of the atmosphere to another.

gases is very slow. It, therefore, results that the atmosphere is never permitted to assume the equilibrium density which has been investigated on the hypothesis that the atmosphere is at a uniform temperature. As an element of atmosphere moves from

one layer
its

to another, its

to

temperature. that of ite new surroundings it moves again to some other Hence the density of the atmosphere is determined not layer.

density changes arid consequently also But before it,s eniperature Las adjusted itself

by the equalisation of temperature necessary io a permanent state, but by the condition "that an element of atmosphere on
being moved from one place
quisite

by

pressura and volume conduction taking place."

another shall take up the rewithout any loss or gain of heat


to

pressure

and volume
5*11.

in

the

the law connecting the atmosphere is the adiabatic law

That

is,

found in

We

thus have for the equilibrium of atmosphere

dp^gPdz.
Also the adiabatic law connecting

p and

is

dz

174

where P

is

the density at

z~o.

=
*

RPT,

JJ-

=fcP

X1 =RT

*x

'

or,

T~ T

const,

where

Ta

is

the temperature at 3=0.


the

Thus the temperature decreases as we move upwards, amount of decrease being proportional to the height.
Ez.
1.

Show

that there is a superior

limit to the height of

an atmosphere in convective equilibrium.

This limit

is

where p and P are the pressure and density


su rface.

at the earth'*

Ex.

2.

If the

given function
pressures at

a height z is a ratio of the (z) of the height, show that the

absolute

temperature

at

two heights z^

z% is given

by

* e,. K f'ji ^1
P\
<yj_.

A/

,\

} z

k being a constant defined by the equation pIcPT.


the temperature and pressure an simultaneously recorded and a curve is drawn plotting the absolute temperature against the logarithm ot the pressure, prove that

As an asroplaao asudnds

height asceuded between two readings

is

175

where x = log
Ex.
that
3.

p.

and absolute temperature

In a vertical column of perfect gas the pressure at any height z are and T. Prove

where

p^
is

T^ are

pressure,

density and absolute

temperature at the botroin.

Height

measured in an aeroplane by means of a specially

graduated aneroid barometer. The graduations are such that the true height would be read direct if the temperature oi

atm sphere were uniformly


pressure at ground level.

at

10c.

Show

that the instrument will

read differences of height correctly

whatever the barometric

height when the temperature is not uniform it is necessai'y to read the temperature during ascent Show that the true height corresponding to a recorded height
the true
?+

To ;md

fTl

j:
and

933

^ Zl

'

w ^ere

Zo

i8

the reading at ground level


the reading
(Tripos
is z\.

the absolute temperature

when

Pt

IT,

1921).

Ex.

4.

Assuming the temperature of the

air to

diminish

uniformly with the height piove that the difference of level of

two

stations is

where
centigrade,

H is the height
T 0> T
lf

of the

homogeneous atmosphere

at

are

the absolute temperatures of the stations,

(
ho> h\

176

are the barometeric heights reduced to centigrade. Prove also that if as an approximation the temperature were
.

taken to be constant and equal to \ (To+Ti) the calculated height would be too great by a fraction of its true value equal
rn

roughly

to J

*
[-

AO

m rriM assuming (T
2
I

-T

t )

to

be small.

-M
flexible

Ex,

5.

of total mass
rature
lift

perfectly m. At the

balloon contains
level
it

a light gas

ground
air.

is at
it

the

as the surrounding
it

Prove that

will

same tempeexert tbe same


air

at all heights if
it
;

remains at the same temperature as the

round
lift

at height

but that if the gas inside expands adiabatically the will be less than the lift at the ground level by the

amount
I

myv
where
under
cr

-0I
,

1-

c'-x

is

the ratio of the density


conditions,
X,

of the

air to that

of gas

standard

X' the

are

the

ratio

of specific
at.j

heats for air and ihe gas and


t. e.

H
is

is

height of the

ospheie
vanifeh.

height at

which pressure, temperature and density


never
fully

It is

supposed that the balloon

expanded.
let P'

Let P be the density of the


that of the

air at

height z and

be

gas inside the balloon when

at this height.
is

Then

the volume of the gas inside tbe balloon at thin height

-' The
and

volume of the

air displaced

by the balloon

is

accordingly-^
is

therefore weight of the air displaced

by the balloon
z

P.

Hence
balloon
is

the

upward

lift

at
.

any height

by the

air outside the

mg

fp>)

where

P the ratio of the density of surround^is

ing

air

at

height z to that of the gas inside the balloon.

Now if

177

)
is

he temperature
it,

T
we

outside

the balloon

the

same as that of

the gas inside


the gas.

have^==R,

for the air

and
-j!j~

=R

/
l

for

The

balloon being flexible,


at

p=p'

at

auy height and

T=T'

by hypothesis, hence
P
R'

any height z

Hence the upward


heights.
If the gas inside

thrust of the

air

is

the same at all

where p' P' gas and Y is the


y

l=: k P' expands adiabaticallywe have p are the pressure and density respectively of the
ratio of specific heals. for air is

y/f

The equation of pressure


Also

dp=

p~ kP
=
3

/p^~l
But
/.

gz + cons. y

P^o when
p

H
r i- ^i nj &.
1

^--^ re*
orP =

K'[l-g]
at height z is equal to

/. pressure

Thi
the bolloon
at height *

is

also
is

the pressure of the gas inside the balloon as Hence if P' be the density of the gas flexible.

178

*K'

V
[1

?
JLI

-e
1

'

X-X
1

Hence P'/*K'\x'
"P

^~F
if

"

M 11

"^
l
'

* N ~7x-i)x'

In particular

=
x
1

I=

'

/.upward thrust

at the

ground

level

minus that at a height

Vis

m
/
f*

( "^

\\*t

Ex.

6.

cylinderical

well contains air at constant

tern*
.

perature, and densities at the top and bottom are P and P Assuming g to be constant show that the mean density of the
air in the well is

..

log.P,
("pro

\ '

(M.A. Punjab Univ. 1935)

We

have

for equilibrium of air

179

where p=k P
-

*rf. *

_#?
or,

P=P e

dZ

A verage density P

=-

_
hg
h beiag the height of Ihe cylinder.

Since P 1

=P

P1 r

Po

Hence P

Ex.

7.

A be^
its

Lube of uniform bore, the arms of which are

at right angles rovolves with constant angular velocity

about

the axis of one of

immersed

in water.

its extremity arms, which is vertical Prove that the height to which water will
is

and has

rise iu the vertical

arm

JL(i

180

a being the length of the horizontal arm, IT the atmospheric of water, and k the ratio of the pressure, and P the density pressure of the atmosphere to its density.

Ex. 8

When

the pressure of a given mass of a gas


is

is

p, its

volume

is v. If

the gas

allowed to expand slowly from volume

V to volume

u,

show

that

W, the work done

is

f p
I

"

dv.

J
For a gas which
.

v*

is

such that for p


Q

>p

W = jtw+ p

+p

- 4"

const,

verify that v decreases as

increases from

p and

find v in terms

of p.

(Tripos part I 1938)

-I

Q
I

Ans. #*=

-e

PQ
Ex.
9.

pe

*P

hollow closed rigid ve* sel whose walls are of negligible thickness has volume V and contains air of density a
at atmospheric pressure
is
If

and absolute temperature


is

held below the surface of a lake of

The vessel uniform density P and temT.

perature T.
vessel

small hole

made

at

the

lowest

point of the

allowing water to enter the vassel but no air to escape. All changes take place isothermally. When water ceases to
enter the vessel, the water surface inside
it is

at a depth h

below
float

the surface of the lake.


freely (totally

If the vessel,

when

released,

can

of M, V,

IT, <J,

P and

immersed) in this position, determine h in terms is the mass of the vessel when g where

empty.

Find whether the equilibrium


vertical displacements,

is

stable

or

unstable

for

(Tripos 1939).

Ex. 10. Two cylindrical gasholders, of T and weights and of cross sections A! and A 2 float in water with their f The combined tops at the same height h above the water. of gas in them would quantity occupy a volume V at the atmosProve that pheric pressure TT.

W
where P
is

the density of water, and the buoyancy of the water the side of the gasholders is displaced by neglected.

In ascending a mountain the teinperaiure of the air a quantity proportional to the by height ascended and A, k are the observed heights of the barometer at
is

Ex. 11
to

found

decrease

two

as A

stations whose difference of altitude is z. Show that varies m m & where is a certa n and where changes conataut, of density in the barometer are uoglected.
,

CHAPTER
When
examined

VI.

Capillarity.
6. 1.

the
it is

surface of water

contained in a rase is

caraiully

be imperfectly level becoming considerably ctirv3d in the vicinity of the walls of the vaae. Again, if the lower end of a vertical tube having a narrow bore
to
is

found

immersed in water, the


is

liquid

rises

some distance above the

level of the outer surface of water.

within the bore

curved and

free surface of the liquid appears to cling to the wall of the


is

The

tube at a definite angle.


If a

angle of small quantity of mercury is poured on a capillarity." If the quantity of mercury is horizontal plate, it forms a drop. assumes a nearly spherical shape. A exceedingly small it
larger

This angle

called the "the

spreads out into a cake-shaped mass with a nearly flat top and with rounded edges. These and other allied phenomena are explained on the hypothesis that the molecules
quantity
liquid
forces,
is

of the liquid and the solid with which the attract one another with forces, capillary
<i action is

in

contact

whose range

very small.
introduction.

6*2

Historical

Many

of the

phenomena due

to capillary forces were described us early us 1670 by Borelli; but the first accurate observations of capillary action of tubes ancj

were made by Francis Hawksbee (1713), who glass plates ascribed the action to the attraction between glass arid the liquid.

He observed

that

the

effect

was
.t

the

same

in

thick

tubes

as in thin tubes

and concluded th

which are very near the


phenomena.
liquids

only those particles of glass, surface have any influence on the

Olairaut in his "Theorie de la figure de la Terre" (1743) uttemp,ed unsuccessfully to calculate the elevation of
in capillary tubes.

In 1805 Thomas
to

the

free

surface

of a liquid

that

of a

Young compared membrane equally

183

stretcnea in

all

directions.

He

also

gave

for

the

first

time

the differential

by such a surface. But a-? Laplace and Poisson have pointed out, "the identity of the surface of a liquid with that of a stretched membrane can only be
the consequence of
1

equation to be satisfied

and not the principle of the

solution of the

problem'

.*

explanation of the capillary phenomena on the basis of a mathematical theory was not forthcoming till
satisfactory

1806,

when Laplace formulated


liquid

his

theory of capillary action.

The fundamental hypothesis


molecules of the
acts

of his theory was that any two attract one another with a force which

along the

line joining

them and whose magnitude

is

pro1

portional to their
to

masses but depends on their distance according an unknown law. Laplace also assumed that these molecular

forces

become

infinitely small

when

the distance between the


8

molecules exceeds a small distance

called

the "radius of

molecular activity."

On
all

he explained nearly known.


It

the basis of these simple assumptions the phenomena of capillary action then

had been noticed that the angle of capillarity for a given contact of liquid and solid was constant along the curve of suffered from the the liquid and the solid. Laplace's theory
defect

that

it

was not

the case of a cylinder. of the angle of capillarity by applying the theory of virtual work.

able to explain this constancy except in In 1830, Gauss proved the constancy

Abont the same time (1831) Poission in


The'erie de
1,

his

"Nouvell

action

Capillaire" pointed

out that the theories

of Laplace and Gauss had taken no account of the fact that the density of the liquid near the free surface is not constant By

means of extremely complicated but very well conducted

cal*

dilations he proved the constancy of the angle of capillarity and established other results already given by Laplace and Gauss*

simplified considerably by Mathieu & Poincare/ Hagen, Brunner and others studied with the utmost precision the elevation and depression of liquids by capillary

Poisson's analysis

was

and showed that the theoretical results of Laplace,, Poisson, Gauss etc, are completely in accord with experiment. Plateau's celebrated work,- "Sur Les Liquides Soumis aux seules
action
forces

moleculaires"

contains

an

elaborate

account

of the

mathematical theory of capillarity and of his


series of experiments.
6. 3.

own

extensive

Theory of capillarity,

in its

most general form,

is

the

study of the forms assumed by the surfaces of liquids in contact with each other ]and with solid bodies. The curious forms of

euch surfaces become explicable only on the hypothesis of the existence of intense molecular forces having an extremely small

range of

action.

Let m\ and m* be masses of two molecules of the liquid be the distance between them. The molecular force and

between them

is

then assumed to be

being an unknown function, which becomes infinitely "the radius of molecular activity". exceeds email when r
/i
if
,

Here

it

is

tacitly

assumed that the linear dimensions of the

molecules are infinitely smaller than the length of any line from the surface of one to the other, for otherwise the quantity rig

would have no meaning


supposed to be spherical.

unless,

of course, the

molecules are

proceed further it is now necssary to make some assumption concerning the constitution of matter. No doubt the application of Integral Calculus to the determination of the

In order

t;>

produced on a molecule of a body by the neighbouring molecules is rigorously justifiable only on the assumption that matter is completely continuous and that there is no vacant
action

185

however small, between the molecules. Still such an On the other hypothesis, as remarked by Lame', is inadmissible. band the assumption that the molecules of matter are very distant from each other so that the space surrounding any molecule is comspace,

paratively void of matter,

makes the

application

of mathematical

media analysis almost impossible. We, therefore, s*rike a via and assume that matter is quite closelv packed, the dimensions
of molecules being greafr compared with the distances between Matter may, therethe surfaces of two contiguous molecules.
fore,

application

be regarded as sufficiently continuous so as to permit the Calculus at any rate as a very close of
Integral

approximation to truth. But on this assumption, when calculating the molecular force between two very clo-^e molecules m\ and m the molecules as subdivided it will be necessary to imagine both into infinitesimal m^ and &m. so that the dimensions of
'

parts

each part are infinitely smaller than two points on the parts cons dered.

the distance between

any

If r is be the distance bet{

these parts, the attraction between them will be Sm &M, we shall obtain f (rig ). Integrating over the two molecules In whnt follows, therefore, we the force acting between them.

ween

were con* apply the Integral Calculus as though nutter tinuoua


shall

Consider a homogeneous liquid in contact with a Let (X,Y, Z) be the components of the external solid body.
6. 4.

force per

unit

mass acting on the

liquid.

In order to determine

the equilibrium of the liquid


virtual work.

we

shall

apply the principle of

,,?tt a .

..
..

m;,

..

be

the

molecules of the

... those of the solid. Let rb be Ma, the distance between the molecules wj and m t and R u that

liquid

and

M lf M

lf

between

and

m m

and M.. Let and w?, M,

m m,f (r
{

i8

be the attraction between

(r .) be the attraction
{

between

and M..

Then

the virtual

work of two equal and opposite

forces

186

between m\ and displacement S r

acting along

r, g

consequent upon a slight

ia

is

Similarly the

virtual

work of

the force between n,

and

M.is

-m M
{

(R |8 ) S

Ru

The

virtual

work of the external

force (X, Y, Z) is

Since the

system

is in

equilibrium, the

sum

of the virtual

works must be zero

for

any displacement consistent with the

laisons or constraints of the system.

Or,

S^
S^.nii M. F (RJ&

B =o
ia

...

(1)

Hera the
the second

first

sum extends over each molecule


is

of the liquid f

double extends over every arrangement of two molecules of the liquid ; and the thirl sum, which of the (solid paired is also double, extends over every molecule

sum, which

have taken the factor with every molecule of the liquid. in the second sum in order to ensure that each pair of molecules
of the liquid contributes towards the

We

sum only

once.

fa,

Je
r

f(r)dr=

Jr
when
r

since /(r) is infinitely small

>8.

Similarly let

ir)

*<?=

(r)

dr=V(r)

..

/(r)

$r=0 (r), anrf-F (r) ^r=^^ (r)

Accordingly equation (l) becomes

187

Remark

Since/
so are
<j>

(r) awrf

(r) are negligibly small

when

r76

(r)

and

(r).

Now, set

U=
SU^o

condition of equilibrium is, therefoie, for every consistent with the laisons of the possible Displacement system.
6. 5.

The

We
first

shall

now

evaluate the double sums encountered

in the preceeding article.

Consider the sum


so

S S
5

rn {

m$ #(r

if

).

Take
radius S
liquid.

a molecule mi
is

situated

that the

round m\ as centre

completely

sphere of immersed in the

The sum

m S m
{

<t>

(rig ),

where

extends to every

molecule of the liquid except m^ is clearly the same as if the sum extended only to molecules within the sphere of molecular 6. activity, for 0(r i8) vanishes for rit

>

Accordingly
8

S.

w.

<l>

(r^^P y(4 ^dr)


(say),

<t>

(r),

#
o

(r)

6?r=cons==L

where P

is

the density of

the liquid,

surround every molecule by the sphere of molecular activity in such a way that the latter remains 2 .S nm mj>(r) would wholly inside the liquid, tho calculation of be a simple matter, for it would be L=ML, being
If
it

were possible

to

=2^

the total mass of the liquid.


to do so for molecules Actually, however, it is impossible at a distance less than 6 from the free surface of the liquid or the

surface of contact of the liquid with (the solid.

We

are,

however,

able to do so

by

thickdrawing an imaginary shell of constant

188

ness 6 surrounding the free surface and the surface of contact between the liquid and the solid. It is clear then that

MLaeSjSs [m

0(ris)]+

S S m
i

m,

(r^)

where the summation 8 exteds over mole of the umles imaginary shall. We now proceed to prove thet Si 8 8 m m* (r ) is proportional to the sum of area of the free surface and the area of the
{ <t>

is

surface of the liquid in contact with the solid.

and A' B7 be the surface of the liquid and <he outer surface of
Let
the imaginary
shell

AB

respectively.

The two

by hypothesis the normal distance between parallel, them beings. Consider now an element of volume at Q, whose mass is Pds dz where z pQ, and ds is an
elementary
area
of

surfaces are

the

surface

through

AB

parallel to the surface or A' B'. Draw a sphere of radius

round Q.
axin of

coordinates

with
at right

Q
dS

as

pole, Q^> as
to

the

Take polar and any line

through

Q
)

angles

Qp

as the axis of ox.

Let us take
P(
r,0,

from

same as

QP = r.

so small that the distance of any point any point of the elementary area c/S at; Q is the

Hence

Si

Sa

m-,

r is

(P dSJz) (Pr*dr sin B dB dty$ (r)


(r)

6d

of the triple integral withui brackets are to be the field of integration is the volume between the such that 7 surfaces A B and A' B and the sphere of molecular activity
limits

The

round Q.

Since tho radius


to the free surfac
*

of the

sphere

is infinitely sn>all

compared

of the liquid and the

surface in

contact with the solid, the sections of the sphere by the surfaces A B and A' B' may be taken to be planes.

189

Hence

in e

~
JJJ
2^
I

sector
1

a
z

Qbp

JaQ
2

a',

Z>Q 5
-1

JJJ

a'Qo>'

cos"

2 sec
-

cos
'

3e
-

rf*

st"

6^

Wf^'-f

JV

fl

cZ0

* ( y) dr

Jo

Jo

COS

^ J

-S)secfl
c?V
-

e c?6

r2

(r)rfr

J
;

cos

z-ej

li

(a
;

(say).
(ri8 )

Heuce S S4 m, m.

=
s
t(

P2

ff
where
Bjis a constant

,=B S

surface

and S is the sum of the area of the free and the surface of contact of the liquid with the solid. Let the former be a and the latter be X, then

SI. mi

m,

(ris )

= ML
A,
Similarly

(X+cr)B

being constants.
easy to establish that
i8 )

it is

SJ S. Wi jl/.V(tf
Henoe by equation
(.2)

=EX.

of
*-

6.4

U=

./(x dx

190

+Y dy +
positive

dz) m,

+ A-B

- E)X.

where A,B,E are

numbers.

by

have assumed that the density P is uniform As mentioned in 6.2, this assumption throughout the liquid. Las been criticised by Poission and Mathieu. The results obtained them on the assumption that t ie density of the liquid near

Remark:

We

the free surface varies ara, however, similar to those obtained

above.

Equilibrium of a liquid contained in a vase. If the only external force acting on the liquid be that of gravity, then
6.6

taking the upward vertical as z-axis,

we haveX = Yo,

Z=

-g.

Hence

U=

g^ fm,

cte

+ A- *B a
Ba-(iB
|B
Or

(iB-E) X
E)X

(B~ E) X
a
is

where dv

is

an element of volume of the

liquid,

the area of

the free surface


liquid

and X the area of the surface of contact of the

and

vase.
\*e

For equilibrium

must have

or,

gVSfffzdv

Scr

(iB

~ E)$X=o ...... (1)


A'

We now proceed to
tion.

calculate the variation

of each of the three terms of the

above equa-

/*'*'

Let

AB

be the free surface of the liquid

in

its

and AJ equilibrium position

its

position
sur-

after a virtual displacement.

These two

faces enclose

a volume between themselves.

volume into small elements by M', NN' at each point of drawing nomals the contour of a small element rfS, (MN), of the surface AB. MM' $n> the volume of the element is c?t= SndS.
split

We

this

If

191

Now ,/V" fgVzdv


the

is

the

with respect to xoy plane.

of the weight of the liquid Its variation is therefore clearly

moment

moment
and

of the weight of the liquid contained in the surfaces

AB

A^.

Hence S
where the right hand

fffgVzdv^Vgff*Snd$>..\%\
a of the free surface area,
CTj
,

field

of integration of the double integral on the side is the free surface of the liquid.

To

find the variation

we

require
1
.

the difference between


/.
cf

a and

the areas of
...

A B and A 1 B
...

= <7,

_a

(3)

Let
vase and
at

L be

the curve of contact of the surface


.

AB

with the

L l the corresponding curve of A a B x Draw normals each point of the curve L to the surface AB. These normals intersect the surface A. L *I$i in a curve L', bounding an area a'
of the surface

A1 B
L'.

Let
curves

cr"
x

be the area of the surface

AA B

bounded by the

and

Accordingly,

$ <T

//

cr'-fa

-- a

rfS

being an element of the surface

...... A B and rfS of A B.


8
7

(4)

Let us now compare

d&

and dS.

principal linos of curvature of the surface

Imagine the net work of drawn out on it.

192

MN and MP be two such lines through M, then the normals at M and N intersect at C and the normals at M and P, at C' andMC and
Let

MC'

are the radii of principal curvature.

Since
of the

<PMN is a right
MPQN
is

angle,

area

mesh

MNXMP.

dS The

normals at the points M, N, P, etc trace a


corresponding mesh M'N^Q'P' on the surface A'B', whose area dS=M'N'XM'P'.

Now MM'

&n.

Pence from the

triangle

MPC' we have

MF^Mcy""
R! being a
Similarly,

MCf

T^
M.

principal radius of curvature of the surface at

WN'

MN

Sn "17

neglecting term of the second infinitesimal order*

Hence

dS'-dS= - Sn (4-

+) dS
(5)

L'

Let us

now

evaluate the annular area


surface

L! and L' of the


curve
at

A
to

between the curves on the B Take two points A and


cr"

infinitely near

each other.

The normals

A A'

and

CO

and C respectively

the surface

AB

meet the curveL'

in points

A C
;
,

infinitely

near each other.

193

From A' and


to

C'

draw

lines

A'Ai

& 0'0

lying in the surface

A B
l

and perpendicular
ele-

AA' and CC'

respectively.
^

These lines divide the areacr" into ments A'A^C', whose area is sensibly

d* being the element


Defining
the

AC

of the curve L.

between the

free

the angle capillarity' to be 'angle of surface and the surface of the solid at any

point of the curve L, it is easy to see that angle A AiA' after the displacement can be identified with 0, the angie of capillarity.

Hence A! A'

A A'
cos
*t n

cot 9

=
sin

an,

80 that

= f Cdw~ Cc J J J si sin
it

follows from (4)

and

(5) that

Sv~ f w-lsnit- f f (A + 1 ^ n J J L RI R a J LStn 9


Finally we require the variation liquid in contact with the solid vase.

(6)

\
It

of the surface of the


is

clear that

$X

is

the area comprised between the lines this area is AA X C^C.

L and

Lj.

An

element of

Hence

f
df.

tin 6

194

The Equation

of Equilibrium (1) accordingly becomes

! f f 4-Kr> 2y*/LR 1 R t

satisfied whatever be the value of $n, of course, ^that the value of $n is itself compatible provided, with the laisons of the system. Now the liquid being assumed

This equation must be

to be incompressible, the algebraic

sum

of the

variations of the

volume resulting from the virtual displacement of the bounding We have already seen that the volume surfaces must be fero.
of an element

comprised between the free surface and

its dis-

placed position is &n dS. liquid imposes the contraint,


o
If
isfied

Hence the incompressibility of the

(8)

take into account this equation, equation (7) is satn compatible with tbe laifor any arbitrary value of

we

sons if

-0P+ y-fg +
1

= const.=K

(9)

and (E

| )- 5 6 2

cos

6=0

(10)

These conditions are obviously sufficient; for if they are satisfied, the second term of equation (7) vanishes, whereas the
iirst

reduces to the product

SndS and

is

therefore

zero

>y (8).

195
also

necessary for if they are not satisfied, we can arrange a displacement of such a kind that although equation (8) may vanish yet equation (7) will not*

These conditions are

Let us
stant.
its

first

suppose that

gP*-\-~ (5- +r,


2
Jtt|

is

not a

con-

Jttg

It is

then a function of the coordinates of the surface and

value at any point lies between a relative mazimum and minimum as the field of variation of the function id a finite
region.
If

be a value intermediate between this

maximum

and minimum, the function

-,P.f(i+i)-K
is

(ID

sometime positive and sometime negative.


Let us give to &n a value zero along the curve

L and

value of the same sign as the expression (11) at other points of the surface. These values can always be found so as to
satisfy (8).

We then
and
therefore,

have

"

<+ >i*
is

Now

by hypothesis $n

zero

integral of Equation (7) vanishes. as expression (II), so that the surface integral ot (7) id not zero

along L so that the line Also $n is of the same sign

as each element is positive. It follows then that Equation (7) cannot be satisfied so that equilibrium is impossible. Condition
(9) is therefore necessary.

Let us
(10),

we

but not suppose that condition (9) is satisfied can always choose values of Sn satisfying (8) and such

now

196

)
is

that along the line L, the sign of

$n

the

same

as that of

<E
The
first

-?)-!.
is

integral of Equation \7)

zero

while the second

is positive.

Equation (7) cannot be

satisfied.

then that Equation (9), and sufficient condiitons of equilibrium.


It follows

(10) are

the necessary

the equation of the free surface of the liquid and Equation (10) shows that the angle of capillarity 6 is constant at every point L of the curve of contact of the free surface

Equation

(9) is

of the liquid and the containing vase (or the adjacent solidX
6. 7.

We now

proceed to give the differential equation of

the free surface of the liquid.

We

have already seen that

if

B lf R a

be the principal

radii

of curvature of the free surface at

any point

or,

a'

(I J\l

+| )(-*)
Jti2

where a'

the positive quantity

B
,

and h

K
y*

From

solid

geometry we know that

The

differential equation of the free surface

becomes

197

Remark

If the

free

surface

is

surface of revolution

generated by the revolution of a curve =/ () about the (vertical) axis of 2, the principal radii of curvature at any point are known to be the radius of curvature of the meridian
section

and the normal intercepted between the point and the

axis of revolution.

The former

radius of curvature
:

is

3/8

and the

latter is

Vi+

Hence n

The
surface is

differential equation of the

generating curve of the free

(a)

6.

m =-?^

...(2)

This result
equation
(1).
II.

is,

of course,

also

directly

deducible

from

Remark

The "Constant

of Capillary forces"
t

?
^

is called

surface tension' of the liquid.


6. 8,

See also

10
Consider

Weight of liquid

rising in a cylindrical tube.


liquid.

a cylinderical tube plunged vertically in the o f the free surface can be written as

Tha equation

198

dy^ dx _ *-k
}

dp ,dq

(1+
'

or,

da
</

.1

/ v

jf

h *- -

or *'

^
rfy
. ,

where U^=

a
fl

,v=

Multiplying both sides of this equation by dso dy and integrating, the field of integration being a cross section of the
cylinder

Now

the

left

Aand side by Green's Theorem

is

easily trans.

formed into a line integral.


It is equal to

where 6
right

the angle between the normal to the curve of the section of the cylinder and there-axis.
is

Hence

ff J J dx

d (~+ dy' dxdy= ~) y

CP ***+<ni*
Jc Vl+p-f j*

the direction cosines of the normal to the free surface and the normal to the wall of the cylinder are reep3ctively r

Now

cos

n e

Hence

if

is

the angle of capillarity

199

+ g*'ni
Accordingly,

/j? 4-ji + j V"l


c
1

cos e

4" g

*****

6ck

*=co* 6

Cds
/

cos

since 9 is constant;

being the length of the contour of a normal

section of the cylinder.

Hence

ff

(z

h)

dxdy

a2

cos 8.

represents the z coordinate of the level surface of the liquid outside the tube. Hence ti) dxdy (z of the liquid which rises in the tube above the is the volume v

The quantity A

ff*

horizontal surface outride the tube.


/.

a2

cos 6.

If angle 9 is obtuse as in the case of a glass tube

dipped in

mercury the value of v


f

is

negative which indicates that the liquid

is

depressed in the tube.

Ex.

1.

Prove that the volume of the liquid that

rises in

the space between two cylinderical tubes of the dipped in a liquid is proportional to

same material

(W) cos
where
Z,

0,

V are the lengths of the contours of tho normal sections of the tubes and 6 is the angle of capillarity for both the tubes
tube of any cross section is plunged in a liquid so that the angle between the generators and the vertical is a. Show that the volume of the liquid that rises in the
2.

Ex.

A cylinderical

tube

is

(sec

a) times the volume that would risa

if

the tube

is

plunged
Ex.

vertically.
3.

large drop of a liquid having a nearly flat top rests on a horizontal plane in the form of a surface of revolution,

200

the radius of the section of the surface by the horizontal the angle of capillarity, prove that plane on which it rests and of the drop is the volume
If r
is

2 ^ra a sin

<t>

+^h

ra)

the height of the highest point of the drop above the horizontal plane and gPa* is the surface tension of the liquid.

where A

is

Let us take the vertical axis of revolution of the solid as axis


of n

and the origin of coordinates

i x

the horizontal plane.


is

We have

seen that the equation of the surface

(*-A)

-a (I -fi Hi

...

...

**

(1)

whero A

is

a constant and a*

R * ,~
zgr

R
so that
-

= surface

tension

~g9a*.
very large and has a flat top the principal be the radii of curvature at the top must be infinite so that if above the horizontal plane, we have height of the highest point
Since the drop
is

ZQ

"

SSS O.

Hence h

is

the height of the highest

point

above

the

horizontal plane.

Multiply both sides of Equation (I) by dx dy and integrate over the projection of the free surface of the drop on the
horizontal plane.

/.//* dxdy
or,

ffdxdy =a' ff

&+
t\\

dxdy

*+*

But

ff

~
K,

+4 K

)
s

dxdy =2

r sin

<t>

(see

6.8)

201

Hence V=2"ra* sin * -f ""hr*.


drop of mercury resting on a horizontal plane is in the from of a surface of revolution. If h be height of the of the drop above the horizontal plane and i the topmost point
4.

Ex.

angle made by the surface of the drop with horieontal, prove


that

2a 2 ,2a 2
r

l-

where b is the radius of curvature of the meridian curve at a the top and r is the radius of the base of the drop, and #Pa
is

the surface tension of the liquid.

We know

that

1-

1 + R =?
a

A/

Taking a horizontal

line

through

asx-axis and the down-

ward

vertical as s-axis,

we have

=^=(-1 a
a

R,

K/
P,

the topmost point are 4, b we have principal curvature

Since

*=o

at

where the

radii of

Set

-=t
a 03

a;

Equation

(l), therefore,

becomes

*L

Sin
a:

Intergrating,

202

to

Now

0=>0 when 2=0, therefore,

fc

-,-.,+ fjWj.+'V f 2a
3
I

a;

y o

a
by
(2).
i,

the radius of curvature at the top is very large, the last term of the above equation is very small and may therefore be neglected.

Assuming

that

Let the distance a of any point of the free surface from the
axis be denoted

by (r+u) where r

is

the

radius of the base.

We have then
u+r
,

2a*
If r

be large compared with a 8

we have

to

the

first

approximation.
:

(1

C08

Sin

or, z

hf *
A

Substituting this value in the integral

I I

Sin

wTr

w-fr

203

*0

tZ!*Jl 4a
Neglecting the
first,

iHF^SV

fl-C08

T^

_
3

last

term as very small compared with the

we have
cos 3

Sin>

'

be the acute angle made by the surface of the drop with the horizontal plane, we hatre $*s'rr ^' 1 at any point of the
if i

Now

base f where

=A

(say)

and w=o.

3r

cos

This gives the height of the drop.

Ex

5.

Prove that the formula


I4

2<z

co8 a

g-t^/
the preceding

8a 8 (1 "*""&"
,

Sin 8

(1
'

Sir.

"?'

gives a better approximation

than the formula given in

exampla
that the radius of curvature 6 at the top of
4
!

Ex,

6.

Show

the surface is given by

204

where r

is

the radius of the section at the base.

drop of mercury rests in the form of a surface of revolution on a horizontal pla, e. Show that if r be the radius
7.

Ex

of the base and

the greatest radius of

any horizontal

section

where /k=R r and t is the angle between the tangent at the top to the meridian curve and the horizontal.
Ex. 8. A drop of a liquid whose volume is v rests in the form of a surface of revolution, show that radius r of the section
of the horizontal plane on the quadratic equation

which

it

rests is the positive root of

1-Sia 3

1
6. 9.

2
vertically

i& of liquid near a plane plunged

in

it.

Let us take x-axis to be such as to be perpendicular to the axis being vertical and If the plane, in the plane. lying plane be so large that we can neglect what is happening at its
the free surface of the liquid assumes the form of a cylinder whose generators are parallel to Hence the y-axis. coordinate z of any point of the free surface is independent of y.
extremities,

The

equation (1) of

6.

7 for

the free surface becomes

205

d*z
dx*

or integrating

=0
1

(*-h)*

count z from the natural leval of the liquid, the plane **o meets the free surface asymptotically and the radius of
If

we

iurvature is infinite*

Hence

for 3

= 0, -

-r-

dx*

ax

so that

f=0andC=

1.

we, therefore, have


i

dx

(2a
?

g2)

V4a a

z*

(D

(2)

Let 6 be the angle of capillarity, then 9 is the angle made by the normal at P to the free surface with a5 axis. If the P=fe so liquid rises to a height k against the plane, then
that

206

k V4a a

/c*

or,

=
V

2'

+ or
If the

eiga being taken according as the liquid rises or is

liquid

wets the plane,

o so that the height to

which

it rises is

Ex.

1.

If a plane

is

inclined at an angle i to the horizontal,


is

the height to which the liquid rises


2 a sin

where g a 2
Kx.
2,

is

the surface tension.


parallel planes are

Two

plunged vertically 01 a liquid,

the angle of capillarity of both the planes being #. If the distance beiween the planes be very small, show that the height of the
liquid

which
tt

rises against

any plane

is

approximately
5 4

cos
i

sin $ cos $ -+

4 ooa~y
is

2 whereZis the distance between the planes and gPu

the surface,

tension.

capillary tube of narrow bore and circular cross Section is plunged in a liquid.^ If the angle of capillarity is radius of the tube r, shew that the free surface of acute and the
3*

Ex

the liquid in of the curve

the

tube

is

the surface obtained by the revolution

about the s-axis, which


circular section.

is

the vertical through the centre of a

Here
r9
3tt
2

I
C0S
8

sin 2

is

surface tension of the liquid.

Let us take the origin in the plane of the level surface.


If

x be the

distance of any point

on,

the surface from the

axis,

we have
d*z

^
i

dz^

p..

^
i

/ V

dz^

ai
J

Multiplying both sides of (1) by

x and

then integrating

with respect to

x we have

r'

dx

J:
suppose that the surface meniscus differs only slightly from the portion of a sphere.
If
r
is

very small^

we may

of

the

The meridian curve


whose equation
is

of the surface will,

thereforef be a circle

and

being constants.

Let us

set

where u

is

a small function of x.
ctz

We

have then

*~

"~~

dx

208

we have then

gj*
%AS

(,
C

ate

ax

.8

/ ^

^.2

\a

.7..

3a 2

neglecting

\uxdx. a*J

Equation (2) then leads to


a:

8a

(3)

Since u must not be large


first

when x

differs slightly

from

c,

the

term must vanish.

,,-Jf + fo
Also for a;=0, 2
is zero,

(4)

so that

c-\-u

**o

The equation

of the surface of meniscus

is

209

If the angle of capillarity^

is

acute

dx
cot

0== :

(1-

c-K'c
Sin
c=

-a*

or

cos0
6. 9,

cos

i(appo*)

Surface Tension-

We

have already seen in


surface of a
in
all

6.

2
to

that

that of a

Thomas Young compared the free membrane eqally stretched

liquid

directions

and

obtained the differential equation of the free surface of a liquid on this hypothesis. Laplace's objection to this mode of treat-

ment of the theory


shall

here indicate

of capillarity has already been noted. how the hypothesis of a uniform surface
results as already obtained.

We

tension leads to the

same

Let assume that the free


state

surface

of the liquid

is

in

of uniform
across

surface

tension

T,

so that the tension on the


is

surface

any

short

length $s in the surface

&s at

right angles to the line &s.

Consider
liquid at rest

now

a portion of the free


act-ion of gravity.

sm face ABOD

of a

under the

Let PQ,

PS

be

elements
of the

of arcs of
at
P.

two

principal sections

surface

the two principal sections OR and SR at the points Q and S. thus obtain a

Draw

We

small area
sider

PQRS of the free surface. Con* an element of the surface bounded by PQRS and of infinitely small thickness 8.
is

It

in

equilibrium

under the

action

of

210

acting across the contour, the weight, and the on the element. (We neglect liquid pressure acting normally resolve all the forces atmospheric pressure at the top). If the pressure at P', a point immePC' along the normal
surface

tensions

We

diately below

isjp,

the total hydrostatical thrust

element iap (PQ) (PS) =pd$ ds', along CT. downwards. element is gP ds ds acting vertically
,

acting on the The weight of the


If the

angle between the normal and the vertical


of weight along

is

ct,

the

component

PC

is

gP ds
If

ds'S cos a.
across PS,

T
is

is

the tension per unit of length


its

the whole

tension across PS, (which acts at


to
is

middle point perpendicularly


the normal PC'

PS)

T (Pb)=lWs"

Its

component along

therefore

cos

QPC'

Tds' cos

(~ 2

= TW

Sin

The

tension across

QR

also gives a

component of the same


is

magnitude; hence the sum of these two components


Similarly the

-^r^~ KI
tensions

sum

of the normal components of the


ar.d

acting on the sides

PQ

SR

is

T^*'K2

Hence the normal component of the tensions acting acroe*


the whole contour

PQRS
a

is

(~+ jp) dsds'. K K,

For equilibrium, therefore

211

cos

Since 8

is infinitely

small compared with the other terms

of the equation, this leads to

a liquid at rest under gravity we have dp This equation will hold through-out the liquid until a Hence we can use it to distance C from the free surface.

Now

for

determine pressure^? at the point


distance e from the free surface.

P',

which

is

taken to be at a

Hence p^g?

(z

A),

h being a constant

We,

therefore,

have

Comparing

this

equation with that obtained in


1'

6* 7,

we

see that the "constant of capillary forces


surface tension

-5 is identical with the

R B

T>

of the liquid.

That

is

why

the constant.

was

called the surface tension of the liquid.

Ex.

1.

Show

that the equation of a liquid film enclosing a

mass of

air at pressure

is

given by

where

T
2.

is

the tension of the liquid film and

TT

is

the external

pressure of the atmosphere.

Ex.

density P and liquid and the substance

partly immersed in a liquid of surface tension t. The angle of capillarity for the

plane plate
is

is

ft

and the

at plate is inclined

an angle

212

a, to the horizontal. Prove that the difference of the liquid on the two sides of the plate above the undisturbed surface level is

ft \ p \p-J

-w
8
ft

We

know

that A x

co,

=4
,

a cos

-- TT

2/3

2a
;
,

cos

where <=^P a 2 or
Ex.
2.

a=

* (

Tj )

volume f ^ c 3 of gravitating liquid and of density P is surrounded by an atmosphere of pressure Tf and contains a concentric cavity filled with air, whose volume at this atmosphe-

ric

3 The surface tension of the liquid is pressure is f TT a in the configuration ef prove that the radius x of the cavity
.
;

equilibrium

is

given by

two soap bubbles of radii r and r are blown, from the same liquid and if they coalesce into a single bubble of
Ex.
3.

If

radius B, prove that


TT

if TT

be the atmospheric pressure, tension

is

B'-t^r
2

+r'

Ra
made
of a material
tensile
stress

Ex.

4.

A spherical balloon of radius r is


It

which can sustain a maximum


(thickness).

of amount

T-r-

contains a quantity of gas of density a at the

213

The pressure IT of the atmosphere on the surface of earth. balloon rises vertically against gravity supposed constant, prove that it will burst when its velocity is given by

where p=k

P.

(Higher

I.

C. 8. 1934)

We
height
z.

have dp

^-gP

dz, for

any point of the atmosphere at

But p

Jc

P
~zr

& dP^-ids
\M

p
or,

^ =

TT

-I*
fc

and P == k

*,- *

the buoyancy of the fluid on the balloon is ^PV, the weight of the ballon being appraximately g ff V, so that equation of motion of the balloon is
at

Now

any height

a,

"

dv

TT

SJ

koe
Since v=o,

g'

when a = o,

^
Now the
balloon bursts

when

214

= ,1(1 -**'*
a
Ex.
its 5.

).

to rest

circular cylinder is

made

on a liquid with

axis fixed horizontally at a height h above the level surface of the liquid. If a be the angle of capillarity and 26 the angle

subtended at the axis by the arc of the cross section in contact withtheliquid, prove that the surface of the liquid near the cylinder
is

a cylinder whose right section


is

is

the curve whose parametric

equation

Also shew that


c cos

- *) = 4T
If

6.

11

Liquid Films:

we

dip a frame work formed by

in a liquid e.g. a solution of soap and water and rigid wires then draw it out slowly we obtain a system of infinitely thin

We proceed of these films. We have to find the surface of equilibrium 6. 6 that the force function, U, of a in system already proved in contact with solids is consisting of a liquid resting
films of the liquid

which are bounded by the wires.

-gfff P* dv + A~
where a
is

<r

+ (E.the

) X,

the area of the free surface of

liquid

and X the

area of the liquid in contact with the solid.

215

we

In the particular case of infinitesimally thin liquid films, may neglect that last term as the area X of the surface of

contact of the liquid

and

solids is proportional
to

to

the thickness
first
it is

be exceedingly thin. The assumed of the film, term is also negligible in comparison with the third because

which

is

also proportional to the thickness of the film, provided, of course,


r>

which
A)

is

the surface tension of the liquid film,

is

not itself
also be

very small.

The secmd term, which

is

a constant

may

ignored as the addition of a constant to the force function of a system makes no difference in the equations of equilibrium of
the system derived from it

We
to

thus tike the force function

in this particular instance

be

p
>

cr.

The

condition of stable equilibrium

is

that

R
or
2i

be

a maximum.

That

is,

the

area a of the free surface must be a


is

minimum.

In the case of a single film, the free surface a


faces of the
film,

whose surfaces are sensibly is very small. Hence in order that the equilibrium of a film be stable, it is necessary and
identical as the thickness of the film
sufficient that area

composed of two

of one of the two faces be a

minimum.

by means of a simple experiment due to Van der Mensbrugghe. Take a circular wire of copper and suspend it by means of three strings. Dip the wire in soap solution and withdraw it gently from the liquid means of strings. A thin film is thus obtained. Form by
is

This conclusion

easily

verified

a loop of a piece of thread and moisten it with soap solution. Place it gently on the film. The thread can assume form whatever. Now perforate the thin film inside the any
loop by means of a pin, It will be found that the loop of the thread is instantly drawn out into a circle c by the contracting

(,216
Of the closed curves having a given perimeter circle has the maximum area. The area comprised between the circumference of the circle c and the metallic wire is thus least when
film.

the liquid
6. 12.

is in

equilibrium.

Liquid Films (Minimal Surfaces).

Let

MB

be one of the faces of a

liquid
If
}

film

and
a

AiMjBj
surface of
differ

a parallel

minimum
at

surface infinitely close to it. and area, the areas A

A MB is
1

MB

A M

B 1 must

by infinitesimals
surface

of

the second order.

Let us draw normals

to

every point of the contour of AB. These enormals intersect the suiface A 1 1 B 1 in a curve L', which
this

encloses an area

a'.

Let a" be the annular area between the


(See
6.

contours AiBi and A'B'.

6)

we then have

a 1= a'-{-cr''
Also

$8=1^

<r=<r'-{-cr"

<r

by equation

(6) of

6.6.
is

The

condition of equilibrium

therefore,

JJ

Ri

+~)SndSR
2

J
$n

Since this condition must be satisfied for any value of


arbitrarily taken,

we must have

Rx
The two
therefore

Ra
principal
radii

and cot0=o.
of curvatures of the surface must

be equal and of opposite signs.

Such surfaces are

217

called

minimal surfaces
*

The second equation expresses the

fact that the angle of capillarity is *

~ Since 2

the

direction

of the

taugen plane to the wire with which the film is in contact is as saying that the surface of arbitrary, this is the same thing
the film passes through the contour of the wire.

The problem of determining the surface of a liquid film is the same as that of flniiag a surface of minimum area passing
through a given contour.

Ex.1.
surface
is

Prove that the

diffsr ntial

equation of a

minimal

8* a~

3s*

3a 3y 3*3.y

!L+
\

Ex.

2.

Prove that a helicoide

is

a possible surface of equili-

brium

for a liquid film.

3.

Prove that the surface of equilibrium of a liquid

supported by the circumferences of two circles whose planes are perpendicular to their line of centres is a catenoid formed by
the revolution of a catenary about the line of centres

Ex.

4.

Show
is

that a surface

whose mean curvature


liquid
films.

is

zero

every where

a surface of equilibrium for

Prove

also that the equilibrium is stable

(Schwarz)
oil.

1 3.

Euqttibrtum of a drop oj

Let us consider the equilibrium of a drop of olive oil trapped between two wires of any shape or two circular discs) and inserted in a mixture of water and alcohbl of the same density
as
itself.

In order to fiLd the condition of

**q

lilibrium

it

is

sufficient

to determine the condition that the force function of the

system

218

formed by the

oil

Now

the forces

capillary forces
liquid

minimum. that act on the system are the weight and the of the molecules The centre of gravity of the
and
liquid is either a

maximum

or a

and

oil

does not change whatever

may be

the displace-

ment of the oil drop since the surrounding liquid is of the same Hence the work done by weight ia any virtual disdensity.
placement
i

zero

It

is

therefore not

necessary to take into

account the external force of gravity.

The

capillary

actions

are

of

several

kinds.

They

are

attractions.

(1) of wires (2) of oil


1

on the
itself

oil,

on

3) of the mixture on

itself,
oil,

(4) of the mixture on


(5) of the mixture

on the wires

We neglect the

attractions of the material of the containing

vase on the liquids which it contains because they do not figure in the variation of the force function when oil drop is slightly
displaceed, provided that the oil drop is always at a great ared with 8), distance from the walls ot tho vase.

comp-

Let us denote

!>y

a=the

surface of contact of the oil


surface

and the mixture,


and
its

\i=the

of contact of the oil

solid supports,

X=the

total surface of the solid supports,

the function relative to the mutual attractions of the

molecules of the

oil

E! =the
'

function relative to the attractions of the molecules of the oil and the solid

supports
for the

BX',

E, corresponding functions

mixture and

Ef

the

function relative to the attractions of the oil

and the mixture.

219

We have for the force function resulting from


actions enumerated above,

the capillary

(4)

B. a

(5)

ET'^-XJ,

neglecting constant quantities.

Hence the
a constant),

total

force function for the

system

is,

(except

,~
Now
since the

--) a+ (E -+-..-.El
1

')

X,

framework of wires
therefore take

is solid,

the surface

remains constant.

We may

as our force function.

For equilibrium, therefore,

Comparing

this

with equation (1) of

6.

giving the

condition of equilibrium of a liquid in contact with solid walls see that they are of the same form. They only differ by the values of the coefficients of cr and x x and by the disappearance

we

of the term relatiug to weight Following the same reasonings as there, it is easily seen that this condition of equilibrium leads to two conditions viz.

220

const, and

0= const.
Hence the mean curvature of the surface of separation of the oil drop and the mixture must be a constant and the surface must cut the solid supports at a constant angle.
6. 14.

If the surface of equilibrium of the


is

drop

is

a surface

of revolution the meridian curve


conic

rolling on a straight line. of oil be circles whose planes are perpendicular to the drop line joining their centres, the surface of the oil drop is evidently a surface of revolution, the line of centres being the axis of
revolution.

the path of the focus of a If the two wires holding the

be any point on the meridian curve and let xy be the axis of revolution. Draw Fc the normal at F intersecting

Let

xy

in

I.

Then

it is

well

known

that the principal radii of cur-

vature at

and FI.

are FO, the radius of curvature of the meridian curve, Since the mean curvature of the ^surface must be con-

stant in order that the drop be in equilibrium

221

proceed to show now that this condition is satisfied in the case of a curve generated by the focus a of conic rolling
Consider an ellipse whose focii are F,F/ The point of contact I is the instantaneous centre of rotation,

We

along a straight
therefore,

line.

we have
Velocity of Velocity of

F Fx
given straight line xy, on This point will have the same velocity
in the

Let

G be the image of F x
ellipse rolls.

which the
as

F1

Also by a
so that I

known
G.

property of the ellipse

will lie

on

I,

F^I
'

Hence lience

v elocityofF
velocity of

_ IP IG
to

'

m U)
this

Through a point F' infinitely close point, draw a normal to this curve. Let
then this
is

on the locus of

this

normal meet FI in

the centre of curvature of the curve, for

FIG

is

also

normal

to the curve

Now FG-FI

-f

IGFI+IFi
ellipse

= major axis of the


Hence the
locus

const.
to that of F.

of

is

a curve parallel

Consequently the two infinitely small triangles are similar,

FCF' aud GOG'

222

FF ~ __CF
GG'

CG

Velocity of F Velocity of G

= CF
IF

CG

Hence the
so that

four points F,

I,

and

form a harmonic range,

11

2
-i^^r

== const.

Hence the condition

that

must hold

in

the

case

of the

meridian curve of the surface of equilibrium is satisfied by the This curve is known as Ondulotd*. Similarly it locus of F.
can be shown that the locus of the focus of a hyperbola rolling on a straight line has the same property. This curve is known
as nadoide.

In the case of a parabola, the locus of the focus

is

such

that.

if

FC
is

==a

Such a curve

a catenary, with the straight line on vshich


its directrix.

the parabola rolls as

Ex.

1.

drop of

oil rests in

a liquid of the same density

as itself without being in contact with any solid supports. that the surface of equilibrium is a sphere.

Prove

[Hint
to

X = X 1 =0,

so that the condition of equilibrim reduces

$a=o.]
Ex.
2.

If the drop in the above

example

is

made

to

rotate

about an axis passing through its centre with velocity cu, prove that the surface of the drop is a surface of revolution whose

meridian curve

is

223

where

*
c

t/+

5
,

=
|J
>

being the

surface

tension and 0,X arbitrary constants.

be the free surface and a* a neighbouring surface obtained by an infinitesimal deformation of a. The work done
cr

Let

by

capillary forces in a virtual displacement

is,

as

we have

seen,

E f f*
where

*crf

is

the surface tension.

Again, the rotation of the drop sets up centrifrugal forces

whose work in any

virtual displacement is

where I
rotation.

is

the

moment

of inertia of the drop about the axis of

Hence

for equilibrium

SI
So

T *<ro

or,

=.

$1 ss a

On

account of the deformation of a to an element ab ==

of this surface, there corresponds an element a'V dS' of the surface obtained by drawing normals along the contour al to 9. The variation of a is

d8

224

The

variation of I

is

the

sum

of the
a'b'*

moments of

inertia

of

the elements of volume such as ab

If r be the distance of

the centre of gravity of this element from the axis of rotation

we have

.".

The

condition of equilibrium

is

dS
for

*
(

K!
every Sn such that

+1
%

~r') = o
>

Jtla

ff Sn dS
which expresses the
This
is satisfied if

fact that

volume of the drop does not change.

5
Jtvi

^"w

*= const

ft

rtj

If this surface is a surface of revolution,

of curvature are
curve,

MO,

the principal radii the radius of curvature of the meridian

and MN.

Hence
,

as

-r
C050

This gives for the equation of meridian curve

as

y
axis

taking the axis of revolution as x

dy =5
,*.

sin$ ds

y sin $ dt

+ cost dy = (a y* + Py) dy

42

225

being an arbitrary constant.

or, cos

0=

y
/ tan

or,

J
A

dV
V v
1

drop of liquid under no forces except uniform external pressure and surface tension rotates as a rigid body

Ex.

3.

about an axis.
tant,

Shew

3
that

1
=r- ) is
x

on the surface (-5 K

cons-

Kj|

R x R,
,

being the principal radii of curvature of the surface,


Closed liquid films.

6. 15.

Soap

Bubble.

dip a network of wires in a liquid we obtain in certain cases a system of liquid films, which form a closed surface
If

we

shall now show that the surfaces enclosing a mass of air. of the equilibrium of these films are the same as the surfaces of

We

equilibrium of a drop of oil plac&d in alcohol of the same density.

a mixture of water and

Consider the system formed by these films and the mass of air enclosed within. Imagine a virtual displacement of the

system from

its

equilibrium position

The

virtual

work due

to the

weight of the system

may

be

neglected, because the mass of the enclosed air and the mass of the liquid forming the films are very small.

Let a be the
with external
air
is air.

total area of the faces of the

films

in

contact

The area
cr f

of the faces in contact with internal

as the two faces of any film being very sensibly near each other can be regarded as parallel. The virtual work
also

of the capillary forces on account of a variation area is

fcr of this

226

B!, being the function relative to the action of the molecules of the liquid on themselves. If at the same time the area of contact
of the liquid and the framework varies, we shall have to consider the virtual work of the capillary forces resulting from this But this surface being proportional to the thickness variation.

of the films
therefore,

is

very small and

may
Sa

have only

Bl

therefore be neglected We, as the virtual work of capillary

forces due

to the displacement considered

To

this

we must add

the virtual

p be

the pressure of the air works of the pressures due to a displacement of an the Tl$n dS respectively. elementary area dS are p$n dS and
virtual

work of the pressures. If within and TT that of the air without,

The condition of equilibrium of the system

is

B x Sa
But we have already seon that

J
?"" 11 "^ IT
Since this
is

ftn
L

cattds.

+ ^ B| ]* n ds ~~ B
whatever
-the

'

satisfied

deformation

we must

have
)

BU

IT

cot

0=0

or

^-rr*
TT

being uniform, (since the weight of the air has been neglected), the first condition shows that the mean curvature of the surface is constant. This is oi*e of the
pressures
conditions of equilibrium of a drop

The

p and

of

oil of

the same density

227

as the

mixture in which

it is

inserted.

The

condition

0=

~ &

= appears more restrictive than the analogous condition of constant found for the drop. But in reality both the conditions are
identical in

express the fact that the surface of the films or of the drop must pass through the wires of the
as

much

as both

frame work.
In particular if we form a soap bubble passing through the contours of two parallel wires of the same radius we must obtain for figures of equilibrium, onduloide, or a right circular
cylinder, or a nodoide, observed by Plateau.

or

a catenoide,

This has been actually

Ex.

1.

Two
a

soap bubbles of radii

and R'

intersect

one

another in through which passes a thin soap film. If the film forms part of a sphera whose radius is. R", prove
circle

that

TT
Ex.
2,

R^R

7'

Two

circular rings

with a

common
-

axis at right

angles to their planes support a closed liquid film containing air


at a greater pressure than the external air;

shew

that the ends of


surface

the

film

are

spheres of radius

T a=s2*
P

,and that the

bet/ween the rings is a surface of revolution of which the ridian curve has an intrinsic equation

me-

wherft $ is the inclination of the normal to the axis is the distance from the line of centres of the rings.

and

a;

Ex,

3.

A liquid film of total


-parallel

surface tension
discs

is
r,

by two equal

circular

of radius

supported with their

centres at a distance h apart on a line perpendicular to their If a pin hole is planes. The film is in the form of a cylinder.

228

made

in one of the discs so that the air slowly escapes,

shew

that a total quantity

will escape,

where P

and

TT

are atmospheric density and r


c

pressure and c

is

given by cosh

--nr"

initial pressure the cylinder be jp, so inside

Let the

that

?
The mass of the
2
"

(1)
air

in

r the cylinder is P, (h) P being the density of air Since inside the cylinder.

n-^r
18 7T
<*

therefore

mass of air

Now if a hole be made in one of the discs, air will escape the pressures inside and outside are equal and the liquid film then assumes the form of a catenoid, that is the surface of
till

revolution formed

by revolving a catenary about

its directrix,

which in
discs.

this

case will coincide with the line of centres of the

If V Q

be the
air

volume enclosed in the


is

liquid

film,

the

mass of the

remaining

*>

density of air being

now

the same as that of atmosphere.

The mass of air escaping


TT

is

r P oo

229

orby(l)

We have

now

to

calculate

the volume v

of the surface of

volution of the catenary

u=c
about

cosh
c

OB

its directrix,

which
h
i

is

the line of centres of the disc.

Since tha point

B
5 %c

m (-~, r) lies on the catenary,

r=c cosh

or,

cosh ~2c

v
,

whch gives

c.

^o=2

A 2
<

= 2-

2
c
2

cosh

Hence by
Ex. 4

(2)

we have

the required result.

A wire in the form of a circle of radius a isplaced in the

surfaceofsoapy water and raised gently, so as to draw after it afilncu Prove that neglecting its weight, the meridian section of the

230

film

is

a catenary. Also prove that the parameter of the meridian

caternary,
% is

when

the area of the film

is

* a* equal to

is

~ where z

given by
-i
a

cosh

r=z

6. 16.

Equilibrium of a drop resting on liquid of greater

density.

Let
a'
it

the

ABOD be the drop. surface ADC in contact


Let
-

Let a be

its free

surface

ABO

and

rests.
'

cr"

with the denser liquid on which be the free surface of the liquid itself. Let

R R' B" ^JEL

222

be the surface tensions of these surfaces.

The work
is

done by capillary forces in a virtual displacement of the drop

density of the liquid and PA that of the drop. Let V, Vi be the respective volumes of these liquids and Z, Z t from the xotj the distances of their centres of gravity have for the virtual work of the weights of the plane; we

Let P be

the

liquids

(gP

Z + gP^ VZ

),

axis being supposed to

bedirected upwards.

The

condition of equilibrium

is,

therefore
'

The

equations of constraint are

and

tf.V'^Oj

since the volumes of the liquids do not change.

231

In order to transform these eqnations, let


the
If

S S/
,

" Le

new
at

positions of

<x,

or/

a".

each

point

of

the

original

surfaces

we

draw

normals to these surfaces the


lengths

Sn

of these normals

comprised
surfaces

between

these
result

and those which

from their deformations will

determine the
of a,

latter surfaces.

Let dS be the area of an element

we have then
n
dscot

ds being an element of the curve of contact and ^ the angle between the line OF, joining the two positions of a point of this
curve with the tangent at the triangle GHF, where OH
sensibly a right angle.

F
is

to the

deformed
<r,

surface.

Now

in
is

normal to

the angle at

We,

therefore,

have

OH
f

= OF sin HFC
sin<l>.

Sn^&n*

$n being the normal


.'.

distance between cr"and


'

/;
.

* cr=

SS &n(
shall find

ds cos

Similarly

we

-, +-7 dS+S M ds cos


'

g^
is

dS"+

f M ds cos*"
cr

For calculating Vi we remark that if the surface deformed, we have for the variation of volume

alone

ff Sn dS.

232

But since the surface

a' is also

deformed

we have

$V a **ff$n d&
Similarly

ff Sn dSf
+ fftn d3".

*V

~Sf$n

</S'

The volumes being

constant

V *Z=
The
last

members of these equations express the fact that the variation of the moment of the liquid with respact to xo y plane is equal to the sum of the moments of the weight of their elements with respect to the same plane.

The equation

of equilibrium becomes

ff tn

dS>[

Pz-

/B
Sn'dS'
(

cos

+ B' &
are
^7C!// W ab

-5 cos4>'

+ B" ^ *

cos<t>")

=o

(1)

The equations of constraint


/* /*f>

ff$n d'-Sf$n rfS' =o M / /*^^.


^/

oft tto

^JCJ/

_l

/0 v

\~

JJ

I ^5/

Equation (1) must be


(2).

satisfied for

any $n

satisfying equations

These are

satisfied

by taking rfS"=o
(3)

Hence

(1) leads to
( v

-^-H

tf

233

and B

cos

+ B' cos 0'


a 1 a" being
,

+ B"

cos

#" =o,

constants.

In order that equilibrium may be posible, equation (1) must be satisfied whatever be the displacement provided that it is It must, therefore, be satisfied compatible with the constraints*

even when the conditions (3) are not


(2) are.

satisfied

provided equations

But taking into account equations


(l) reduces to
of

(4)

we

find

that equation

or,

(+a'-a")

fftn dS=o,
may
be satisfied even when
the

by means of the equation

(2) of constraint.

In order that this equation


integral

J*J*n dS is

not zero,

This

is
.

the

new

condition

that

must be combined with

equations (4

The
surfaces

first

three equations of (4) give

the

equations of the

a,

a',

a" respectively.

the fact that the


point

sum

fourth equation expresses of the projections of the tensions at the


line

The
is

on ths straight

OF

zero, since 0,

0',

0" are the

makes with the tangents through F in the normal plane at this point to the curva of intersection As the direction of OF is arbitrary, this implies that the three are in equilibrium. tensions at
angles this straight line

(
6. 17.

234

Pressure on a solid of revolution immersed in a

liquid.

Let

AaB
as

be a solid of revolution whose axis


vertical.

is

AB, which

we

take

We

propose
solid

to calculate the pressure

on the

when

it is

in equilibrium in a

liquid

of density P. To do so let us give to the solid a slight displacement


8 such that

the torm

of

the

free

surface

Ma

remains unaltered, so that simply prolonged so as


its

this surface is

to meet the solid in

new

position

A'a'B',

Let us now write the sum

of the virtual works of all forces

The

virtual
6.5
...

work of the

already by

B-n
ftfT
I

capillary

forces

is,

as

we know

'TT

where 8 a
and $X
liquid.

is the variation of the free, surface of the liquid the variation of the surface of contact of the solid and the

Let r be the radius of the parallel section passing through We have then the curve of contact a ft.

toct'

the original position of the point corresponding in the displaced position of the solid.
ati is

where

Also since

R
and(

R
~
)

are related together by

where*

is

the angle of capillarity,

(See Equation (10) of

6.7)

235

Vre_-| *

(2"r) (aoj cos 6

aa')
the projection of the contour

Now
CL
/

the term (ac^cosfl


f
.

da

) is

aa l on \cta This is accordingly equal to the projection of the vertical a j a' on aa', But a ! a' =* A A = e. Hence this expression
7

the angle between a^a 1 and aa' which is equal .to the angle between the tangent plane at the point a to the free surface and the vertical.
is

equal to

8 cos

<t>

where

is

Hence
Let us

T =^Br
c

8 cos

&
work due
to the weight.

now
is

evaluate the virtual


in equilibrium, its

weight is equal and opposite to the pressure exercised by the liquid on it. Denoting it by X, the virtual work of the weight due to the displacement considered
Since the solid

must be
X6.

We

have now

to

compute the virtual work due


liquid-

to the

weight of the surrounding


the variation of
to

This

is

evidently equal to

moment

any horizontal great distance from the


is
it

of the weight of the liquid with respect plane e.g. the free surface of the liquid at a
solid*

Assuming

that the virtual dis-

placement

such that tha free

surface of the liquid remains

unaltered so that
the variation of this

just extends to the solid in its


is

new

position,

moment
of

Tw =

Moment

eBB'+moment Of ELe-f moment of aa'EL


of

~
LEHH',

Moment of LHeB'-f moment

EH0B+ moment of aa EL
;

Hut moment of

LHeB'^ moment

of

EU'eH'* moment

of

where
plane

EE

intersection of

drawn through E, the point of the meridian curve of the solid and the horizontal
X

is

the

vertical

Now

the volume LE'

HH' differs
H' = 6;
it

little

circular cylinder of height

is

from that of a right therefore an infinite-

236
its

simal of the
izontal

first

order and

moment with
of

respect to the hor-

plane

ELH

is therefore

second order.

We

can

accordingly neglect

its contribution.

Tw =
ofaa'EL.

(moment of E'HWmoment

of

EHeB) + moment

But E'H'^B'
through
8,

is

merely volume
of their
8

EH0B

the

difference

moments

displaced vertically is equal to the

product of the displacement these volumes,


/.

by the weight of any one of

Tw=~- 8</P
now

Vol EHeB-hnoment of aa'EL.

It

remains

to evaluate the

moment

of

volume ffa'EL.

The

triangle aa'a", being aa infinitesimal of second order, the volume generated by its revolution can be neglected in comparison with that generated by the revolution
surface of the

of area aa'EL, so that

we have

moment
or f

of

aa'EL=moment
aa'EL=moment

of

aa"EL
a /7 LF, aa" with
of

moment

of

of

aEF moment

the point of intersection of the vertical the horizontal EH. If we prolong this vertical length to FF' so we obtain a triangle a"E'F' generating a volume that
is

where

FF'e

whose moment

differs

from that of the volume a''LF by


\

in-

finitesimals of second order,


/.

moment

of

aa'EL == moment

to

aEF

moment

of a"E'F'

the difference between their

The volumes generated by aEF and a"E'F' being equal, moments with respect of the same
6

horizontal plane is equal to

volume and a quantity moved.


/.

the produQt of the weight of this by which the centre of gravity has

moment

of

aa'EL

tyP.

Vol a EF.

/.

Tw =

60P Vol

EH0B-M0 PVoL (annular aEF)

337

Since the

sum

of the various virtual works is zero,


cos *

we have

Tc+Tw +XiBr
or,

g9e (Vol

EHaBVol aEF)

X=

"Br

cos

0+0P

(Vol

EH*B

Vol aEF),
solid*

giving the vertical thrust of the liquid on the


If the surface of the

liquid

were plane, the vertical thrust

has the value

g9

Vol

We see
larity
is

then that

the vertical thrust

acute, the capillary forces diminish acting on the solid. If the angle of capilif
is

obtuse, then

so is angle

<t>

in general.

Moreover

it is

easy to see that the term g9 Vol aEF must take the positive sign when the point a is below the horizontal plane.

Hence the

capillary

forces

sometime enhance the


it

vertical
if its

pressure exerted on a solid and thus enable density is greater than that of the liquid.

to float

even

Ex,
its

1.

Prove that in the case of a cylinder floating with

axis horizontal, the vertical pressure is given

by

B
where
Ex.
2.
I

cos <t>+gP (Vol

EH0B-2

Vol aEF).

is

the height of the cylinder.

Prove that conditions of equilibrium of a floating

needle are.

Sin (0

0+tfP

(c

0+c 2 Sin
*

cos

$-2

he Sin0)*=t0
a

and
where

Sin

f)=Pc

cos

-A

angle of capillarity, w is the weight per unit length of the needle, c its radius, h the height of its centre above the natural level of water and 20 is the angle subtended
i is the

at the surface

by the curve of contact

238

free

Here the angle made between the tangent a T at a to the surface of the liquid and the vertical through a is clearly

(-

0+*)where< oOP=*

Hence by Ex.
B/ COB

I the

vertical

pressure on the needle

is

JL

*+*)+tfP (Vol

EPH2 VolctEF)

=
Now
Vol.

El Sin (*-i)-f0P (Vol


{

EPH2 Vol oEF).


0)
}

EPH=Z
Z

(20)- 4 * (2c Sin


ch Sin
{

(c5

),

whereO =
}

<EOP.

Volume aEF=^Z

area

aEF

=J

area of sector

oOE

area of triangle

OEN

-~area of triangle

oFN

=i

4c" (6

^)P
^

(c

Sin 8 -h tan a)
A> (c Sin ^

(c CO80-

A tan 0)

where
.*.Vol

N is the

intersection of
{

Oa
8

with EH.
2Ac Sin *
}

EPH-2VoZaEF-Z
vertical pressure

c*

a-fc

Sin ^ cos

The

is,

therefore,
{ <tf-l-c

K Sin (0

*>+0PcJ

Sin * cos 0-*2A Sia0

we have, then,
B Sin

and

this

must be equal

to

the weight of needle viz

tel

(0

t')+Pc

c0-f-c

Sin^ oos

2A Sin

<t>

} =to,

239

which

is

the

first

condition to be proved.

To

obtain the second condition

we

refer

to

6. 9

from form

which we obtain the equation of the


*

free surface in the

)*
where z
is

measured from the

level

surface

and a*

g
^rp
.

If ty be the angle made by a tangent to the free surface with the horizontal, we have

*?

ax

= tan *,
surface leads to the relation
2

so that the differential equation of the

z***2a* (1

a sin costy **4a

=-

Now

at the point a, ^ =
.'.

(c

cos0

A) cuid

0,

(o

co50-

a3

_ Ex.
3.

2B

i'-* -its

needle floats on water with


o>

axis in the natural

level of the surface; if

respect

the specific gravity of steel with to water, i the angle of capillarity, and 2er the angle

be

subtended at the axis by the arc of a cross section in contact with water, prove that
--i' ) ss cosa

co

CHAPTER
The Figures of
7. 1.

VII

Equilibrium of a

Mass

of Rotating Liquid.

chapter the shapes assumed of rotating liquids which are acted upon by no other by masses forces save their own gravitation f confining ourselves to the eimplest case in which matter is supposed to be homogeneous

We shall study in this

Even this simple case is complicated and incompressible. enough to have taxed the ingenuity of some of the greatest
mathematicians.
7.

History of the Problem:

that the law of gravitation must account He showed for the figure of stars as well as their movements. that the centrifugal force combined with its weight must make

Newton had seen

earth an ellipsoid of revolution flattened at the poles. He calculated the ellipticity of the earth assuming it to be of Maupertuis studied the equilibrium of a homogeneous density.

plane figure turning round an axis normal to its plane, on taking But it account of a central attraction and a centrifugal force. mass of was Maclaurin who in 1742 applied to a homogeneous
the equality liquid in rotation the principle of Pascal regarding of pressure even before the principles of hydrostatics were firmly
the ellipsoid of revolution is a Simpson in 1743 studied the possible figure of equilibrium. He also conditions of existence of these ellipsoids of Maclaurin
established.

He showed

that

on taking discovered an upper limit to h*= o 7rx p


velocity as a parameter.

tu,

the angular

This expression, where P is the density '2247 for and X, the constant of gravitation, must be less than
ellipsoidal equilibrium.

book Figure de la Terre (1743) gave to these results and demonstrations their definite form.
Olairaut, in his remarkable

241

He

further extended the calculations to the study of a heteroge-

He gave neous mass, as the earth and f lanets actually are. formulae connecting the ellipticity and the weight with the density and centrifugal force in the case of a sufficiently slow
angular velocity.
solution of the

In

1776 Laplace remarked that a general

that only particular problem was impossible and

cases could be studied

and purely

einpircal solutions given.

method Legendre made known his fundamental which permitted one to search directly the figures of equilibrium It is this method that is of in the neighbourhood of a sphere.

little Idler

primary importance in Mathematical Physics and to this Poincare is due most of the recent progress in this Problem,

and Liapounoff have generalised this method of spherical harmonics and applied to ellipsoids by using Lame's functions
instead of spherical harmonics.

was believed that there were no other figures of equilibrium except the Maclaurin ellipsoids and the It was in 1834 that Jacobi showed ring round a central body that an ellipsoid with three unequal axes turning round the
For a long time
it

least axis is also a possible

form of equilibrium.

Liouville

gave

Otto Meyer gave a complete disas the cussion of the equations obtained in this case by taking
this.

an analytical proof of
variable.

In a celebrated paper which appeared in Acta, Mathematica, 7, 1885, Poincare discussed the problem of figures of equilibrium in a more general iranner. He pointed out that the possible

depending on a single that to each parameter, such as the angular velocity, and such value of the parameter corresponds either one and (only), one or else a finite number of figures and that these figures
figures of equilibrium form a linear
series

Among vary in a continuous manner as the parameter is varied be others who have specially worked at the problem Jeans, Appell, mentioned Liopounoff, Darwin, Lord Kelvin, Globa Mikhailenko.

242

For a more detailed account of the


subject

latest

researches on the

Chapter I of Paul Appell's 'Traite De Mecanique Rationnelle" tome quatrieme, from which the above historical account is taken.
the reader
is

referred to

7. 3.

Mass without
shall first take

rotation.

up the prpblem of determining the form of equilibrium of a mass of liquid whose eleinen^ attract according to Newton's law and are at rest
Let P (a constant) be the density of the
the free surface, supposing the equilibrium to to the liquid mass. equations of equilibrium
liquid,
exist.

We

Let S be

Apply the

We have then

/.

p-

PV+cons.

On

the free surface, S,

we have ^=0, hence

at

every point

on 8 we must Lave

PV=cons.
That
is S,

the free surface, must be an equipotential surface.


at

We
An
sphere

arrive
(jp
)

the

same

result

even

if the

pressure at the

free surface

be not zero.
is,

evident form of equilibrium

therefore, a

sphere; since

is also

an equipotential surface
is

The sphere
solution?
trated.
It is

To

a solution of the problem. But is this the pajy probable ; but this ,cannot be rigorously demonsprove this it would be necessary to stow that if tl\e

potential of a

homogeneous mass bounded by surface S

is

such

that

itself is

an equipotential surface thon S must necessarily b

a sphere.
7
4.

General Equations.

Suppose a homogeneous mass of a liquid of density P rotate round Z axis with angular velocity w. The conditions
relative equilibrium are given by the equations

<

the pressure at Since P gravitational potential.


is

Here p

is

the point (a?,y,2,) and a constant

is

t.b

or

^-

pressure must be zero (or constant) over the ire Thus the necessary and sufficient condition that an surface. configuration shall be a possible figure of equilibrium for

The

homogeneous

liquid of density P rotating with

angular

velocit;

is

that

shall be constant over its surface,

V being

the gravitations

potential of the mass.

Let us now apply guratkm given by


7. 5.

this result to

the ellipsoidal

confi

244

The necessary and


with angular velicity

sufficient

condition that ellipsoid (1)

shall be a figure of equilibrium for a

homogeneous mass rotating

is

that

shall be constant over the boundary.


/*-)

Now,

V =^VP
s

abc

(T~+^Er+Ti
o

J
where

^/T

A=(a

+X),

B(5
d\^
ZA

+ X), C=(c2-fX),

A =(ABOj.*
d\ AZX

SetJ^-^xPabcl
I

JA^^P

abc

f
1

00

w/ O

^
J C =XP abc
I

Hence

2
(aj

JA+y J B+22J.

J)-i

shall be constant over the surface (1;.


It follows that

=c 2 (J c

(2)
(2')

(3)

Equation (2) can also be rewritten


CO

XdX

This equation
(ii> if

is

satisfied

in

two ways viz

(i) if

=b

or

00

*
2

= C
J
.

xrfx

x P a6c

A(a +X)(6+X)

245

7. 6.

when

= b we see that an
00

Maclaurin'e Spheroids.

.Taking the

first

case

viz.

ellipsoid of revolution- is

a possible

form of equilibrium provided

is

given by (3\

a*w* *\Pa 2 c

_f
J
oo

the integral on the right hand side of this equality c. necessarily positive, w is real only if a

As

is

>

Thus only an oblate spheroid


brium; a prolate spheroid
is

a possible form of equilinot a possible form of equilibrium.


is

In order to evaluate the Integral on the right hand side of


(6

we

write
00
x

00
y ^y \

i sJ

CO

00

00

where a a
00
/

c a=s c*Z a

Now,

Differentiating with respect to

00

f J

*adu

1
a

(a+
l

a
cZ

"*

el.
f

a'+ c 8

Hence replacing
2<fo f J c'Z+

by

we have

,
3
)

cZ 8

cot C

v
z

_L /
c*l*
00

ta an

~""*
I

1+

Simlarly,
cx>

-2

Hence equation
o
a
.

(7)

becomes

(3+Z
where

a )

a*=( 1-K 2
if

)c*.

given, equation (8) determines the elliptHty Such spheroids are called of the spheroid of equilibrium. Maclaurin's spheroids.

Thus

w be

7.7.

Discussion of equation

(8).

Set
to*
I

Since tan

_
Z

Z8

-~ 3

7*
>

-|-

for small values of

?,

we

have from (8)

A=

1
.

Hence

for

Z=o, A=0, and


is

for

increasing h >o.
j*

When

tends to infinity, h

of the order

and, therefore, tends to zero.


I

Thus h

is

zero between these two limits of

and increases in the

beginning.
Differentiating h with respect to
I

we have

The sign of ^values of


19

is

the

same

at that of

(I).

For small

we have

135

The derivative
infinity ty(Z)

is at first zero,

then becomes positive.

At

reduces to

^and the derivative becomes negative.


(Z)

Let us now take the derivative of/

with respect to

This expression

is at first positive,

zero for

V3,

and then
1st

becomes negative
therefore, zero for

for ever afterwards.


I

The
I

expression f(l)

=o, increases

till

V3

when/'

(Z)

848

becomes zero, then begins

to decrease for

>

V3 and tends
2

to

&

Hence /(Z)

=o has

one and only one

root,

>V3.

It

will, therefore,

be the same for

Let us designate by
It is

this

v?due of

for

which

=-

and

y(Z) are zero.

given by

We find

2*53...

and the corresponding value A

of h

For

i!

<

we have

^
Zo;

>o;
A,

the

function

A,

therefore,

increases; for

>
2

1C9 --?

dt

therefore,

decreases ahd

is

maximum

for

Taking two axes Oh and 02 we represent the curves

curve
!?4

touches 02 at 0, ita ordiaate inqreasee


Z"*Z =2-53...
;

till

for

then

it

approaches OZ asymptotically.

249

For a given value of h <. 224 there correspond two values


of I

Hence

as

A**-

gTT^Jp

5' for a

given

w
7

there are two possible

ellipsoids of revolution,
**
.

w
provided
there

<

'

224.

If

<

*224,

are

two spheroids of Maclaurin; r

if

??-g

==' 224, there is only

one spheroid of equilibrium;

if
-

<

*224. there is

no

ellipsoid of revolution, r

which can be

a figure of equilibrium.
Maclaurin's spheroids cannot be a form of equilibrium
velocity of rotation
if

the

becomes greater than (.448 ^p)*'


or

Remark
are

(i)

If the celestial bodies like the earth

planets

Maclaurin 's spheroids, these must correpond to the lower For if l> 2.53 root ofZ, assuming the ellipticity to be small
ellipticity

VI

+4

and

6 is

known

to

be much smaller than $ in the case of

the earth and planets.

Remark

(ii).

Ellipticity of the earth.

We have

24X60

'

" 4X3.14X40,000,000
242 X6
=.00227.

2fO

For

this value of h,

we

solve the equation

-.
The corresponding
ellipticity 6

and take the lower


.

root.

is~r
231

This value

certainly
ellipticity
is

disagrees with the geodetic

measurements.

The
1

given by Helmert in
It is

901

is

^and by Hayford in
if

909

~~~

necessary to

conclude that

the

earth

is

an

ellipsoid of

revolution

and

if it

was

fluid

sometime

it is

not homogeneous.

Remark
a sphere.

(iii)

when w tends

to zero,

one of the values of I


is

tends to zero, and

the corresponding spheroid

accordingly

The other

If I is very large, increases indefinitely. of the liquid to be finite, assuming the mass

we

have,

=-a-c=--- _
,

(1-K?)*

Hence
It

c~f| ^anda-

follows that c

S>

o and a

-~>

infinity.

We

have

a figure resembling a disc of infinitely large radius and infinitely small central thickness.
?. 8.

Jacobi's Ellipsoids.

Let us now take the second alternative of satisfying.


equation (4) of
7.5

From

equations

(2')

and
+

(3) of
a

7.

we

also

have
(10)

c(a

5V J e

251

or, (6"

<

Since J 5^ a,

we have

to discuss

now

the equations

Too

>*

and,

r
2

2 o &

c
a

2
.
'

J
To reduce

(a

+X)

(6 -f-X)

c?X ,,, Q

+X
a

these integrals let us set

Xc

a>

we have then

where

(o;)=V(l+sa;) (l+to)
/.a

(1+^X and

.,2

And equation

(13) becomes

These equations give 5, t when we are given w. problem is to reduce these equations.

The

NOTE

* s 1)> since equation (16) it is evident that (1 both the integrals must be positive and s, t are also positive.
:

From

Hence s-f t< 1.


therefore
or, c
.*

Since

s, t

are positive,

$<l,t<l <a andc*<i 2


2

c is

the least axis.


it is

For equilibrium
be the least
7. 0.

necessary that the axis of rotation must

axis.

Discussion of equations (14) and (16).

00

xdx
CO

Aleo

,\

A B Ml**) JC~2\3
t\

<fo*

5*

C 3?dX 3 J

'

We proceed to show that ^=o has always a


one root in s
for
,

root

and only

every value of t and that the expression h has one for every value of / or s, with always one value and only
its

maximum
There
is

for 3*= *=<<,.

in every case one ellipsoid of Jacobi as a figure

of

equilibrium.

These two equations define a curve of space by means of


the three variables h, *, t. point P of the curve projects into a point R pn the -st plane. The coordinates (5, t) of the point

B will clearly satisfy


Hence
to trace
to

%,*)

o.

tr&fefe tlfe

above skew eurve


tfce

it

is

only necessary

^ (,)

o on

r plane and then to emit at

of this curve an ordinate RP equal to the corresponding point value of h. The locus 'of points P is thd Squired curve.

(i)

(s, t)

=o

has only one root in

when

t is

given

between zdro and one.


Set

s^t^p

and

st

then

dp^ds+dt and dq
Denoting by

<

and B the integrals in ^


(17)
00

= (1-^M- 2JB,
00

fr,

A* =
=(

Differentiating

V with

respect to a

00

A
.*

3A
t

r^
J

3a

00

where

A,*! JjHilfi

254

00

(A+AJ
00

*,...

(20)

where

A = f
*/ O

*Z-^

(2+

3aj

pa

and similar expression


Consider

for

now

the equation

00
00

0.

00

Then 2 A

-2A -G=| Jf a -^
O
Z

From

this value of

A we

have
oo

-|(8-)pD
The
/.

integral denoted

by

being always positive.

2A

+ 3B
?=
3fi

is

always

positive.

Now we may

write

A.-f-B

=A (l- | *)+ O
member

{ (2A O

+3B

.)

As we have *<1,

the second

of the above equatioi

255
^

is

always positive so that

^
08

is

always negative.
varies in the

Hence ^ is a decreasing function of s. same sense and, therefore, cannot be zero

It

for

more than once

when
same

varies between

its limits, t
t

being a given constant.


s
is

The

is true

with respect to
us put s

when

given.

Now

let

=o

and
CO

5=1 in V we

have

as

It follows therefore that for

a given

t< 1,^=0
Thus

has one and

only one root in 4 between zero and one. two ellipticities is known, so is the other.

if

one of the

Considering s as a function of Ot we will have being defined by

t,

the functional relation

a*

But

-^r

of

< o and
ot

^ o,

hence

~
at

is

negative,
-

Hence

and
for

always vary in opposite directions. For $= t =o, 5=1 and so on for every value of s and t.
*

o,

* =1, and To each pair

of

values of

s\

and

t\

of a and

corresponds another pair viz

256

tx

and

s x since the

expression

^
to

involves
these
b

s,

symmetrically.

The

ellipsoids

corresponding

identical

where merely the a and


of s

two pairs of values are axes have been interchanged.


at

The two values


value, say
tQ,

and

become equal

we have then
00

for s

= =
t

an intermediate
t Q is

c,

where

given

00

x2 dx

Here

A=

(1

+t c x) V 1+x.
in

The curve * (s^^o

the

s(9tf

plane can

now

be traced.

We

notice that

(i)

sj are essentially positive and

lie

between o and

I;

(ii)

(s, <) i*

curve

is

symmetrical with respect to s nd therefore symmetrical with respect

the

to the

bisector of angle
(iii)

If

s~0j

==

and vice verca;


decreases and vice versa; the curve
is

(iv)

If s increases

(v)

Since s-f t
line

L1
1;

always below the straight

s+t

(vi)

Given a value s, of a between zero and one, there is always one and only one value of t corresponding to it. That is, any line through P parallel to Ot
axis meets the curve once

and once

only.

The

coordinates of the point

C situated on
(s,<)

the bisector of the

angle sOt are given by the relation The curve ^ will calculate shortly.

s^i^o^'SSOB, whch we

=o

is

drawn below.

257

/-'-i

-y

We

have now to examine the equation h


t

Taking

as defined

by the

relation

Vfat)

o,

We have

^5

>p

dgsdp
from

substituting for ds,

c?f

dp~ds+dt and dq^tds

-f

Substituting for ^' 8 and

^ t by their
7

values already found

A
Nowsetr*(s
r is
*)

B
2

= j?*

4j,

so

that 2

p dp=dr+4

dq.

always positive, being zero for s^t^tot and varies from o to 1. can consider h as a function of r and q, and g as

We

258

function of r defined
its

by V =o. value in terms of r we have

On

replacing the value of

dp by

- 2p
But

V~ A

c/r+ (4A

+2p B
20

dq.

4A +2;> B ^4A.+2/> B

where

B and

are integrals already defined.

Hence
Since

p dV = A.

rfr

-f (3

y)

efy*=o.

A and B
now

are positive,
,

--2 is always

dT

<o.

consider the equation h / ($,<) giving A in terms of s and f or q and r. shall prove that h varies in

Let us

We
it

the

same sense

as r

and that

is

maximum

for

r=o

or

*=
f
-'

since

from (17),

Y=

(l

p)

B=o.
p 4-

Hence,

c?A(A.' rf*

rfp+rf g )A

+ (1
!

^(A/

rfs

4 A
a,

But

At' dt

fr
*/
.>

"'^"JT?) ^A

(rfp 4-

from values of A'8 A\ already found vide equation


,

(18).

.:dh= (-dp+di)

259

= (-

dp

+ dq)
4-

A
or,

2 , <**=

I (1

a:

>

(I~p+g)a;(2p^ 2 +dr4-4rf2)

>

(21)

(22)

On

taking count of the expression

Hence we have

always-;- <o, since -f- <o.

dr

dr

This proves that h varies in the oppoeite sense with

r.

Tho expression

for

dV

gives
,

if

we make s=
dg

for

=
dh=o

s=/, and from (22) <fro.

Hence

for

dp^dq^o

and therefore from (21)

That

is,

dp^dg^dh^dr^o.

280

Also h varies in
for

the.

inverse sense
for
0;

as

r^o.

Hence h
1

increases

between
is,

and dk is zero 1 and t=t and

decreases for s between /


for

aud

therefore,

maximum

for

s=t

Moreover on replacing dr by
i.

its

value in terms of ds and


(-VV

at

we nave

Cv fi

r-

as

=i

>/

\ /^*6

\~\U

ft

*\t

s) v~r"""" !;] *jc/5

witn

'j.'L

ar

<^.
rf5

.\2Lis-f-iveif
C?5

t>s
,

and

ive

if

t<s.

Hence

when

increases from

to

remains less than

so that h increases

when

s increases.

We now proceed to
1

determire

and A

For s=i=<o the two fundamental equations become


co

=o

a
I

xd*

TOO

xdx

JOT
(

, *

~~ r _x*dx

In order to evaluate these integrals let us put


transform the integrals

= 1+Z 2
'

and

by substituting

!+#=

an(j

then resplacing u by x

r)]

dx

261

Hence,

of*
+/ o I

x*d*
"TV
CC

PS-Jl

The

last intergral

on the right hand side

is

-o.

Also

^=0.
i

r-^

**

^"

=
r-fT*flj"2
*

iz^

Jo

^
already seen

Also for an ellipsoid of revolution, (Maclaurin's spheroid that

wo have

0=

3-H*.

~"

-*
we have
Z

Eliminating A

is

Fqr l=o, the above expressioa

zero.

Its derivative

For Z<1, the derivative is negative, and positive for The function F, which vanishes for l=*o decreases at first and
is

negative, but

when Z>1,

it

begins to increase and tends to


I

It therefore

has only one root for

>!

This root
I

/ is

determined

by the equation

F=a

Substituting for tan

its

expansion in

powers of

in the expression for

h & we have

262

>

The
For an

calculations
ellipsoid

give

= r395,

='3396,

h =*'187l
figure of

of

3 unequal axes
.1871.

to be a pos&ibla

equilibrium-^-^2^ V r

<

This value being lees* than the

corresponding maximum lor Maciaiirin's spheroids, we also have two Maclaurin's ellipsoids of revolution as possible figures
of equilibrium for the

same values
the

of w.

We may

now draw
,

skew curve defined by the

surface

this

and cylinder ^(*/)=0. Recalling the remark made in 7.9 we see that points on curve are obtaiuded by erecting ordinates of the height

A=/(si

A/

(s,i)

at the various points of the plane curve ^($, t)s= fi=i 0%

already diawn,

A being symmetrical with respect to ^V; the skew 'curve is symmetrical with respect to the plane hOs. It is sufficient, to trace the curve on the one side of this plane. therefore,
Starting from point
at this point
ot the

oi
1

the curve V(8,t)=o

= o. we

find that
is

i^o,

and therefore h = o.

Hence A
s

a point

skew curve we are searching.


o

When

decreases
value.

from

to

tv A increases from

to

.1571, its

maximum
To have

This

the ellipsoids figure. point is represnted by corresponding to a given value of w, we merely search for the two points P', P" on this curve in which the horizontal plane at

in the

height h intersects the skew curve.

These

points, P',P" are

263

clearly

symmetrically situated with respect to the plane hOs.

Boths these points give rise to the same ellipsoid as already remarked. Ojily the axes a aud b are interchanged.
tends to zero, the limiting form of equilibrium assumed by the corresponding ellipsoid of Jacobi is very different from that assumed by Maelaurin's spheroid. Here for
to

Remark:

When

w=o we have &=o


the

and s^o or
If

=o, both the values giving


a
is

same

ellipsoid.

s= 0,0=0, and

infinite,

6=0

but

Hence when w tends to zero, the larger axis extends the other two axes shrink indefinitely in such a way indefinitely, that their ratio tends to 1. The form of equilibrium is therefore " a needle shaped figure, sensibly round but infinitely large."
tends to
1.

Poincare called
j

"
it

Figures d Equilibre
7.

Une aiguille allongee d' Une Masse Fluide, page

''

(See Poincare's
162).

10.

Graphical Eepresentation cf the

results.

We have found that

when

w**o, there are 3 figures of equilibrium : The sphere, the u 2 I aiguille elliptic disc, and the elongated needle or
allongee
".

w
_

f 2
.

Spheroids Spheroi of Maclaurin,


Ellipsoi of Jacobi. Ellipsoid

2?rVP

:-o

L
f

.1871
*V-/ I

"

Jk,

I 1 L
f

2
1

Spheroids of Maclaurin,
Ellipsoid of Jacobi, which in the limiting case is an ellipsoid of revolution.

1871 L

~- <.2247
9-7TVP

............. 2

Spheroids of Maclaurin.

964

There are no

ellipsoidal

forms

We

of equilibrium. have already traced the curve which represents Maclaurins' In order to facilitate comspheroids taking I as the variable.
parison with
.

corresponding
.

results

pertaining
^a
f w

to
I

Jaeobi's

ellipsoid let us take the .... * v

same
.

variable s=
-

Hence -

V
^
I*
|

i <

^)

Wtan

We have then

=v-

s varies

is

this

maximum for 3 = 135; lor any other maximum we have two values of 5, viz
The

from o

to

1.

For

o,

o,

and

for s

1,

= o;

value of h less than


op'>

op".

totality of above results is very simply represented The skew curve in Fig. 2 of 7*9 is the line of graphically. Japbi'a ellipsoids and the above curve which lies in the plane fiOs
7'9 represents the line of Maclaurin'fi spheroids. of Fig. 2 of Both the curves are shown in the Fig. below. It is evident

265

that the
is

curve of Jacobi's ellipsoids also an ordinate of the line of Maclaurin's spheroids so that
ordinate
is

maximum
common

GE of the
two

Moving along the line of Maclaurin's spheroids from the point S (which corresThe ellipsoid represented ponds to a sphere) we reach point E.
the
point of the
curves.

by the point E is common spheroids and the series of


"

to

both series

viz.

the series of

ellipsoids.

At E

the series of

Maclaurin's spheroids bifurcates into that of Jacobi's ellipsoids, The e lipaoid represented by E is called the "ellipsoid of bifurca" tion Thus the various forms of equilibrium constitute a con" " linear tinuous series or a series when w is regarded as a
variable parameter, tbe series of Maclaurin's spheroids bifurcating into the series of Jacobi's ellipsoids.

For further

details

reference

may

be made to Appell's

Mecanique Rationnell tome IV, PoiLcare's Figures' d Equilibre rf'une Masse Fluide, and "Astronomy and Cosmogony" by Jeans.

(l)Show
equilibrium for

that

an

elliptic

an

infinite

cylmd-r is a possible form of mass of homogeneous gravitating

the axis of the cylinder liquid, rotating as if rigid, about


If a, b be the semi-axes of the cylinder
velicity, prove that

an

the

angular

a-

a+t>

_ ITZT^ ~ *VP V
be a

'

P being the density of of


cylinder cannot

the

liquid

Deduce that
of
equilibrium

elliptic

possible

form

unless

<1 ^1
[Hint:
i

The

potential V! at au

internal

(2)

point (x>y) is given

%TW + *in
A. solid gravitating sphere of radius a and density

Ex
f* is

surrounded by a gravitating liquid of volume'^ *

266

and density

o.

Show

that the form of the free surface of the

e given liquid is the spheroid of small ellipticity

by

-e P
}

where, e
~--j

15

w2

-^

-^

P 2l being Legendre's

coefficient of the

2nd order.

(I.

8.,

1935)
that

To

solve this problem

we

recall the fact

the

of a liquid of density cr bounded an internal point is given by

by a trarface r = b (l +X,

potential P a ) at

- P" (V L3
V

- )a+ ? ^5 3
X

a6-(
v

y )-

X Pa 1

fc

Hence the

potential of a solid sphere of density P

and of a

homogeneous liquid of density a surrounding the sphere and bounded by the surface r~b U+*> P a )

For equilibrium

to

be possible

'

2
-

*i

2P, *
o

,v

'jsince

must be constant over the surface


Replacing r by b (l-MP a ) and neglecting powers of X 2 higher than unity and products of & and X, this expression reduces to

As
zero.

this

must be a constant, the

coefficient

of

P# must

be

'

Sb

or,

X=
=-

Hence
Bx.

not

fill

homogeneous gravitating fluid just a rigid envelope in the form of an oblate spheroid.
(3<)*:

does

The

fluid is rotating in relative equilibrium

energy E. If it envelope is a free surface of zero pressure. Prove that for all values of E whether greater or less than E lr the tension per
kinetic

round the polar axis with rotates with kinetic energy EI the

unit length across the equatorial section of the envelope is

15

E~E
A
the area of
193ft:
a*

32

where

A
(I.

is

polar section of the ellipsoid.

C, S.
:

Higher Applied Mathematics)


the
ellipticity
its

Ex. (4;

Prove

that

of

nearly
is

spherical mass of
1

liquid rotating round

axis of

symmetry

jfi^p

being the angular velocity

Ex, (5):

Two

and whose

densities

not mix gravitating liquids which do are P; cr(P >o) are enclosed in a rigid
?

spherical envelope*

The whole

rotates

in

relative

equilibrium

268

with a small uniform angular velocity <o about a diameter of the Show that the possible form of common surface of sphere.
the two liquids is an oblate spheroid of ellipticity
-j

Ex.

(6)

subject to its

mass of homogeneous liquid of density P own gravitation and the gravitational attraction
:

If the liquid rotates of any number of other external masses round an axis with angular velocity w, prove that relative be the form assumed equilibrium is impossible, whatever may

w2
by
the liquid mass, if

oiryp

>1
(Poincare's

Theorem)

necessary that the resultant of the forces of attraction and centrifugal force, be directed towards

For equilibrium

to exist

it is

the interior of the liquid, for otherwise a

pan

will be detached.

Let us take the axis of rotation as * axis and o#, oy two other lines perpendicular to it as axes of x and^y respectively, Let V be the potential of the external masses at any point
inside the liqtrid

mass or on

its surface.

Let

V be the

potential

of the liquid mass itself at the


If,

same

point.

therefore,

we

set

the force per unit mass acting at a point of the liquid is At a point on the free surface derivable fro'i, the potential U.
force along the

outward normal
f

is,

therefore,

3U --on

JSow by Green's

heorem

//.

<

the surface integral being taken on the free surface and volume integral through the liquid mass

269

But

o and

V V
a

=4 7r VP, by Poisson's Theorem.

where v
If

is

the volume of the liquid inas?.

l?V;P >1|J/J
*/

A2Urft;

is

clearly

negative.

Hence

^/

f (~ o?i

dS

is

negative.

or,

Tj
)

)dS

isjpositive.

But (

at

any point on the facs surface


outward normal.
It follows

is

the resultant

force acting along the


ftu

from the above,


is

that(

) is positive at

some points of the

surface, that

the

resultant of the forces of attraction

and centrifugal

force at
is

some

points of the free surface acts outward.


impossible,

Equilibrium

therefore

You might also like